You are on page 1of 94

Weekly Optimum Current Q & A PDF – Based On Recent Exam Trends Second Week of January (9th January –16th

of January (9th January –16th January 2024)

National /State News

1. In January as per report Seven products from Odisha, ranging from the Similipal Kai chutney made with red weaver ants to the
embroidered Kapdaganda shawl, have bagged the coveted Geographical Indication (GI) tag in recognition of their exclusivity to the state.
In total, 17 products from different states have been allocated the GI tag, recently.
Which of the following state has the highest number of GI tag as of January 10th, 2024?
A. Odisha
B. Karnataka
C. Tamil Nadu
D. Uttar Pradesh
E. Maharashtra

 Seven products from Odisha,


ranging from the Similipal Kai
chutney made with red weaver
ants to the embroidered
Kapdaganda shawl, have
bagged the coveted
Geographical Indication (GI) tag
in recognition of their
exclusivity to the state.
 In total, 17 products from
different states have been
allocated the GI tag, recently.
 The Department for Promotion
of Industry and Internal Trade,
Ministry of Commerce and
Industry awards GI Tag in India.
 In India, Geographical
Indications registration is
administered by the
Geographical Indications of
Goods (Registration and
Protection) Act, 1999.
 A geographical indication or GI
is a sign used on products that
have a specific geographical
origin and possess qualities or a
reputation that are due to that
origin.
 Geographical Indications are part of the intellectual property rights that comes under the Paris Convention for the Protection of Industrial
Property.
 There are over 500 GI tags as of January 7, 2023. There are 34 classes of products that can get GI tags, ranging from chemicals and paint to
foodstuff, handicrafts, musical instruments, firearms, locomotives etc.
 The products in the GI registry fall under five major categories. Handicrafts dominate the list, with over half the GI tags being given to
products crafted by skilled artisans.
 Every State in India has at least one GI tag which is a proxy for unique cultural items.
 If a State has more GI tags compared to another, it does not necessarily mean that the state is more culturally rich; it just means that more
items have been registered.
 Tamil Nadu (61) has the highest number of GI tags compared to other States.

Click Here For Ultimate Bundle PDF Course 2022| Click Here to Subscribe Our Yearly Mock Test Package Follow Us- Telegram. Facebook , Twitter ,Instagram
Weekly Optimum Current Q & A PDF – Based On Recent Exam Trends Second Week of January (9th January –16th January 2024)

 Uttar Pradesh (56) has the second highest number of GI tags.


 Banaras offers 11 unique crafts and agricultural items (the highest from a single place), including the famous Banarasi paan.
 News Link - Exploring India’s diverse cultural heritage through GI tags | Data - The Hindu

2. In January as per report India's first Inland Waterways Development Council meeting was held in Kolkata.
As per report Minister of Ports Shipping and Waterways Sarbananda Sonowal has told that central government will
invest _________ crore rupees for the development of tourism in waterways.
A. 42,000 Crore
B. 35,000 Crore
C. 45,000 Crore
D. 47,000 Crore
E. 49,000 Crore

 The maiden edition of Inland Waterways Development Council (IWDC) in Kolkata culminated with many firsts in an
effort to ramp up capacity and augment viability of inland waterways of the country.
 The meet was chaired by the Union Minister of Ports, Shipping & Waterways and Ayush, Shri Sarbananda Sonowal.
 The meet, with an objective to enable inland waterways as conduits of economic growth and commerce in the country,
committed an investment ₹45,000 crore for development of river cruise tourism in the country.
 Of this ambitious sum, an estimated ₹35,000 crore has been earmarked for cruise vessels and another ₹10,000 crore for
development of cruise terminal infrastructure at the end of Amrit Kaal, i.e. by 2047.
 To amp up inland waterways for cargo trade, an investment of ₹15,200 crore has been arrived at the Global Maritime
India Summit (GMIS) held in Mumbai in October, 2023.
 At the IWDC, a roadmap was chalked to enable capacity in an additional 26 waterways, fit for River Cruise tourism
from operational strength of 8 waterways.
 The number of cruise circuits with night stays to be increased from 17 to 80 during the same time.
 Building on the capacity of enhanced circuits, the cruise tourism traffic with night stays to be move up from 5,000 to
1.20 lakh by 2047. Similarly, the local cruise tourism traffic on National Waterways without night stay to be increased
from 2 lakh to 15 lakh by 2047.
 The one-day meet was held on board vessel MV Ganges Queen at the Kolkata Dock Complex.
 News Link - Press Information Bureau (pib.gov.in) Govt. Plans 45,000 Crore Investment for Waterway Tourism
(newsonair.gov.in)

3. What is the name of the guidelines launched by Union Minister of Ports, Shipping & Waterways and Ayush, Shri
Sarbananda Sonowal at the inaugural session of IWDC in Kollkata?
A. Harit Naav
B. Harit Nauka
C. Harit Parakh
D. Harit Patshala
E. Harit Manthan

 Shri Sonowal also launched ‘Harit Nauka’ guidelines and ‘River


Cruise Tourism Roadmap, 2047’ at the inaugural session of IWDC in
Kollkata.
 With the launch of 'Harit Nauka – Guidelines for Green Transition
of Inland Vessels,' MoPSW embarks on a journey towards a
sustainable and eco-friendly future for our inland waterways.

Click Here For Ultimate Bundle PDF Course 2022| Click Here to Subscribe Our Yearly Mock Test Package Follow Us- Telegram. Facebook , Twitter ,Instagram
Weekly Optimum Current Q & A PDF – Based On Recent Exam Trends Second Week of January (9th January –16th January 2024)

 The Government, in alignment with its vision to enhance the role of IWT, initiated various measures, including the flagship Jal Marg Vikas
Project (JMVP) for the development of the Ganga-Bhagirathi-Hooghly river system (NW 1).
 Ministry of Ports, Shipping, and Waterways (MoPSW) set ambitious targets, aiming to increase the modal share of IWT from 2% to 5%, as
outlined in the Maritime India Vision 2030.
 There are 46 initiatives identified to develop IWT under the Maritime Amrit Kaal Vision 2047.
 News Link - Press Information Bureau (pib.gov.in)

4. Consider the following statement regarding Swachh Survekshan Awards 2023. Identify the incorrect statement?
1. Indore and Surat were declared the joint cleanest cities of India as part of the Swachh Survekshan Awards 2023 in the category of cities
with a population of more than one lakh.
2. In the category with a population of less than one lakh, Saswad in Maharashtra, Patan in Chhattisgarh, and Lonavala in Maharashtra
secured the top three positions among the cleanest cities.
3. Meanwhile, Maharashtra was given the first prize for cleanliness in the states category, Madhya Pradesh was given the second prize
and Chhattisgarh was given the third clean state award.
4. Varanasi and Prayagraj from Uttar Pradesh took the top two positions among clean Ganga cities.
5. All of the above are correct

 The President of India, Smt. Droupadi Murmu conferred


Swachh Survekshan awards 2023 at Bharat Mandapam, New
Delhi hosted by the Ministry of Housing and Urban Affairs
(MoHUA).
 13 awardees received felicitations under categories of Clean
Cities, Cleanest Cantonment, SafaiMitra Suraksha, Ganga
Towns and Best Performing State were given away.
 This year the cleanest city award showcased joint winners.
 Port city Surat bagged the top honours, alongside Indore,
who had conquered the top spot alone for 6 consecutive
years.
 In the category of cities with a population of less than 1 lakh,
Sasvad, Patan and Lonavala secured the top three spots.
Mhow Cantonment Board in Madhya Pradesh was adjudged
the Cleanest Cantonment Board.
 Varanasi and Prayagraj won the top two awards amongst
the Cleanest Ganga Towns. Maharashtra, Madhya Pradesh
and Chhattisgarh won the top three awards for Best
Performing State.
 Chandigarh walked away with the award for the Best
Safaimitra Surakshit Sheher. 110 awards were bestowed
during the ceremony.
 President of India launched the Swachh Survekshan 2023
dashboard.
 “The theme for the year 2023 “Waste to Wealth” is an
important topic to ponder upon.
 For 2024, the theme is “Reduce, Reuse and Recycle”.

 The 9 years of Swachh Bharat Mission and the spirit of swachhata was reinvigorated in the presence of the Hon’ble President, through an
audio visual presentation highlighting Ek Tareekh Ek Ghanta Ek Saath campaign and the release of the Swachh Bharat Mission Anthem for
2024.
 Playback singer, Kailash Kher has lent his voice to the anthem 'Naya Sankalp Hai, Naya Prakalp Hai'.

Click Here For Ultimate Bundle PDF Course 2022| Click Here to Subscribe Our Yearly Mock Test Package Follow Us- Telegram. Facebook , Twitter ,Instagram
Weekly Optimum Current Q & A PDF – Based On Recent Exam Trends Second Week of January (9th January –16th January 2024)

 Swachh Survekshan is an annual survey of cleanliness, hygiene


and sanitation in cities and towns across India, launched as part
of the Swachh Bharat Abhiyan (specifically under SBA-Urban).
 It was launched by the Ministry of Housing and Urban Affairs
(MoHUA) with Quality Council of India (QCI) as its
implementation partner.
 The SBA was launched (on 2nd October 2014) to make India
clean and free of open defecation by 2nd October 2019.
 The first survey was undertaken in 2016 (covering 73 cities),
which had grown to cover 4242 (by the 2020 survey).
 The methodology for measuring cleanliness rests on two main
criteria – citizen feedback and field assessment.
 News Link - Press Information Bureau (pib.gov.in)

5. Recently as per report Central government has launched a new


solar scheme worth ________crore for electrification in
Particularly Vulnerable Tribal Groups (PVTG) habitations and
villages under the Pradhan Mantri Janjati Adivasi Nyaya Abhiyan
(PM-JANMAN) programme.
A. Rs 435 crore
B. Rs 515 crore
C. Rs 589 crore
D. Rs 658 crore
E. Rs 456 crore

 The Central government has launched a new solar scheme worth Rs 515 crore for electrification in Particularly Vulnerable Tribal Groups
(PVTG) habitations and villages under the Pradhan Mantri Janjati Adivasi Nyaya Abhiyan (PM-JANMAN) programme.
 It will be disbursed over three years, starting with Rs 20 crore in the current financial year, Rs 255 crore in 2024-25, and Rs 240 crore in
2025-26.
 For electrification of houses, the government will provide Rs 50,000 crore per household or the actual cost. For solar lighting, the
government will provide Rs 1 lakh per MPC.
 In a notification dated January 4, the Ministry of New and Renewable Energy (MNRE) said, “The sanction of the President is accorded for
the implementation of a New Solar Power Scheme (for Particularly Vulnerable Tribal Groups (PVTG) Habitation/Villages) under Pradhan
Mantri Janjati Adivasi Nyaya Abhiyan (PM-JANMAN) during 2023-24 to 2025-26.”
 Launched in November 2023, the PM-JANMAN scheme is aimed at improving the socio-economic status of PVTGs by bridging gaps in
health, education, livelihoods, by improving basic infrastructure in Particularly Vulnerable Tribal Group (PVTG) communities, habitations,
and families.
 The Union Cabinet approved PM-JANMAN with an outlay of Rs 24,104 crore to focus on 11 critical interventions through the existing
programmes of nine line ministries.
 The Solar Scheme focuses on electrifying 1 lakh un-electrified households in PVTG areas, identified by the Ministry of Tribal Affairs (MoTA)
across 18 states and 1 Union Territory.
 It will operate during the period from 2023-24 to 2025-26.
 News Link - Centre approves over ₹24,000 crore for tribal solar electrification scheme, ET EnergyWorld (indiatimes.com) Govt launches Rs
515-cr solar scheme for electrification of houses in tribal areas (psuwatch.com)

6. In January as per report Prime Minister Narendra Modi released the first instalment of PM-JANMAN scheme for constructing permanent
houses of poor tribals and said the Scheduled Tribes have been the biggest beneficiaries of the Aspirational Districts Programme.
These tribals will get __________lakh each for constructing a permanent house with electricity, gas connection, piped water and toilet.

Click Here For Ultimate Bundle PDF Course 2022| Click Here to Subscribe Our Yearly Mock Test Package Follow Us- Telegram. Facebook , Twitter ,Instagram
Weekly Optimum Current Q & A PDF – Based On Recent Exam Trends Second Week of January (9th January –16th January 2024)

A. ₹1.5 lakh
B. ₹2.5 lakh
C. ₹3.5 lakh
D. ₹4.5 lakh
E. ₹5.5 lakh

 Prime Minister Narendra Modi released the first instalment of PM-JANMAN scheme for
constructing permanent houses of poor tribals and said the Scheduled Tribes have been the
biggest beneficiaries of the Aspirational Districts Programme.
 He also invoked Mata Shabari, a tribal woman depicted in the Ramayana as an ardent devotee
of Lord Ram, and said the epic is not complete without her.
 PM-JANMAN, an acronym for Pradhan Mantri-Janjatiya Adivasi Nyaya Maha Abhiyan, was
launched on November 15, 2023, which is celebrated as Janjatiya Gaurav Diwas.
 A budget of ₹24,104 crore has been allocated for various welfare schemes under this head for
the benefit of tribals.
 The 100,000 beneficiaries from the particularly vulnerable tribal groups (PVVTGs), for whom
funds were released, will benefit under the PM Awas Yojana-Gramin.
 These tribals will get ₹2.5 lakh each for constructing a permanent house with electricity, gas
connection, piped water and toilet.
 The scheme (comprising Central Sector and Centrally Sponsored Schemes) will be implemented
by the Ministry of Tribal Affairs, in collaboration with the State governments and the PVTG
communities.
 The scheme will concentrate on 11 critical interventions overseen by 9 line Ministries, ensuring
the implementation of existing schemes in villages inhabited by PVTGs.
 News Link - Press Information Bureau (pib.gov.in) PM releases ₹2.5L each for 1L tribals for
building homes - The Economic Times (indiatimes.com)

7. Consider the following statement regarding PM-JANMAN scheme. Which of the following
statement is incorrect?
1. PM Modi released ₹540 crores as the first installment of funding.
2. PM-JANMAN was launched for the socio-economic welfare of Particularly Vulnerable Tribal
Groups (PVTGs) in November 2023.
3. It has a budget of around 24,104 crore rupees.
4. It focuses on 11 critical interventions through nine ministries.
5. All of the above are correct

 PM Modi released the first installment to 1 lakh beneficiaries of


PMAY-G under the PM-JANMAN scheme.
 PM Modi released the first installment of Pradhan Mantri Awas
Yojana – Gramin under the Pradhan Mantri Janjati Adivasi Nyaya
Maha Abhiyan (PM-JANMAN).
 PM Modi released ₹540 crores as the first installment of funding.
 Since the launch of the PM-JANMAN scheme, had sanctioned
projects worth over ₹4,700 crore through 9 ministries under the
PM-JANMAN package.
 PM-JANMAN was launched for the socio-economic welfare of
Particularly Vulnerable Tribal Groups (PVTGs) in November 2023.
 It has a budget of around 24,104 crore rupees. It focuses on 11
critical interventions through nine ministries.
Click Here For Ultimate Bundle PDF Course 2022| Click Here to Subscribe Our Yearly Mock Test Package Follow Us- Telegram. Facebook , Twitter ,Instagram
Weekly Optimum Current Q & A PDF – Based On Recent Exam Trends Second Week of January (9th January –16th January 2024)

 It aims to improve the socio-economic conditions of the PVTGs by


saturating PVTG households and habitations with basic facilities.
 Safe housing, clean water to drink, better access to healthcare,
nutrition, education, energy, road and telecom connectivity, and
chances for sustainable livelihood are some of these facilities.
 Pradhan Mantri Awaas Yojana- Gramin (PMAY-G) was launched by the
Ministry of Rural Development.
 Under the PMAY- G, the beneficiaries are provided financial assistance
of Rs 1.2 lakh in plains and Rs 1.3 lakh in hilly states-Northeastern
states and union territories (UTs) of Jammu & Kashmir and Ladakh-as
well as difficult areas and Integrated Action Plan (IAP) districts.
 Beneficiaries are based on housing deprivation parameters of Socio-
Economic and Caste Census (SECC), 2011 and are subjected to 13-point
exclusion criteria followed by Gram Sabha verification.
 This scheme is meant for people who do not own a house and people
who live in kutcha houses or houses which are severely damaged.
 At present, the minimum size of the houses to be built under the PMAY-
G scheme has been increased to 25 sq. mt. from 20 sq. mt.
 News Link - Press Information Bureau (pib.gov.in) PM-JANMAN-
Operational Guidelines.pdf (tribal.gov.in)

8. In January as per report Ministry of Statistics and Programme Implementation (MoSPI) Rao Inderjit Singh launched the _____________
mobile application for revised fund flow procedure under MPLAD Scheme.
A. e-Sampark
B. e-Sadhan
C. e-Sakshi
D. e-Mitra
E. e-Purna

 The Minister of State (Independent Charge) of the Ministry of Statistics and Programme Implementation (MoSPI) Rao Inderjit Singh
launched the MPLADS e-SAKSHI Mobile Application for revised fund flow procedure under MPLAD Scheme, at Khurshid Lal Bhawan, New
Delhi.
 The objective of MPLAD Scheme is to enable the Members of Parliament (MPs) to recommend works of developmental nature with
emphasis on the creation of durable community assets based on the locally felt needs.
 The launching of e-SAKSHI mobile application for Members of Parliament (MPs) under the MPLAD scheme will bring forth a myriad of
benefits, revolutionizing the way they engage with and manage development projects in their constituencies.
 Additionally, the mobile app will promote transparency by providing MPs with instant updates on the status and progress of their
proposed projects.
 Members of Parliament Local Area Development Scheme (MPLADS) is an ongoing Central Sector Scheme which was launched in 1993-94.
 The Scheme allows the Members of Parliament (MP) to recommend developmental works such as drinking water, education, public health,
sanitation, roads etc, in their constituencies.
 MPs have to recommend at least 15% of the MPLADS funds of the year for areas inhabited by the Scheduled Caste population and 7.5% of
Funds for areas inhabited by the Scheduled Tribes population.
 News Link - Press Information Bureau (pib.gov.in)

9. In Janury as per report Department of Empowerment of Persons with Disabilities (DEPWD) have inked a Memorandum of Understanding
with _________to aggregate employment opportunities for Divyangjans through the innovative PM-DAKSH-DEPWD digital portal.
A. National High Speed Rail Corporation Limited

Click Here For Ultimate Bundle PDF Course 2022| Click Here to Subscribe Our Yearly Mock Test Package Follow Us- Telegram. Facebook , Twitter ,Instagram
Weekly Optimum Current Q & A PDF – Based On Recent Exam Trends Second Week of January (9th January –16th January 2024)

B. Rail Vikas Nigam


C. Indian Railway Finance Corporation
D. Indian Railway Catering and Tourism Corporation
E. National Human Resources Development Network

 In a landmark move, the Department of Empowerment of Persons with Disabilities (DEPWD) and the National Human Resources
Development Network (NHRDN) have inked a Memorandum of Understanding on the closing day of the International Purple Fest in Goa.
 This strategic collaboration aims to aggregate employment opportunities for Divyangjans through the innovative PM-DAKSH-DEPWD
digital portal.
 The objective is to amplify efforts in facilitating meaningful employment opportunities for Divyangjans, fostering a more inclusive and
diverse workforce.
 News Link - Press Information Bureau (pib.gov.in)

10. Recently as per report Government has set a target to reduce Accident Deaths by 50% by ___________.
A. 2025
B. 2026
C. 2030
D. 2032
E. 2035

 Union Minister for Road Transport and Highways Shri Nitin Gadkari said Road Safety is
Top-most priority of the Government with target to reduce Accident Deaths by 50% by
2030.
 Addressing CII National Conclave on ‘Road Safety - Indian Roads@2030 - Raising the Bar
of Safety’ Shri Gadkari said change in social behaviour is very important along with
focusing on strengthening ‘4Es of Road Safety’ -Engineering (Road & Vehicle Engineering)
- Enforcement - Education and Emergency Medical Service.
 As per the latest Report on Road Accidents 2022 there have been 4.6 Lakh road accidents,
1.68 Lakh deaths and 4 Lakh serious injuries.
 There has been increase in Road Accidents by 12% , increase in Road Accidents Deaths by
10% resulting in socio-economic loss of 3.14 % to GDP .
 News Link - Press Information Bureau (pib.gov.in)

11. In January as per report 5 Lok Sabha MPs have been selected for the Sansad Ratna
Awards. Which of the following is not in the list?
A. Sukanta Majumdar
B. Shrikant Eknath Shinde
C. Anurag Thakur
D. Sudhir Gupta
E. Amol Ramsing Kolhe

 Five Lok Sabha MPs, including Sukanta Majumdar of BJP and Shrikant Eknath Shinde of Shiv Sena, have been selected for the Sansad Ratna
Awards, which were announced on 7 January.
 BJP's Sudhir Gupta, NC's Amol Ramsing Kolhe, and Congress's Kuldeep Rai Sharma are the other three MPs selected for this award.
 The awards will be presented at a function on 17 February.

Click Here For Ultimate Bundle PDF Course 2022| Click Here to Subscribe Our Yearly Mock Test Package Follow Us- Telegram. Facebook , Twitter ,Instagram
Weekly Optimum Current Q & A PDF – Based On Recent Exam Trends Second Week of January (9th January –16th January 2024)

 The awards are based on comprehensive performance, with nominees selected by a jury committee headed by Minister of Law and
Parliamentary Affairs Arjun Ram Meghwal and former Chief Election Commissioner of India TS Krishnamurthy.
 Sansad Ratna Awards are presented annually to top-performing MPs on the basis of performance.
 The Sansad Maha Ratna Awards are presented once in five years for consistency in performance in the Lok Sabha.
 The Sansad Ratna Awards were instituted on the suggestion of Dr. APJ Abdul Kalam to recognize outstanding parliamentarians.
 In 2010, the first edition of the award ceremony was hosted in Chennai.
 News Link - Sansad Ratna Awards: 5 LS MPs selected for Sansad Ratna Awards | Delhi News - Times of India (indiatimes.com)

12. In January as per report India and _________ signed a Letter of Arrangement (LoA) on defence collaboration in Research and Development.
A. Japan
B. UK
C. Egypt
D. Guyana
E. USA

 India and U.K signed a Letter of Arrangement (LoA) on defence


collaboration in Research and Development.
 The LoA on R&D was signed between India’s Defence Research and
Development Organisation (DRDO) and the UK’s Defence Science and
Technology Laboratory (DSTL).
 It was signed after the bilateral meeting held between Defence
Minister Rajnath Singh and his British counterpart Grant Shapps in
London.
 Both leaders discussed a range of defence, security and cooperation
matters with particular emphasis on enhancing defence-industrial
collaboration.
 It is the first visit by an Indian Defence Minister to the U.K. in the last
two decades.
 Shri Rajnath Singh was present at the ceremonial Guard of Honour at
the Horse Guards Parade Ground before the bilateral meeting with his
UK counterpart.
 News Link - pib.gov.in/PressReleaseIframePage.aspx?PRID=1994638

13. In January as per report ___________ has become the first state in India to map all 784 accident black spots on the Mappls App, a
navigation system developed by MapMyIndia.
A. Uttar Pradesh
B. Punjab
C. Karnataka
D. Odisha
E. Haryana

 Punjab has become the first state in India to map all 784 accident black spots on the Mappls App, a navigation system
developed by MapMyIndia.
 Punjab Police team-up with MapmyIndia for real-time traffic updates and alerts on Mappls App about Black and Blind spots.
 This initiative comes as part of the preparations for the launch of Chief Minister Bhagwant Singh Mann's flagship project, the 'Sadak
Suraksha Force'.

Click Here For Ultimate Bundle PDF Course 2022| Click Here to Subscribe Our Yearly Mock Test Package Follow Us- Telegram. Facebook , Twitter ,Instagram
Weekly Optimum Current Q & A PDF – Based On Recent Exam Trends Second Week of January (9th January –16th January 2024)

 MapmyIndia is an Indian technology company that builds digital map data, telematics services, location-based SaaS and GIS AI
technologies. The company was founded in 1995 and is headquartered at New Delhi.
 Founders: Rakesh Verma, Rashmi Verma.
 News Link - Punjab app makes history by mapping all accident spots | Chandigarh News - The Indian Express

14. In January as per report Odisha government has signed an MoU with the National Dairy Development Board (NDDB) for establishing a
facility for production of anthrax and ENT vaccines in _________
A. Puri
B. Balasore
C. Cuttack
D. Debagarh
E. Ganjam

 The Odisha government has signed an MoU with the National Dairy Development Board (NDDB) for establishing a facility for production of
anthrax and ENT vaccines in Ganjam district.
 The agreement for the Rs 52-crore facility to be set up at the satellite unit of Odisha Biological Products Institute in Berhampur.
 The Good Manufacturing Practice (GMP) laboratory will comply with WHO norms and the Drugs and Cosmetics Act 1940 of India. With the
setting up of the new laboratory, Odisha can produce 2 crore doses of ENTV and 50 lakh doses of ASV in a year.
 The National Dairy Development Board is a statutory body set up by an Act of the Parliament of India.
 The main office is in Anand, Gujarat. Chairman: Meenesh Shah.
 News Link - Odisha govt signs pact with NDDB for vaccine production (ptinews.com)

15. Recently as per report Uttar Pradesh government is setting up a pharma park in
_________ covering 1,472-acre across five villages.
A. Rampur
B. Lalitpur
C. Largan
D. Saidpur
E. Gadolikala

 The Uttar Pradesh government is setting up a pharma park in Lalitpur, Bundelkhand, covering 1,472-acre across
five villages, according to the Uttar Pradesh State Industrial Development Authority (UPSIDA).
 The plan will unfold in two phases, with an initial focus on immediate development efforts covering 300 acre of
land.
 The villages identified for the survey include Saidpur (426 acre), Gadolikala (249 acre), Largan (239 acre),
Karounda (116 acre), and Rampur (441 acre).
 The park is expected to see an infusion of about ₹8,000 crore by the State.
 UPSIDA is also taking steps to expedite development projects at site-1 and site-2 in Orai, along with the Plastic
City project in Dibiyapur.
 The plastic park, spread across 274.4 acre, encompasses industrial units, residential areas, and basic infrastructure.
 News Link - Uttar Pradesh to set up pharma park - The Hindu BusinessLine

16. In January as per report __________ government launched 'Baayu,' an app-based electric bike taxi service that is an innovative and
forward-thinking sustainable mobility initiative of the state government.
A. Uttar Pradesh
B. Odisha

Click Here For Ultimate Bundle PDF Course 2022| Click Here to Subscribe Our Yearly Mock Test Package Follow Us- Telegram. Facebook , Twitter ,Instagram
Weekly Optimum Current Q & A PDF – Based On Recent Exam Trends Second Week of January (9th January –16th January 2024)

C. Assam
D. Karnataka
E. Maharashtra

 The Assam government launched 'Baayu,' an app-based electric bike taxi service that is an innovative and forward-thinking sustainable
mobility initiative of the state government-run Assam State Transport Corporation (ASTC).
 The app-based electric bike taxi service was flagged off by Assam Minister of Transport Parimal Suklabaidya in the state capital Guwahati.
 The event witnessed the unveiling of India's first app-based 100% electric and decentralized bike taxi services.
 Baayu (an App based Electric Bike Taxi Services) by ASTC in collaboration with Bikozee Ecotech a startup from Assam is launching India's
first 100% electric and decentralized bike taxi services.
 It is Supported by NITI Aayog's Shoonya Zero pollution mobility campaign.
 News Link - Assam launches India's first 100 pc Electric Bike Taxi Service via 'Baayu' - The Economic Times (indiatimes.com)

17. Recently in January Scheme and Web-portal (http:/www.pmawards.gov.in) for Prime Minister’s Awards for Excellence in Public
Administration 2023 was launched on 8th January, 2024.
The Prize money for PM’s Awards 2023 will be ______.
A. Rs.10 lakh
B. Rs.30 lakh
C. Rs.15 lakh
D. Rs.25 lakh
E. Rs.20 lakh

 The Scheme and Web-portal (http://www.pmawards.gov.in) for Prime Minister’s Awards for Excellence in Public Administration 2023 was
launched on 8th January, 2024 at 11.00 AM by the Department of Administrative Reforms & Public Grievances.
 Under the leadership of Prime Minister Narendra Modi, the entire concept and format of the PM's Excellence Award has undergone a
revolutionary change since 2014.
 The objective of the Scheme is to encourage Constructive Competition, Innovation, Replication and Institutionalization of Best practices.
 The Award Scheme has now, this year, been restructured to recognize the performance of the District Collector through targeted individual
beneficiaries and implementation with a saturation approach With this focus, the applications for Awards would be evaluated on three
parameters, Good Governance, Qualitative and Quantitative.
 The PM’s Awards, 2023 will consist of: (i) Trophy, (ii)Scroll and (iii) an incentive of Rs. 20 lakh to the awarded District/organization to be
utilized for implementation of project/programme or bridging resources gaps in any area of public welfare.
 News Link - Press Information Bureau (pib.gov.in)

18. Which of the following is participating in Bharat Tex as the Partner State?
A. Odisha
B. Uttar Pradesh
C. Tamil Nadu
D. Telangana
E. Karnataka

 The Union Minister for Textiles, Consumer Affairs, Food and Public Distribution and Commerce and Industry, Shri Piyush Goyal took review
meeting of the steering committee in view of the upcoming mega textiles event- Bharat Tex 2024.
 Bharat Tex is scheduled from 26th to 29th Feb, 2024 at Bharat Mandapam and Yashobhoomi in New Delhi. Bharat Tex 2024 is a global
textile mega event organized by 11 Textile EPCs and supported by the Ministry of Textiles.
 Aditya Birla Group and Reliance Industries Ltd. are participating as Platinum and Gold partners respectively.

Click Here For Ultimate Bundle PDF Course 2022| Click Here to Subscribe Our Yearly Mock Test Package Follow Us- Telegram. Facebook , Twitter ,Instagram
Weekly Optimum Current Q & A PDF – Based On Recent Exam Trends Second Week of January (9th January –16th January 2024)

 Uttar Pradesh is participating in Bharat Tex as the Partner State and Madhya Pradesh as the Focus State.
 News Link - Press Information Bureau (pib.gov.in)

19. Recently seen in news Yogyasree has been launched by which of the following state government for the Scheduled Caste and Scheduled
Tribe students for various entrance and competitive examinations?
A. West Bengal
B. Jharkhand
C. Odisha
D. Bihar
E. Tamil Nadu

 The 'Yogyasree' was launched by CM Mamata Banerjee for SC/ST students in West Bengal.
 Under the name of "Yogyasree", a free training module has been introduced for the Scheduled Caste and Scheduled Tribe students in West
Bengal for various entrance and competitive examinations.
 CM introduced a toll-free anti-ragging number for the students and said that notification should be sent to all colleges and universities.
 On 8 January, the Chief Minister has launched this scheme at a program in Kolkata.
 “Fifty centers will be opened across the state where SC and ST students will be able to take full training for competitive examinations.
 She also started a student internship program, where 2500 students will receive such an internship.
 She spoke about the various social welfare schemes introduced for students, including Kanyasree, Sabujshree, and the Student's Credit
Card scheme.
 News Link - Mamata Banerjee launches 'Yogyasree' for SC/ST students in West Bengal, unveils anti-ragging helpline - The Economic Times
(indiatimes.com)

20. Recently seen in news Mukhyamantri Mahila Udyamita Abhiyaan is associated with which of the following state?
A. Andhra Pradesh
B. Odisha
C. Karnataka
D. Uttar Pradesh
E. Assam

 Assam government has announced a new financial assistance scheme "Mukhyamantri Mahila
Udyamita Abhiyaan" for women entrepreneurs in rural areas, but for this, the government has
set a condition to avail its benefits.
 The Assam government announced that women belonging to general and OBC categories should
not have more than three children if they want to avail the benefits of the scheme.
 At the same time, for Scheduled Tribe (ST) and Scheduled Caste (SC) women, this limit is four
children.
 The Moran, Motok, and 'Tea tribes', who are demanding ST status, have also been imposed a
limit of four children.
 This is in line with his announcement in 2021 that the state government will have a two-child
policy to avail benefits under specific state-funded schemes.
 The scheme aims to help women who are part of self-help groups in rural parts of the state to
develop as "rural micro-entrepreneurs", with a target of an annual income of Rs 1 lakh for each
member.
 The government will provide them with Rs 10,000 in the first year if they meet the basic eligibility criteria.
 Depending on whether they use this money or not, they will be given a loan of Rs 12,500 by the government and Rs 12,500 from the bank
in the next two years.

Click Here For Ultimate Bundle PDF Course 2022| Click Here to Subscribe Our Yearly Mock Test Package Follow Us- Telegram. Facebook , Twitter ,Instagram
Weekly Optimum Current Q & A PDF – Based On Recent Exam Trends Second Week of January (9th January –16th January 2024)

 Apart from a limit on the number of children, beneficiaries must also fulfill two other conditions.
 If they have girls, they should be enrolled in school. If the girl is not of school-going age, the woman must sign an affidavit that she will be
enrolled in school when the time comes.
 Also, the trees they planted last year under the government's tree plantation drive, Amrit Vriksha Andolan, should survive.
 News Link - Assam Extends Welfare Schemes to Small Families for Population Control (newsonair.gov.in)

21. Consider the following statement regarding PM-eBus Sewa Scheme. Which of the following statement is incorrect?
1. Under the PM-eBus Sewa Scheme, 10,000 e-buses will be deployed across cities in the country.
2. This scheme will support bus operations for 20 years.
3. It has been allocated a total funding of Rs 57,613 crore.
4. Out of this financial provision, the central government will contribute Rs 20,000 crore, while the remaining portion will be covered by
the state governments.
5. It will cover cities with a population of 300,000 and above and will include all the capital cities of the ‘Union Territories, North Eastern
Region, and Hill States’.

 The Union Housing and Urban Affairs Minister recently said that the Ministry has floated tenders for procuring buses under the PM-eBus
Sewa scheme.
 The buses are in three different sizes — 7 metre, 9 metre and 12 metre.
 Under the PM-eBus Sewa Scheme, 10,000 e-buses will be deployed across cities in the country.
 The scheme was introduced to give a push to the slow adoption of e-buses in public transport.
 Under this scheme, city bus operations will be done on a Public Private Partnership (PPP) model.
 This scheme will support bus operations for 10 years. The government said that priority will be given to the cities which do not have an
organised bus service.
 States/Cities shall be responsible for running the bus services and making payments to the bus operators.
 The Central Government will support these bus operations by providing subsidies to the extent specified in the proposed scheme.
 It has been allocated a total funding of Rs 57,613 crore.
 Out of this financial provision, the central government will contribute Rs 20,000 crore, while the remaining portion will be covered by the
state governments.
 It will cover cities with a population of 300,000 and above
and will include all the capital cities of the ‘Union
Territories, North Eastern Region, and Hill States’.
 There are two segments of the scheme: Augmenting the
city bus services and Associated Infrastructure, and Green
Urban Mobility initiatives.
 Augmenting the city bus services and Associated
Infrastructure: Under this segment, the scheme will
augment city electric bus operations by extending Central
Assistance (CA) for 10,000 electric bus operation on Public
Private Partnership (PPP) model, bus depots and behind-
the-meter power infrastructure. In 169 cities, 10,000 e-
buses will be deployed using public-private partnership
(PPP) model.
 Green Urban Mobility Initiatives (GUMI): Under this segment, the scheme will support the implementation of GUMI projects for
complementing bus services and demonstrating a reduction in Greenhouse Gas (GHG) emissions in urban areas. In 181 other cities,
infrastructure will be upgraded under the green urban mobility initiatives.
 The segment will include support for implementing the National Common Mobility Card (NCMC)-based Automatic Fare Collection System
(AFCS), bus priority infrastructure, etc.
 News Link - Tenders for e-buses floated under PM-eBus Sewa: Union Urban Affairs Minister - The Hindu

Click Here For Ultimate Bundle PDF Course 2022| Click Here to Subscribe Our Yearly Mock Test Package Follow Us- Telegram. Facebook , Twitter ,Instagram
Weekly Optimum Current Q & A PDF – Based On Recent Exam Trends Second Week of January (9th January –16th January 2024)

22. In January as per report ___________ has been designated as India's first Dark Sky Park and the fifth in Asia, protecting the night sky and
preventing light pollution.
A. Bandhavgarh National Park
B. Pench Tiger Reserve
C. Kanha Tiger Reserve
D. Tadoba-Andhari Tiger Reserve
E. Panna National Park

 The Pench Tiger Reserve in Maharashtra has been


designated as India's first Dark Sky Park and the fifth
in Asia, protecting the night sky and preventing light
pollution.
 The Pench Tiger Reserve (PTR) in Maharashtra is
India's first Dark Sky Park within a tiger reserve for
earmarking areas around the park that restrict light
pollution for stargazers to access pristine dark skies.
 The Dark and Quiet Skies for Science and Society Working Group, led by the International Astronomical Union, has recommended the
establishment of 'Dark Sky Oases' by national and local governments.
 PTR became the fifth such park in Asia.
 The certification was given by the International Dark-Sky Association, a global dark-sky movement to promote astronomy.
 Pench Tiger Reserve or Pench National Park is one of the premier tiger reserves of India and the first one to straddle across two states -
Madhya Pradesh and Maharashtra.
 A dark-sky preserve is an area, usually surrounding a park or observatory, that restricts artificial light pollution. The purpose of the dark-
sky movement is generally to promote astronomy.
 The Indian Astronomical Observatory (IAO), a high-altitude astronomy station located in Hanle, and operated by the Indian Institute of
Astrophysics, is India's first dark-sky preserve.
 Situated in the Western Himalayas at an elevation of 4,500 meters
(14,764 ft), the IAO is one of the world's highest located sites for
optical, infrared and gamma-ray telescopes.
 News Link - Maharashtra Pench Tiger Reserve becomes India's first
Dark Sky Park. All you need to know | Mint (livemint.com)

23. In January as per report India’s Oldest Living City has been found in
______________.
A. Dholera
B. Dholavira
C. Vadnagar
D. Gola Dhoro
E. Lothal

 An archaeological excavation at Gujarat's Vadnagar has discovered


evidence of human settlement dating back to 1,400 BCE.
 The excavation was done by a team of scientists from IIT Kharagpur,
Archaeological Survey of India, Physical Research Laboratory,
Jawaharlal Nehru University and Deccan College
 The study was funded by Infosys Foundation and Gujarat
government's Directorate of Archaeology & Museums that is
entrusted with building India's first experiential digital museum at
Vadnagar.
Click Here For Ultimate Bundle PDF Course 2022| Click Here to Subscribe Our Yearly Mock Test Package Follow Us- Telegram. Facebook , Twitter ,Instagram
Weekly Optimum Current Q & A PDF – Based On Recent Exam Trends Second Week of January (9th January –16th January 2024)

 The excavation at Vadnagar was carried out from 2016 till early 2023.
 The Archaeological Survey of India (ASI) was established in the year 1861 by Alexander Cunningham.
 After independence, it was established as a statutory body under the Ancient Monuments and Archaeological Sites and Remains Act, 1958.
 Concerned Ministry: Ministry of Culture
 News Link - India’s oldest living city found in PM Modi’s native village Vadnagar: multi-institution study - The Hindu

24. Recently Government of India has announced the “Rastriya Vigyan Puraskar” in the field of Science, Technology and Innovation.
Which of the following is not among the categories of the Rastriya Vigyan Puraskar?
A. Vigyan Ratna
B. Vigyan Shri
C. Vigyan Yuva
D. Vigyan Team
E. Vigyan Padma

 The Government of India has announced the “Rastriya Vigyan Puraskar”


in the field of Science, Technology and Innovation.
 The National Award recognizes outstanding and inspiring scientific,
technological and innovation contributions of researchers, technologist
and innovators.
 The awards shall be given in following four categories:
 Vigyan Ratna (VR): Maximum of three awards to be bestowed to
recognize life time achievements & contributions made in given field of
Science and Technology.
 Vigyan Shri (VS): Maximum of 25 awards to be given to recognize the
distinguished contributions in given field of Science and Technology.
 Vigyan Yuva: Shanti Swarup Bhatnagar (VY-SSB) award: Maximum of 25
awards to be given to recognize and encourage young scientists who
made an exceptional contribution in given field of Science and Technology.
 Vigyan Team (VT) award: Maximum of three awards may be awarded to a
team comprising of three or more scientists/researchers/innovators who
have made an exceptional contribution working in a team in given field of
Science and Technology.
 The Rashtriya Vigyan Puraskar shall be given in the following 13 domains,
namely: Physics, Chemistry, Biological Sciences, Mathematics & Computer
Science, Earth Science, Medicine, Engineering Sciences, Agricultural
Science, Environmental Science, Technology & Innovation, Atomic Energy,
Space Science and Technology, and Others.
 The awards this year are being coordinated by Council of Scientific and Industrial Research (CSIR) under the Ministry of Science and
Technology.
 The awards shall be announced on 11th May 2024 (National Technology Day). The Award Ceremony for all categories of awards will be
held on 23rd August 2024 (National Space Day).
 News Link - pib.gov.in/PressReleaseIframePage.aspx?PRID=1995945

25. What is the current mandatory minimum wage rate applicable across the country?
A. ₹176
B. ₹154
C. ₹196
D. ₹202

Click Here For Ultimate Bundle PDF Course 2022| Click Here to Subscribe Our Yearly Mock Test Package Follow Us- Telegram. Facebook , Twitter ,Instagram
Weekly Optimum Current Q & A PDF – Based On Recent Exam Trends Second Week of January (9th January –16th January 2024)

E. ₹120

 The central government could


fix a higher mandatory
minimum wage rate applicable
across the country ahead of the
general elections, adopting
recommendations of a high-
level expert panel looking into
the matter.
 An expert committee headed by
SP Mukherjee, set up in 2021 for
a period of three years till June
2024, is expected to submit its
report soon, and the new floor
wage could be notified ahead of
the elections likely to be held
during April-May.
 The floor wage, currently at ₹176 per day, was last revised in 2017 and is not statutory for states.
 A committee headed by Anoop Satpathy had proposed a floor wage of ₹375 per day in 2019 but the same was not accepted by the
government because of the financial implications for employers, including the government as it was over 100% higher than the existing
floor wage.
 There has to be a balance between the existing₹176 per day and ₹375 per day as recommended by the Satpathy committee.
 As per the Code on Wages, 2019, the government can fix different floor wages for different geographical areas but it does not allow
lowering of wage in case the minimum rates of wages fixed by the appropriate government (states or the Centre) is more than the existing
floor wage.
 News Link - minimum wages: Minimum wages may be hiked before polls - The Economic Times (indiatimes.com)
H256aEpVjKylMPz123CrkGG7wj1M3wyNTujDhM9O.pdf (ficci.in)

26. Recently as per report which of the following city will be developed as the India's first vastu-based township?
A. Bhopal
B. Jodhpur
C. Lucknow
D. Ayodhya
E. Indore

 The Uttar Pradesh government has planned a 1,000-acre township in Ayodhya which will be a blend of contemporary and traditional
architectural elements.
Click Here For Ultimate Bundle PDF Course 2022| Click Here to Subscribe Our Yearly Mock Test Package Follow Us- Telegram. Facebook , Twitter ,Instagram
Weekly Optimum Current Q & A PDF – Based On Recent Exam Trends Second Week of January (9th January –16th January 2024)

 The 'New Ayodhya', for which the state government has already secured land, will be India's first vastu-based township.
 New Ayodhya City is going to be a river-centric city with a focus on sustainability.
 News Link - UP plans 1,000-acre township in temple town Ayodhya - The Economic Times (indiatimes.com)

27. Recently as per report idol of 'Ram Lalla' carved by Karnataka's renowned sculptor _________ will be installed in the Ram temple in
Ayodhya.
A. Abhishek Mridul
B. Arun Yogiraj
C. Chandrakant B Sompura
D. Kailesh Choubey
E. Ashish Mishra

 Arun Yogiraj, a distinguished sculptor hailing from a lineage of five generations of renowned sculptors in Mysore, has been honoured with
the selection of his idol for installation in Ayodhya's Ram Temple.
 The idol for which 'Pran Pratishtha' will be done is expected to be around 150-200kgs.
 The 51-inch tall idol will be made of black stone and will be placed in 'Garbh Griha' of the temple.
 News Link - Ram Lalla statue by Karnataka sculptor Arun Yogiraj selected for installation at Ram temple - The Economic Times
(indiatimes.com)

 The grand Ram Temple in Ayodhya is a well-thought-out, scientifically executed and marvellous projection of Indian traditional heritage.
 It was designed by celebrated architect Chandrakant B Sompura, with the help of his son Ashish, and incorporated his vision, which he
outlined 30 years back.
 Standing on 2.7 acres of land in the temple town, the temple is 161 feet tall, 235 feet wide, and has a total length of 360 feet.
 It has been constructed in one of the two distinguishing temple-building styles from ancient India – Nagara – following all the Vedic rituals
with an amalgamation of modern technology.
 During the Gupta Period, from where the Nagara style emerged, the use of iron or steel was not prevalent in the construction of temples.
 The durability of iron is roughly 80-90 years.
 The temple is built using granite, sandstone, and marble with a lock and key mechanism, which ensures a lifespan of up to 1,000 years.
Notably, no cement or mortar has also been utilised in its construction.
 A special ‘Surya Tilak’ mirror, a lens-based apparatus, was designed by a team of scientists from CBRI and the Indian Institute of
Astrophysics (IIA). It will be used for a ceremonial anointment of Lord Ram on every Ram Navami day at noon with the sunlight on the
idol's forehead.
 News Link - Built with no steel or iron, Ram Mandir is an architectural marvel. Know why | Latest News India - Hindustan Times

Click Here For Ultimate Bundle PDF Course 2022| Click Here to Subscribe Our Yearly Mock Test Package Follow Us- Telegram. Facebook , Twitter ,Instagram
Weekly Optimum Current Q & A PDF – Based On Recent Exam Trends Second Week of January (9th January –16th January 2024)

28. Recently as per report government is set to roll out a human papillomavirus (HPV) vaccination campaign for girls in the _________ years
age group.
A. 6-14 years
B. 9-14 years
C. 7-15 years
D. 9-17 years
E. 9-15 years

 IN A significant step that can reduce the incidence


of cervical cancer — the second most common
cancer in women in India — the government is set
to roll out a human papillomavirus (HPV)
vaccination campaign for girls in the 9-14 years age
group.
 The immunisation drive, planned in three phases
over three years, is likely to start from the second
quarter of this year.
 Besides cervical cancer, the vaccine also offers
protection against the HPV strains that cause
cancer of the anus, vagina and oropharynx.
Additionally, it also protects against the HPV strains
that are responsible for genital warts.
 Currently, the two-dose HPV vaccine is available
commercially for about Rs 2,000 per dose. But once
the government includes it in its immunisation
programme, it will be available for free.
 India accounts for about a fifth of the global burden for cervical cancer, recording about 1.25 lakh cases and about 75,000 deaths each
year.
 Cervavac, an indigenously developed quadrivalent vaccine by the Serum Institute of India (SII) in Pune, is already available commercially,
the company is in the process of scaling up its manufacturing to meet government requirements.
 News Link - https://pib.gov.in/PressReleaseIframePage.aspx?PRID=1995855

29. In January as per report Directorate General of Civil Aviation (DGCA) has increased weekly rest periods from 36 hours to ___________
hours for flight crew.
A. 40 hours
B. 45 hours
C. 42 hours
D. 50 hours
E. 48 hours

 Directorate General of Civil Aviation (DGCA) has increased


weekly rest periods from 36 hours to 48 hours for flight crew.
 As per the revised flight duty time rules issued by DGCA, night
duty will now cover the period from midnight to 6 am in the
morning as compared to five am earlier.
 The maximum flight time and maximum flight duty period for
flight operations encroaching night have been restricted to 8
hours flight time and 10 hours flight duty period respectively.

Click Here For Ultimate Bundle PDF Course 2022| Click Here to Subscribe Our Yearly Mock Test Package Follow Us- Telegram. Facebook , Twitter ,Instagram
Weekly Optimum Current Q & A PDF – Based On Recent Exam Trends Second Week of January (9th January –16th January 2024)

 DGCA said that the changes in the regulations have been done to address pilot fatigue, enhance overall flight safety and balance growth in
aviation sector.
 According to the new rules, the number of landings has been limited to only two landings as compared to maximum permissible six
landings under previous regulations during night operations.
 Airline operators are required to comply with revised regulations latest by 1st june, this year.
 DGCA further said that it aims to transition towards a new regime of Fatigue Risk Management System (FRMS) that will be a data-driven
approach to enhance monitoring of flight crew fatigue.
 News Link - DGCA Implements New Rules Flight Duty Time Rules for Flight Crew (newsonair.gov.in)

30. What is the length of the Atal Bihari Vajpayee Sewari - Nhava Sheva Atal Setu, which is the longest bridge in India and also the longest sea
bridge in India?
A. 25.6 km
B. 23.4 km
C. 21.8 km
D. 29.4 km
E. 25.4 km

 PM Modi will inaugurate and lay


the foundation stone of
development projects worth over
30,500 crore rupees in
Maharashtra.
 He will inaugurate Atal Bihari
Vajpayee Sewari - Nhava Sheva
Atal Setu.
 Atal Setu has been built at a total
cost of more than 17,840 crore
rupees.
 It is about a 21.8 km long 6-lane
bridge having about 16.5 km
length over the sea and about
5.5 km on the land.
 Its foundation stone was laid by PM Modi in December 2016.
 Atal Setu is the longest bridge in India and also the longest sea bridge in India. Atal Setu was built by Larsen & Toubro (L&T)
 PM Modi will also lay the foundation stone of the underground road tunnel connecting Eastern Freeway's Orange Gate to Marine Drive.
 It is a 9.2 km tunnel that will be built at a cost of more than 8,700 crore rupees. It will reduce travel time between Orange Gate and Marine
Drive.
 PM Modi will dedicate phase 1 of the Surya regional bulk drinking water project to the Nation.
 It is developed at a cost of more than 1975 crore rupees. It will provide drinking water supply to the Palghar and Thane districts of
Maharashtra.
 PM Modi will dedicate about 2,000 crore rupees worth of railway projects to the nation.
 Modi will also inaugurate 'Bharat Ratnam' (Mega Common Facilitation Centre) for the gems and jewellery sector at Santacruz Electronic
Export Processing Zone- Special Economic Zone (SEEPZ SEZ) in Mumbai, which is first of its kind in India with best available machines in the
world, including 3D Metal printing.
 He will launch Namo Mahila Shashaktikaran Abhiyaan.
 This Abhiyaan is aimed at empowering women in Maharashtra by providing them with skill development training and exposure to
entrepreneurship development.
 News Link - Press Information Bureau (pib.gov.in)

Click Here For Ultimate Bundle PDF Course 2022| Click Here to Subscribe Our Yearly Mock Test Package Follow Us- Telegram. Facebook , Twitter ,Instagram
Weekly Optimum Current Q & A PDF – Based On Recent Exam Trends Second Week of January (9th January –16th January 2024)

31. In January as per report PM Modi inaugurated the 10th edition of the Vibrant Gujarat Global Summit in Gandhinagar.
What is the theme of the tenth edition of the Vibrant Gujarat Global Summit?
A. India Development Road
B. Road to India Future
C. Gujarat Going Global
D. Gateway to the Future
E. Shaping a new India

 On 10th January 2024, PM Modi inaugurated the Vibrant Gujarat Global Summit at Mahatma Mandir in Gandhinagar.
 UAE President Mohamed bin Zayed Al Nahyan, President of Mozambique Filipe Nyusi and President of Timor Leste José Ramos-Horta
attended the summit.
 Prime Minister Modi will also attend a meeting with CEOs of top global corporations.
 The Global FinTech Leadership Forum will be also organized at the GIFT City.
 'Gateway to the Future' is the theme of the tenth edition of the Vibrant Gujarat Global Summit.
 The main focus of the summit will be on sectors like semiconductors, green hydrogen, e-mobility, and space manufacturing.
 A series of seminars on globally relevant topics like Industry 4.0, Technology and Innovation, and renewable energy will be also organized.
 There are 34 Partner countries and 16 Partner organizations for this year’s Summit. Around 200 global CEOs attended the summit.
 News Link - pib.gov.in/PressReleaseIframePage.aspx?PRID=1993937

32. Whom among the following is the chief guest at the 10th edition of Vibrant Gujarat Global Summit?
A. Mozambique President Filipe Jacinto Nyusi
B. United Arab Emirates President Mohamed bin Zayed Al Nahyan
C. Prime Minister of the Czech Republic, H.E. Mr. Petr Fiala
D. Timor-Leste President Jose Ramos-Horta
E. None of the above

 Prime Minister Narendra Modi and United Arab Emirates President


Mohamed bin Zayed Al Nahyan held a bilateral meeting in Gandhinagar on
January 9.
 Al Nahyan is the chief guest at the 10th edition of Vibrant Gujarat Global
Summit.
 This was the fourth meeting between Modi and Al Nahyan in less than seven
months.
 Four MoUs were signed between India and the UAE in the presence of the
two leaders, including one between the Gujarat government and DP World,
a multinational logistics company based in Dubai.
 Three of the agreements between India and the UAE were on investment
cooperation in renewable energy sector, innovative healthcare projects and
food park development.
 In February 2022, India and the UAE inked the Comprehensive Economic
Partnership Agreement (CEPA) to boost trade ties.
 The trade pact will help in taking the two-way trade to $100 billion in five
years.
 The Prime Minister, Shri Narendra Modi inaugurated the 10th edition of Vibrant Gujarat Global Summit 2024 at Mahatma Mandir,
Gandhinagar.
 The theme of this year’s summit is ‘Gateway to the Future’ and includes the participation of 34 partner countries and 16 partner
organizations.

Click Here For Ultimate Bundle PDF Course 2022| Click Here to Subscribe Our Yearly Mock Test Package Follow Us- Telegram. Facebook , Twitter ,Instagram
Weekly Optimum Current Q & A PDF – Based On Recent Exam Trends Second Week of January (9th January –16th January 2024)

 The Summit is also being used as a platform by the Ministry of Development of North-Eastern Region to showcase investment
opportunities in the North-Eastern regions.
 News Link - pib.gov.in/PressReleaseIframePage.aspx?PRID=1994793

33. In January as per report 'Yuva Nidhi' scheme has been launched by Siddaramaiah for unemployed youth in Karnataka's Shivamogga.
Under the scheme government has promised ___________per month to degree holders and Rs 1,500 per month to diploma holders who
were unable to get a job in the last six months and are not pursuing higher studies.
A. Rs 2,000 per month
B. Rs 2,500 per month
C. Rs 3,500 per month
D. Rs 3,000 per month
E. Rs 5,000 per month

 'Yuva Nidhi' scheme launched by Siddaramaiah for


unemployed youth in Karnataka's Shivamogga.
 On 12 January, the fifth and final poll guarantee of the
Congress 'Yuva Nidhi' scheme was launched by
Karnataka Chief Minister Siddaramaiah at Freedom
Park in Shivamogga.
 The scheme provides unemployment assistance to graduates and diploma holders who have obtained their degrees in the academic year
2022-23.
 The government has promised Rs 3,000 per month to degree holders and Rs 1,500 per month to diploma holders who were unable to get a
job in the last six months and are not pursuing higher studies.
 The State Government has earmarked ₹250 crore for the scheme this year. It expects the amount to go up to ₹1,200 crore in 2025, and
₹1,500 crore annually from 2026 onwards.
 The scheme applies only to graduates and diploma holders whose domicile is Karnataka.
 The other four of the five guarantees are:
 1. ‘Shakti’: To provide free rides to Karnataka women in non-luxury government buses
 2. ‘Anna Bhagya’: To offer 10 kg of rice to BPL families
 3. ‘Gruha Jyoti’: To give free electricity up to 200 units to household users
 4. ‘Gruha Lakshmi’: To promise Rs 2,000 a month to women heads of families having APL/BPL ration cards.
 News Link - Yuva Nidhi for unemployed youth launched by Congress govt in Karnataka - The Hindu

34. In January as per report Climate Summit 2024 was held in Mumbai on 12 January.
What was the theme of the Climate Summit 2024?
A. Assessing Impacts in Diverse Ecosystems
B. Human Impacts and Responsibility
C. Decoding the Green Transition for India
D. Climate action can't wait
E. One Earth, One Family, One Future

 Climate Summit 2024 was held in Mumbai on 12 January.


 Climate Summit 2024, themed "Decoding the Green Transition for India", was held in Mumbai on January 12.
 The summit was organized by the Ministry of Environment, Forest and Climate Change, the Government of India.
 The event focused on the important role of the private sector, climate tech startups, and institutional investors in mobilizing financial
resources and technical capabilities.

Click Here For Ultimate Bundle PDF Course 2022| Click Here to Subscribe Our Yearly Mock Test Package Follow Us- Telegram. Facebook , Twitter ,Instagram
Weekly Optimum Current Q & A PDF – Based On Recent Exam Trends Second Week of January (9th January –16th January 2024)

 The conference was organized under the Green Climate Fund Readiness Program in collaboration with Delivery Partner UNDP India and
knowledge partner Avaana Capital.
 The conference outlined India's roadmap to achieve net-zero by 2070.
 It focused on green transition investments to transform energy systems, reduce carbon dioxide emissions, conserve natural resources,
enhance biodiversity, and enhance climate resilience in a socially equitable and inclusive manner.
 News Link - pib.gov.in/PressReleaseIframePage.aspx?PRID=1995506#:~:text=India Stands Ready to Mobilise,Paves the Way%3A G20
Sherpa&text=The Climate Conference 2024%2C themed,2024%2C in Mumbai%2C Maharashtra.

35. In January as per report Enathu Gramam, My Village, a unique scheme for the development of villages was launched by which of the
following state government?
A. Odisha
B. Telangana
C. Andhra Pradesh
D. Tamil Nadu
E. Karnataka

 World Tamil Diaspora Day celebrated on 12 January


in Chennai.
 Awards were given to thirteen exemplary
personalities for their contribution to the
development of Tamils.
 On the occasion, Chief Minister M.K.Stalin said that a special department has been created for the welfare of Tamils living abroad and a
minister has been appointed.
 The Chief Minister distributed medals to 13 important Tamil achievers who have excelled in various fields, including education, art,
literature, and culture.
 In this event, Tamils living in fifty countries participated.
 Enathu Gramam, My Village, a unique scheme for the development of villages was launched.
 Under this scheme, any person can adopt his native village and ensure its overall development.
 News Link - World Tamil Diaspora Day celebrated in Chennai (newsonair.gov.in)

36. Recently as per report India is set to become the world’s largest producer of lentil (masoor) during the 2023-24 crop years on account of
higher acreage.
What is the current MSP of lentil?
A. ₹6,425 per quintal
B. ₹6,125 per quintal
C. ₹6,896 per quintal
D. ₹6,785 per quintal
E. ₹6,225 per quintal

 India is set to be the world’s largest producer of


lentil (masur) this year on the back of an
expected surge in production estimated at about
1.6 million tonnes (mt), Consumer Affairs
Secretary Rohit Kumar Singh said.
 Lentil production in Canada, the top player in the global market, is projected at around 1.5 mt and in Australia at 1.4 mt.
 Sources in the Agriculture Ministry said the lentil output in 2023-24 may rise to a record. The previous high was in 2017-18, when India had
produced 1.62 mt.

Click Here For Ultimate Bundle PDF Course 2022| Click Here to Subscribe Our Yearly Mock Test Package Follow Us- Telegram. Facebook , Twitter ,Instagram
Weekly Optimum Current Q & A PDF – Based On Recent Exam Trends Second Week of January (9th January –16th January 2024)

 This year Pulses24 Convention, to be hosted by India.


 The convention will be jointly organised by Global Pulse Confederation (GPC) and Nafed in Delhi from February 14 to 17.
 There has been a sharp rise in the Minimum Support Price (MSP) of masur from ₹4,250 per quintal in 2017-18 (crop year) to ₹6,425 per
quintal in 2023-24, a rise of over 52 per cent.
 As India started looking for imports after production dropped in 2018-19 to 1.23 mt (24 per cent) from the previous year, the Government
kept increasing MSP every year substantially to motivate farmers towards lentil.
 News Link - India may emerge top producer of lentil globally - The Hindu BusinessLine India masur production: Govt says masur production
may touch record 1.6 mn tons this rabi season - The Economic Times (indiatimes.com)

37. Recently Gujarat has set a target of producing _________ million metric tonne per annum (MMPTA) of green hydrogen by 2030 and has
allocated two lakh hectares of land for this purpose.
A. 4 MMPTA
B. 2 MMPTA
C. 3 MMPTA
D. 5 MMPTA
E. 1 MMPTA

 Launching Gujarat’s mission to produce Green Hydrogen, Gujarat Chief Minister Bhupendra Patel while addressing the 10th edition of the
Vibrant Gujarat summit said the state has set a target of producing 3 million metric tonne per annum (MMPTA) of green hydrogen by 2030
and has allocated two lakh hectares of land for this purpose.
 For green growth in the state, Gujarat has allocated ₹2 lakh crore budget for five years.
 A special land policy has been made for the production of green hydrogen and has allocated 2
lakh hectares of land in Kutch and Banaskantha for this purpose.
 The Prime Minister has set a target of 5 MMPTA of hydrogen production by 2030.
 To meet this target, Gujarat has set itself an ambitious target of producing 3 MMTPA of
hydrogen.
 Gujarat has floated a pilot project for producing 7.58 tonnes of green hydrogen per day.
 Gujarat is also analysing the feasibility of hydrogen blending with Piped Natural Gas (PNG) for
its efficient utilisation.
 According to the Gujarat government, the state will require $20-25 billion of investments by
2030 for setting up just 1 MMPTA of green hydrogen capacity.
 News Link - Gujarat set target to produce 3 MMPTA of green hydrogen by 2030 - The Hindu
BusinessLine

38. In January as per report _______________ has launched a model for “regenerative tourism,”
and become the first state in the country to do so.
A. Goa government
B. Odisha government
C. Tamil Nadu government
D. Uttar Pradesh government
E. West Bengal government

 The Goa government has launched a model for “regenerative tourism,” to become the first
state in the country to do so.
 The approach aims to redefine tourism by promoting environmental restoration, cultural
preservation, and community empowerment.
 The state government is promoting 11 spiritual sites as part of the regenerative tourism model.

Click Here For Ultimate Bundle PDF Course 2022| Click Here to Subscribe Our Yearly Mock Test Package Follow Us- Telegram. Facebook , Twitter ,Instagram
Weekly Optimum Current Q & A PDF – Based On Recent Exam Trends Second Week of January (9th January –16th January 2024)

 The launch of the model follows the G20 tourism ministers working group meet in June 2023, that came up with the Goa road map for
tourism as a method for achieving the sustainable development goals.
 The launch of the model is crucial as the Goa government wishes to enhance foreign and domestic tourist footfalls in the state.
 The Goa government has unfolded the “Ekadasha Teertha” campaign in the regions of eleven places of worship in Goa the state.
 News Link - Goa turns focus to regenerative tourism; 11 sites identified for development, ET TravelWorld (indiatimes.com) Goa launches new
model of regenerative tourism - Economy News | The Financial Express

39. Recently as per report ____________ has recently built India’s First Cow Sanctuary, a centrally funded under the Rashtriya Gokul Mission
(RGM).
A. Tamil Nadu
B. Karnataka
C. Odisha
D. Madhya Pradesh
E. Uttar Pradesh

 The Uttar Pradesh Government has


recently built India’s First Cow
Sanctuary, a Centrally funded at
Muzaffarnagar district under the
Rashtriya Gokul Mission (RGM).
 The sanctuary is situated at the
Tuglakpur-Kamhera village under
the Purkaji block of the
Muzaffarnagar district has been
built with funding of Rs 64 crore
from Rashtriya Gokul Mission
(RGM).
 The sanctuary can shelter up to 5,000 stray cattle.
 Rashtriya Gokul Mission is a centrally-sponsored scheme, launched in 2014.
 Ministry: Ministry of Fisheries, Animal Husbandry & Dairying for the development
 As per the 20th Livestock Census of 2019, the state has 11.84 lakh stray cattle. In the Muzaffarnagar district, the figure stood at 3,207.
 News Link - First centrally funded cow sanctuary in his constituency, Union minister busy seeking fodder donation | India News - The Indian
Express

40. In January as per report Hardeep S. Puri launched ___________ portals that provide real-time progress updates on the mission and
scheme, fostering transparency and accountability for street vendors.
A. e-Sakshi portal
B. PAiSA portal
C. e-Mitra portal
D. Samriddhi portal
E. 5Paisa portal

 The Union Minister for Housing and Urban Affairs, and Petroleum and Natural Gas, Hardeep S. Puri stressed the significance of
establishing and maintaining robust grievance redressal committees under the Street Vendors Act 2014, to resolve disputes and conflicts
among street vendors.

Click Here For Ultimate Bundle PDF Course 2022| Click Here to Subscribe Our Yearly Mock Test Package Follow Us- Telegram. Facebook , Twitter ,Instagram
Weekly Optimum Current Q & A PDF – Based On Recent Exam Trends Second Week of January (9th January –16th January 2024)

 Two websites -- the refreshed PAiSA portal dashboard and the PM SVANidhi Mission Monitoring Portal -- were launched by the minister.
These portals will provide real-time progress updates on the mission and scheme, fostering transparency and accountability, the ministry
said in a statement.
 MoHUA launched PM SVANidhi scheme on June 01, 2020, to address the challenges faced by street vendors and help them in resuming the
businesses adversely impacted by the Covid-19 pandemic.
 “PAiSA,” which is an acronym for Portal for Affordable Credit and Interest Subvention Access.
 News Link - hardeepsinghpuri.com/single-media/mohua-is-committed-to-create-supportive-and-empowering-environment-for-street-
vendors-across-nation-minister-hardeep-s-puri

41. In January as per report Department of Telecommunications (DoT), Ministry of Communications (MoC), has extended machine-to-machine
(M2M) and Wireless Personal Area Network/Wireless Local Area Network (WPAN/WLAN) registration to all entities engaged in the
businesses.
All business entities are advised to register with DoT through a simple and transparent online process via __________.
A. SaralSathi portal
B. SaralSampark portal
C. SaralSanchar portal
D. SaralSampurn portal
E. SaralSakshi portal

 Department of Telecommunications (DoT), Ministry of Communications (MoC), has extended machine-to-machine (M2M) and Wireless
Personal Area Network/Wireless Local Area Network (WPAN/WLAN) registration to all entities engaged in the businesses.
 All business entities including companies, government departments, organizations, partnership firms, LLPs, institutions, undertakings,
proprietorship firms, societies and trusts engaged in M2M service provisioning and WPAN/WLAN connectivity provisioning are advised to
register with DoT through a simple and transparent online process via SaralSanchar portal.
 Non-compliance may lead to withdrawal or disconnection of telecom resources obtained from the authorized telecom licensees.
 The decision has been taken to extend the scope of the registration in order to proliferate the standard-based and secure M2M/IoT
ecosystem.
 News Link - Press Information Bureau (pib.gov.in)

42. Recently CD 11 Module 2 (Morbidity Codes) has been launched by WHO in New Delhi on 10th January.
Data and terminology related to diseases based on _________systems have now been included in WHO's International Classification of
Diseases (ICD) 11 classification.
A. Ayurveda
B. Siddha
C. Unani
D. Only B and C
E. All of the above

 CD 11 Module 2 (Morbidity Codes) has been launched by WHO in New Delhi on 10th January. Data and terminology related to diseases
based on Ayurveda, Siddha, and Unani systems have now been included in WHO's International Classification of Diseases (ICD) 11
classification.
 This effort brought global uniformity in ASU (Ayurveda, Unani, and Siddha) medicine in the form of a code of terminology defining
diseases.
 WHO has developed a classification series called the International Classification of Diseases (ICD) to classify diseases at the international
level.
 The Central Bureau of Health Intelligence (CBHI) is an agency under the Ministry of Health and Family Welfare that serves as the WHO
Collaborating Center for ICD-related activities.

Click Here For Ultimate Bundle PDF Course 2022| Click Here to Subscribe Our Yearly Mock Test Package Follow Us- Telegram. Facebook , Twitter ,Instagram
Weekly Optimum Current Q & A PDF – Based On Recent Exam Trends Second Week of January (9th January –16th January 2024)

 It facilitates the collection and dissemination of data on various diseases and mortality.
 The Ministry of AYUSH has already developed codes for Ayurveda, Siddha, and Unani medicine through the National AYUSH Morbidity and
Standardized Electronic Portal (NAMAST).
 The Ministry of AYUSH in collaboration with WHO has prepared a classification of data and terminology related to diseases based on
AYUSH–Ayurveda, Siddha, and Unani systems under the TM2 module of the ICD11 series.
 The Ministry of AYUSH has also signed a donor agreement with the World Health Organization for this classification.
 This classification includes infectious diseases like malaria and lifestyle diseases like chronic insomnia.
 This effort will further strengthen and expand India's public health service delivery system, research, AYUSH insurance coverage, research
and development, and policy-making systems.
 Moreover, these codes will also be used in formulating future strategies to control various diseases in society.
 The strategy for this was prepared from 2014 to 2023 and the first draft of the WHO strategy for Traditional Medicine for 2025 to 2034 has
been prepared.
 News Link - Press Information Bureau (pib.gov.in)

43. Recently seen in news Jagannath Temple is located in which of the following state?
A. Kerala
B. Odisha
C. Tamil Nadu
D. Karnataka
E. Andhra Pradesh

 Recently, the Chief Minister of Odisha unveiled a Rs 800-crore sprawling heritage corridor around
the Jagannath Temple in Puri.
 'Srimandir Parikrama Prakalpa' or Jagannath Temple Heritage Corridor project opened to the
public on 17 Jan 2024 at Puri in Odisha.
 The project is part of an initiative of the state government to make the pilgrim town a world
heritage city.
 The Temple heritage corridor has been constructed to improve the visual connection between
devotees and the shrine.
 The 75-meter corridor will provide unobstructed pathways around the Jagannath temple’s
Meghanada Pacheri (outer wall).
 Jagannath Temple, Puri is dedicated to Jagannath, a form of Vishnu. It is a Hindu temple. It is
famous for its annual Ratha Yatra.
 News Link - Patnaik inaugurates Rs 800-cr heritage corridor project around Puri's Jagannath temple
– ThePrint – PTIFeed

44. In January as per report ____________ was presented with “Indian of the Year Award" for the year 2023 in the category ‘Outstanding
Achievement’.
A. Indian Army
B. DRDO
C. ISRO
D. State Bank Of India
E. Bank Of Baroda

 Union Minister of State (Independent Charge) Science & Technology; MoS PMO, Personnel, Public Grievances, Pensions, Atomic Energy and
Space, Dr Jitendra Singh presented “Indian of the Year Award" for the year 2023 in the category ‘Outstanding Achievement’ to Team ISRO
(Indian Space Research Organisation).

Click Here For Ultimate Bundle PDF Course 2022| Click Here to Subscribe Our Yearly Mock Test Package Follow Us- Telegram. Facebook , Twitter ,Instagram
Weekly Optimum Current Q & A PDF – Based On Recent Exam Trends Second Week of January (9th January –16th January 2024)

 The award, instituted by a national TV channel, was received by ISRO Chairman S. Somnath and Project Director of Chandrayaan 3, Dr
P.Veeramuthuvel, at a glittering ceremony in New Delhi.
 The pinnacle of ISRO's achievements in 2023 was the successful first-ever soft landing of Chandrayaan-3 on the Moon's uncharted South
Polar region.
 The remarkable growth of the private space sector was also highlighted in the event, with 190 startups and over Rs. 1,000 crore invested in
the current financial year 2023-24.
 News Link - Press Information Bureau (pib.gov.in)

45. In January as per report ) ___________has entered into MoUs with Gujarat State Petroleum Corporation Ltd. (GSPC) and Gujarat Pipavav
Port Ltd. (GPPL) for the blending of Green Hydrogen in the gas network.
A. Oil India
B. ONGC
C. NTPC
D. BPCL
E. IOCL

 NTPC Green Energy Ltd (NGEL) has entered into MoUs with Gujarat State Petroleum Corporation Ltd. (GSPC) and Gujarat Pipavav Port Ltd.
(GPPL) during the three-day Vibrant Gujarat Global Summit 2024, being held in Gandhinagar during January 10 – 12, 2024.
 The MoU with Gujarat State Petroleum Corporation Ltd. aims for the blending of Green Hydrogen in the gas networks of GSPC and
promotion of Green Hydrogen mobility by setting up Green Hydrogen fuelling stations in Gujarat.
 The MoU with Gujarat Pipavav Port Ltd. aims to develop a green hydrogen ecosystem including production of green ammonia for export
and the domestic market, at the land provided by GPPL.
 It also aims to explore the development of Pipavav Port as an anchor port by NGEL for exploration, development and operations of
offshore wind farms in Gujarat.
 NGEL is a wholly-owned subsidiary of NTPC.
 News Link - Press Information Bureau (pib.gov.in)

46. Recently as per report India is set to chair and host _____________from July 21 to 31 this year in New Delhi for the very first time.
A. UNESCO World Heritage Committee
B. UNICEF Economic and Financial Committee
C. UN Disarmament and International Security Committee
D. WHO Special Political and Decolonization Committee
E. UNCTAD Administrative and Budgetary Committee

 In a historic development, India is set to chair and host United Nations Educational, Scientific and Cultural Organization (Unesco)'s World
Heritage Committee from July 21 to 31 this year in New Delhi.
 At the 19th extraordinary session [Unesco, 2023], the World Heritage Committee decided that its 46th session will take place in India.
 It is a committee of the United Nations Educational, Scientific, and Cultural Organization (UNESCO).
 The Committee is responsible for the implementation of the World Heritage Convention, defines the use of the World Heritage Fund, and
allocates financial assistance upon requests from States Parties.
 It decides on the inscription or deletion of properties on the List of World Heritage in Danger.
 It consists of representatives from 21 of the States Parties to the Convention elected by their General Assembly.
 A Committee member's term of office is six years, but most state parties choose voluntarily to be members of the committee for only four
years in order to give other states parties an opportunity to be on the committee.
 News Link - In a first, India set to chair, host Unesco's World Heritage Committee (business-standard.com)

Click Here For Ultimate Bundle PDF Course 2022| Click Here to Subscribe Our Yearly Mock Test Package Follow Us- Telegram. Facebook , Twitter ,Instagram
Weekly Optimum Current Q & A PDF – Based On Recent Exam Trends Second Week of January (9th January –16th January 2024)

47. In January as per report ____________ government to launch Ramlala Darshan Scheme for Ayodhya Pilgrimage.
A. Odisha
B. Chhattisgarh
C. Uttar Pradesh
D. Madhya Pradesh
E. Gujarat

 The Chhattisgarh government has decided to launch the 'Shri


Ramlala Darshan scheme’.
 This scheme will be implemented by the Chhattisgarh Tourism Board
and the Budget Tourism Department.
 Annually, around 20,000 pilgrims will be taken for the Shri Ramlala
Darshan.
 People aged between 18 to 75 will be eligible for availing benefits of
this scheme. The health examination will be conducted by the district
medical board.
 Under this scheme, transportation will be facilitated in collaboration
with the Indian Railways Catering and Tourism Corporation (IRCTC).
 News Link - Chhattisgarh to Introduce Ramlala Darshan Scheme for
Ayodhya Pilgrimage (newsonair.gov.in)

48. In January as per report ___________has introduced a mobile app ‘Report Fish Disease’ to track and monitor fish diseases across the
country.
A. National Surveillance Programme for Aquatic Animal Diseases
B. National Fisheries Development Board
C. National Institute of Fisheries Post Harvest Technology
D. Coastal Aquaculture Authority
E. Central Institute of Fisheries Nautical and Engineering

 The National Surveillance Programme for Aquatic Animal Diseases


(NSPAAD) project has introduced a mobile app ‘Report Fish
Disease’ to track and monitor fish diseases across the country.
 The mobile application intends to empower fish farmers with a
convenient and efficient platform to report diseases on their farms.
 Central Institute of Fisheries Technology (CIFT) is one of the
collaborative partners of the NSPAAD project of which ICAR-NBFGR
is the lead institute, under which this app is developed.
 Its intuitive and user-friendly interface enables easy accessibility to
fish farmers.
 The app offers an easy disease reporting format, where farmers
can easily report disease outbreaks by providing essential
information such as location, species affected, symptoms observed,
and images.
 News Link - ‘Report Fish Disease’, mobile app to monitor fish diseases launched - The Hindu BusinessLine

49. In January as per report Union Health Ministry announced the implementation of the revised Schedule M as part of the Drugs and Cosmetics
Rules, 1945.

Click Here For Ultimate Bundle PDF Course 2022| Click Here to Subscribe Our Yearly Mock Test Package Follow Us- Telegram. Facebook , Twitter ,Instagram
Weekly Optimum Current Q & A PDF – Based On Recent Exam Trends Second Week of January (9th January –16th January 2024)

As per new rules Small and medium manufacturers with an annual


turnover of less than __________crore have 12 months to comply.
A. ₹200 crore
B. ₹150 crore
C. ₹300 crore
D. ₹250 crore
E. ₹100 crore

 The Union Health Ministry announced the implementation of the revised


Schedule M as part of the Drugs and Cosmetics Rules, 1945.
 This amendment introduces stringent Good Manufacturing Practices
(GMP) and mandates the requirements of premises, plant, and
equipment for pharmaceutical products.
 The revision aims to bring Indian GMP recommendations in line with
international standards, particularly those of the World Health
Organization (WHO).
 The revised rules are to be implemented on the basis of company
turnovers where -
 The medium and small manufacturers (with an annual turnover of less
than ₹250 crore) will have to implement the revised rules within 12
months from its date of publication.
 Whereas large manufacturers with an annual turnover of over ₹250 crore
will be given six months to do so.
 The revision mandates manufacturers to assume responsibility for the
quality of pharmaceutical products, ensuring they meet the licence
requirements and are safe for use.
 GMP was first incorporated in Schedule M of the Drugs and Cosmetics
Rules 1945 in the year 1988 and the last amendment was done in 2005.
 News Link - Pharmaceutical Quality System: Govt notifies revised schedule M guidelines for pharma manufacturing units, ET HealthWorld
(indiatimes.com)

50. In January as per report __________Scheme won Best Innovation Award for Innovation Sandbox presentation during Public Policy
Dialogues–2024 at Indian School of Business, Hyderabad.
A. SAMRIDDHI Scheme
B. SVAMITVA Scheme
C. SAFAL Scheme
D. SAMADHAN Scheme
E. SARTAHQ Scheme

 SVAMITVA Scheme wins Best Innovation Award for Innovation Sandbox presentation during Public Policy Dialogues–2024 at Indian School
of Business, Hyderabad.
 SVAMITVA is a Central Sector Scheme under the Ministry of Panchayati Raj, launched by the Prime Minister in April 2020.
 It aims empowering rural communities through accurate land demarcation using drone surveys and GIS mapping.
 SVAMITVA Scheme of the Ministry of Panchayati Raj is a Central Sector Scheme that has significantly contributed to the empowerment of
rural communities in India.
 The scheme with target year of completion of its implementation 2024–25, has achieved many milestones. The drone survey has been
completed in 2.90 lakh villages and 1.66 crore property cards have been prepared for 1.06 lakh villages so far.
 News Link - https://pib.gov.in/PressReleaseIframePage.aspx?PRID=1993736

Click Here For Ultimate Bundle PDF Course 2022| Click Here to Subscribe Our Yearly Mock Test Package Follow Us- Telegram. Facebook , Twitter ,Instagram
Weekly Optimum Current Q & A PDF – Based On Recent Exam Trends Second Week of January (9th January –16th January 2024)

51. Recently Cabinet Committee on Economic Affairs (CCEA) fixed the maximum retail price (MRP) of Urea Gold at __________ per 40 kg bag,
aligning it with 45 kg bags of Neem Coated Urea.
A. ₹266.50 per 40 kg bag
B. ₹254.50 per 40 kg bag
C. ₹345.50 per 40 kg bag
D. ₹271.50 per 40 kg bag
E. ₹289.50 per 40 kg bag

 The Ministry of Chemicals and Fertilizers recently conveyed


the government's decision on the Price of Urea Gold to all
fertilizer manufacturing companies.
 The government introduced "Urea Gold," a sulphur-coated
urea, to combat soil deficiencies.
 The Cabinet Committee on Economic Affairs (CCEA) fixed the
maximum retail price (MRP) of Urea Gold at ₹266.50 per 40
kg bag, aligning it with 45 kg bags of Neem Coated Urea.
 It was made ₹30 costlier due to reduced weight, but may
impact overall urea sales.
 The government aims to encourage environmentally friendly
fertilizers without burdening farmers.
 Sulphur-coated urea (SCU) is called Urea gold because of its yellowish-goldish colour.
 Urea Gold contains 37% Nitrogen (N) and 17% Sulphur (S), to address India's soil deficiencies, particularly in S, crucial for oilseeds and
pulses.
 News Link - Urea Gold to be sold in 40 kg bag, price to remain same at ₹266.50 per bag - The Hindu BusinessLine

52. In January as per report __________in India recently, established a ‘Bio-Imaging Bank’ for utilization of Artificial Intelligence (AI).
A. Indraprastha Apollo Hospital
B. Lilavati Hospital
C. Manipal Hospital
D. Tata Memorial Hospital
E. Christian Medical College

 Mumbai’s Tata Memorial Hospital (TMH), the biggest cancer hospital in India recently, established a ‘Bio-Imaging Bank’ for utilization of
Artificial Intelligence (AI).
 Bio-Imaging Bank project aims to establish a robust repository interweaving radiology and pathology images with clinical data, treatment
specifics, and metadata.
 It is funded by the Department of Biotechnology and collaborates with top institutions like IIT-Bombay, RGCIRC-New Delhi, AIIMS, and
PGIMER-Chandigarh.
 The project focuses initially on head neck and lung cancers.
 The hospital has achieved a 40% reduction in radiation through innovative AI algorithms, ensuring diagnostic quality without
compromising patient safety.
 News Link - How AI can help detect cancer and why India’s biggest cancer treatment hospital is utilising it | Explained News -
The Indian Express

53. In January as per report India and __________signed a bilateral agreement under which India has been allocated a quota of 1,75,025
pilgrims for the annual Haj pilgrimage in 2024.
A. UAE

Click Here For Ultimate Bundle PDF Course 2022| Click Here to Subscribe Our Yearly Mock Test Package Follow Us- Telegram. Facebook , Twitter ,Instagram
Weekly Optimum Current Q & A PDF – Based On Recent Exam Trends Second Week of January (9th January –16th January 2024)

B. Saudi Arabia
C. Oman
D. Jordan
E. Bangladesh

 A Bilateral Haj Agreement 2024 was signed between India and the Kingdom of Saudi Arabia (KSA).
 This agreement has been signed by both countries to further strengthen their relations and give priority to
the well-being of the pilgrims.
 The agreement was between Union Minister Smriti Zubin Irani and Saudi Minister of Haj and Umrah
Affairs Dr. Tawfiq bin Fawzan Al Rabiah in Jeddah.
 On January 7 and 8, Smriti Irani was in Saudi Arabia on a two-day visit.
 A total quota of 1,75,025 Haj pilgrims has been finalized for Haj 2024 from India.
 The agreement highlights India's commitment towards inclusivity, discussing ways to encourage the
participation of women in Hajj without a male guardian (Mehram).
 News Link - pib.gov.in/PressReleaseIframePage.aspx?PRID=1994012

54. As per the new agreement, every state and UT will get an opportunity to showcase its tableau within __________ years at Republic Day
parade.
A. 5years
B. 3 years
C. 6 years
D. 2 years
E. 1 years

 As per the new


agreement, every
state and UT will get
an opportunity to
showcase its tableau
within three years at
Republic Day parade.
 Around 15 tableaux
from the States and
Union Territories are
selected every year for
the Republic Day
parade.
 Now, every State and
UT will get an
opportunity to
showcase its tableau
within three years.

 A three-year memorandum of understanding (MoU) has been proposed. So far, 28 states have signed this MoU.
 The Ministry of Culture has empanelled 30 agencies for the design and fabrication of tableaux through an open selection process.
 In the 2024 Republic Day parade, 16 states and UTs will get the chance to show their tableau.
 The States that could not get an opportunity at this year’s Republic Day Parade have been invited to showcase their tableaux at Bharat
Parv from January 23 to 31, 2024, at Red Fort.

Click Here For Ultimate Bundle PDF Course 2022| Click Here to Subscribe Our Yearly Mock Test Package Follow Us- Telegram. Facebook , Twitter ,Instagram
Weekly Optimum Current Q & A PDF – Based On Recent Exam Trends Second Week of January (9th January –16th January 2024)

 An expert committee comprising artists and Padma awardees will finalize the theme and aesthetics.
 Every year on January 26th, the parade takes place on the Kartavya Path (formally Rajpath) in New Delhi. The first parade took place in
1950 and it has taken place every year since.
 Each year, the MoD comes up with an overarching theme, under which participants can showcase elements relevant to their respective
state/UT/department in their tableaux.
 This year’s theme is “Viksit Bharat” (Developed India) and “Bharat: Loktantra ki Matrika” (India: the Mother of Democracy).
 News Link - All states, UTs to get equal chance to display tableaux during Republic Day parade: Govt | India News - The Indian Express

International / Report News

55. In January as per report ___________has decided to open a new campus at Kandy in Sri Lanka.
A. IIT Madras
B. IIT Ropar
C. IIT Delhi
D. IIT Kanpur
E. IIT Bombay

 IIT Madras has decided to open a new campus at


Kandy in Sri Lanka.
 Sri Lanka’s Education minister Dr. Susil
Premajayantha made this announcement.
 Before the new campus establishment, a team of academics would be sent by the Sri Lankan government to IIT Madras.
 This is part of the Budget 2024 presented by Sri Lanka President Ranil Wickremasinghe.
 Earlier, the Indian Institute of Technology Madras formally launched its campus in Zanzibar, Tanzania.
 Currently, around 50 students have been admitted to this institute.
 News Link - IIT Madras to open new campus at Kandy in Sri Lanka (newsonair.gov.in)

56. In January as per report World Health Organization (WHO) has certified Cabo Verde (also known as Cape Verde) as a malaria-free country.
What is the capital of Cabo Verde?
A. Yamoussoukro
B. Nairobi
C. Port Louis
D. Praia
E. Algiers

 Recently, the World Health Organization (WHO) has certified Cabo Verde (also known as Cape Verde) as a malaria-free country.
 Cabo Verde has become the third country to acquire the status in the global health organisation’s African region. The country has now
joined Mauritius and Algeria, who were certified in 1973 and 2019, respectively.
 Worldwide some 43 countries have been certified as malaria-free by the WHO, which requires showing that the domestic chain of
transmission has been broken for at least three consecutive years.
 The WHO estimates that malaria killed 608,000 people worldwide and infected 250 million in 2022.
 It is also known as Cape Verde which comprises a group of islands that lie off the west coast of Africa.
 Capital: Praia
 News Link - WHO certifies Cabo Verde as malaria-free, marking a historic milestone in the fight against malaria Cape Verde becomes third
African country to eliminate malaria, ET HealthWorld (indiatimes.com)

Click Here For Ultimate Bundle PDF Course 2022| Click Here to Subscribe Our Yearly Mock Test Package Follow Us- Telegram. Facebook , Twitter ,Instagram
Weekly Optimum Current Q & A PDF – Based On Recent Exam Trends Second Week of January (9th January –16th January 2024)

57. In January as per report _____________announced the Green Fuels Alliance India (GFAI) at the Global Investors Meet 2024 to boost
collaborative efforts between the two countries in the sustainable energy solutions sector and advance their joint global goal towards
carbon neutrality.
A. Japan
B. Denmark
C. Netherland
D. USA
E. Germany

 Denmark has announced its Green Fuels Alliance India (GFAI) initiative to boost collaborative efforts between the two countries in the
sustainable energy solutions sector and advance their joint global goal towards carbon neutrality.
 GFAI aims to boost collaboration between India and Denmark in the sustainable energy solutions sector and work towards their joint
global goal of carbon neutrality.
 Led by the Danish Embassy and Consulate-General of Denmark in India, the GFAI aims to play a pivotal role in advancing the green fuels
sector, particularly green hydrogen, through fostering innovation, collaboration, and partnerships between Danish and Indian industries..
 Nine Danish organizations, including Maersk, Topsoe, Umwelt Energy, Mash Makes, European Sustainable Solutions, Novozymes, Danfoss,
Brdr. Christensen, and Hydrogen Denmark, have pledged their commitment as founding members of the GFAI initiative.
 GFAI’s advisory board members include India Hydrogen Alliance, Energy Consortium at the Indian Institute of Technology-Madras, the
Danish Energy Agency and State of Green.
 GFAI activities fall under the Green Strategic Partnership (GSP) signed between India and Denmark in 2020, reflecting a shared
commitment to environmental sustainability and clean energy.
 The Green Strategic Partnership was established in 2020 as a bilateral initiative between India and Denmark.
 It builds on and consolidates the existing agreement establishing a Joint Commission for Cooperation between India and Denmark.
 Meanwhile, Denmark has topped the global Climate Performance Ranking 2024 and is also on the path to achieving carbon neutrality by
2050.
 News Link - Denmark announces alliance on green fuels in India - The Economic Times (indiatimes.com)

58. Recently seen in news Sky Dew has been deployed by which of the following country to monitor Hezbollah activities?
A. UAE
B. Iran
C. Saudi Arabia
D. Russia
E. Israel

 Israel recently deployed Sky Dew along the Lebanese border as tensions with Hezbollah escalated.
 Sky Dew is a massive balloon-like structure in the shape of a puffed aircraft.
 The high-altitude observation balloon acts as a detection radar blip for small unmanned aircraft and cruise missiles.
 It is one of the largest aerial threat warning systems in the world.
 The technology has been jointly developed by Israel Defence and US company. The project was entirely funded by the U.S. Department of
Defence.
 The Israel Air Force was handed over the Sky Dew in 2022.
 It can carry 7,000 pounds and has an operational altitude of 10,000 feet.
 It offers a significant advantage in its ability to remain in surveillance for extended periods without needing fuel or crew replacement.
 News lInk - Sky Dew: Massive radar blimp launched by Israel to monitor Hezbollah activities - Times of India (indiatimes.com)

59. In January as per report International Atomic Energy Agency (IAEA), in cooperation with ____________, launched its first scientific
research expedition to investigate the presence of microplastics in Antarctica.

Click Here For Ultimate Bundle PDF Course 2022| Click Here to Subscribe Our Yearly Mock Test Package Follow Us- Telegram. Facebook , Twitter ,Instagram
Weekly Optimum Current Q & A PDF – Based On Recent Exam Trends Second Week of January (9th January –16th January 2024)

A. Brazil
B. Argentina
C. France
D. Germany
E. India

 The International Atomic Energy Agency (IAEA), in cooperation with Argentina, launched its first scientific research expedition to
investigate the presence of microplastics in Antarctica as part of efforts to combat this growing environmental problem, even in the
planet’s most remote areas.
 Microplastics are tiny plastic particles up to 5mm in diameter.
 The President of Argentina, Javier Milei, and IAEA Director General Rafael Mariano Grossi joined the IAEA scientific team at the Marambio
and Esperanza Argentine Antarctic Bases to mark the start of their mission.
 The IAEA mission to Antarctica, the world’s southernmost continent, is being carried out through the IAEA’s NUTEC plastics initiative.
 Established in 2020, NUTEC is an IAEA flagship initiative to fight plastic pollution with nuclear technologies.
 In a resolution from March 2022, Member States of the United Nations pledged to initiate negotiations for a new global treaty on banning
plastic pollution including in the marine environment, with the objective of formal adoption by 2025.
 The International Atomic Energy Agency is an intergovernmental organization that seeks to promote the peaceful use of nuclear energy.
Headquarters: Vienna, Austria
 News Link - IAEA Scientists Embark on Antarctic Mission to Research Microplastic Impact | IAEA

60. As per the forecast of ILO's 2024 'World Employment and Social Outlook' report, the number of unemployed people will increase by
________ million.
A. 1 million
B. 3 million
C. 2 million
D. 4 million
E. 5 million

 According to the ILO report, the global unemployment rate will increase to 5.2% in
2024.
 The International Labour Organization (ILO) said that this increase is mainly because
of growth in joblessness in advanced economies.
 As per the forecast of ILO's 2024 'World Employment and Social Outlook' report, the
number of unemployed people will increase by 2 million.
 As a result, the global unemployment rate will increase to 5.2% from 5.1% in 2023.
 According to the ILO, aggregate labour productivity growth has swiftly returned to
the low pace observed over the preceding ten years, following a brief growth burst as
countries recovered from the epidemic.
 The scenario is more worrying in high-income nations, where employment growth is
predicted to turn negative in 2024 and show only slight increases in 2025.
 Moreover, working poverty is likely to persist. Despite quickly declining after 2020,
the number of workers living in extreme poverty (earning less than US$2.15 per
person per day in purchasing power parity terms) grew by about 1 million in 2023.
 The number of workers living in moderate poverty (earning less than US$3.65 per day
per person in PPP terms) increased by 8.4 million in 2023.
 International Labour Organization founded in October 1919 under the League of
Nations, it is one of the first and oldest specialised agencies of the UN. Headquarters:
Geneva, Switzerland

Click Here For Ultimate Bundle PDF Course 2022| Click Here to Subscribe Our Yearly Mock Test Package Follow Us- Telegram. Facebook , Twitter ,Instagram
Weekly Optimum Current Q & A PDF – Based On Recent Exam Trends Second Week of January (9th January –16th January 2024)

 News Link - Global unemployment to increase in 2024, warns ILO report | UN News Global Unemployment Set to Reach 5.2% in 2024, ILO
Reports (newsonair.gov.in) WESO Trends 2024: Global unemployment rate set to increase in 2024 while growing social inequalities raise
concerns, says ILO report

61. Consider the following statement regarding Henley Passport Index 2024. Which of the following statement is incorrect?
1. The Indian passport is in the 80th position in the Henley Passport Index 2024.
2. It will allow Indian citizens to travel to 62 countries without a visa.
3. India shares its rank with Bangladesh.
4. Six countries hasve topped the list France, Germany, Italy, Japan, Singapore and Spain
5. The least powerful passport in the world is Afghanistan.

 The Henley Passport Index 2024 was released, highlighting the "power"
of passports of 199 countries.
 The index, which ranks countries based on the number of destinations
that allow visa-free entry to their passport, was topped by France,
Germany, Italy, Japan, Singapore and Spain.
 Passports of all these countries have visa-free access to 194
destinations, three more than last year.
 South Korea, Sweden and Finland rose one spot to take joint second
place, with access to 193 countries. Austria, Denmark, Ireland and the
Netherlands shared third place, allowing travel to 192 places.
 The Indian passport is in the 80th position in the Henley Passport Index
2024.
 It will allow Indian citizens to travel to 62 countries without a visa.
 India shares its rank with Uzbekistan. India’s neighboring country
Pakistan is in the 101st position.
 Other neighbours of India were ranked as follows: Maldives (#58),
China (#62), Bhutan (#87), Myanmar (#92), Sri Lanka (#96), Bangladesh
(#97), and Nepal (#98).
 The least powerful passport in the world is Afghanistan. It ranked at
104th spot with access to 28 countries. It is followed by Syria (103), Iraq
(102), Pakistan (101) and Yemen (100).
 The index is based on exclusive data from the International Air
Transport Association (IATA), the largest, most accurate travel
information database, and enhanced by Henley & Partners’ research
team. (Henley & Partners is a London-based advisory firm).
 News Link - The Official Passport Index Ranking | Henley & Partners (henleyglobal.com) Henley Passport Index 2024: What is India's rank
among other countries? (business-standard.com)

62. According to the think tank Centre for Research on Energy and Clean Air (CREA), which of the following is the top 3 most polluted cities in
India in 2023?
1. Byrnihat
2. Greater Noida
3. Delhi
4. Silchar
5. Begusarai

Select the correct source code:


Click Here For Ultimate Bundle PDF Course 2022| Click Here to Subscribe Our Yearly Mock Test Package Follow Us- Telegram. Facebook , Twitter ,Instagram
Weekly Optimum Current Q & A PDF – Based On Recent Exam Trends Second Week of January (9th January –16th January 2024)

A. Only 2,3 and 4


B. Only 1,2 and 5
C. Only 2,4 and 5
D. Only 1 and 5
E. All of the above

 According to the think tank Centre for Research on Energy and Clean Air
(CREA), Byrnihat in Meghalaya topped the list of the most polluted cities in
India in 2023.
 Byrnihat, recorded the highest annual average PM10 concentration at 301
micrograms per cubic meter.
 Begusarai in Bihar (average annual PM10 level 265 micrograms per cubic
metre) and Greater Noida in Uttar Pradesh (228 micrograms per cubic metre)
took the second and third positions among the most polluted cities.
 Delhi, which is known for persistently high air pollution levels during winter,
ranked as the eighth most polluted city.
 However, Silchar in Assam recorded the lowest PM10 level of 29 micrograms per cubic meter in 2023.
 Of the top 50 polluted cities, 18 are from Bihar, eight from Haryana, and eight from Rajasthan.
 227 cities have been studied with air quality data available for more than 75 percent of the days in 2023.
 Of these cities, 85 were included under the National Clean Air Program (NCAP).
 The data showed that PM10 levels exceeded the NAAQS (60 micrograms per cubic metre) in 78 of the 85 NCAP cities.
 Launched in 2019, NCAP aims to reduce PM2.5 and PM10 concentration by 20-30% by 2024 in 131 cities that did not meet air quality
standards set between 2011 to 2015.
 The government has now set a new target of achieving a 40 percent reduction in particulate matter concentration in these cities by 2026.
 The government has now set a new target of reducing particulate matter concentrations in these cities by 40% by 2026.
 News Link - Press Trust of India: Meghalaya's Byrnihat not Delhi India's most polluted city in 2023: Report (ptinews.com) Delhi is not India's
most polluted city in 2023: CREA - The Economic Times (indiatimes.com)

63. Which of the following organisations has released “Global Risks Report 2024” that has ranked AI-derived misinformation risk ahead of
climate change, war and economic weakness as the biggest risk for 2024?
A. World Bank
B. United Nations Conference on Trade and Development
C. World Economic Forum
D. United Nations International Children's Emergency Fund
E. United Nations Educational, Scientific and Cultural Organization

 On 10 January, the Global Risks Report 2024 was


released by the World Economic Forum.
 The impact of artificial intelligence in disrupting
election outcomes has emerged as the biggest
risk for 2024.
 Artificial Intelligence can build models for
influencing a large number of voters.
 The “Global Risks Report 2024” has ranked AI-
derived misinformation risk ahead of climate
change, war and economic weakness.
 Global Risks Report 2024 has been prepared in
collaboration with Zurich Insurance Group.

Click Here For Ultimate Bundle PDF Course 2022| Click Here to Subscribe Our Yearly Mock Test Package Follow Us- Telegram. Facebook , Twitter ,Instagram
Weekly Optimum Current Q & A PDF – Based On Recent Exam Trends Second Week of January (9th January –16th January 2024)

 It surveyed more than 1,400 global risk experts, policymakers and industry leaders and asked about their biggest global concerns.
 For the next two years, misinformation and disinformation, extreme weather events, cyber insecurity and interstate armed conflict will be
the major global risks.
 Economic opportunity, inflation, involuntary migration, economic downturn and pollution are among the top 10 risks for the world.
 Extreme weather events, critical changes to earth systems, biodiversity loss and ecosystem collapse, natural resource shortages and
misinformation will remain major risks for more than the next 10 years.
 World Economic Forum Annual Meeting 2024 convenes the world’s foremost leaders under the theme, Rebuilding Trust.
 News Link - Global Risks 2024: Disinformation Tops Global Risks 2024 as Environmental Threats Intensify > Press releases | World Economic
Forum (weforum.org) Cyber inequity, AI key risks in 2024, says World Economic Forum study (business-standard.com)

64. Recently World Economic Forum’s 54th annual meeting was started in Davos, Switzerland on 15th January 2024.
What is the theme of the WEF 54th annual meet?
A. Cooperation in a Fragmented World
B. Rebuilding Trust
C. The State of the World
D. The Great Reset
E. Creating a Shared Future in a Fractured World

 World Economic Forum’s 54th annual meeting started in Davos,


Switzerland on 15th January 2024.
 The meeting will take place till 19th January 2024. The theme of meeting is
Rebuilding Trust.
 The meeting has started amid growing concerns over global conflicts,
climate change, and deepfake.
 The meeting emphasizes the focus on exploring the opportunities enabled
by the new technologies and their implications on decision making and
global partnership.
 The opening concert was devoted to the Sahara desert and Amazon
rainforest.
 Annual Crystal Awards were given to three artists. The names of artists
are architect Diebedo Francis Kere, actor Michelle Yeoh, and guitarist Nile
Rodgers.
 The Indian delegation would include three Union ministers and three chief
ministers.
 The three Union ministers are Smriti Irani, Ashwini Vaishnaw and Hardeep
Singh Puri.
 The three chief ministers are Maharashtra's Eknath Shinde, Telangana's
Revanth Reddy and Karnataka's Siddaramaiah.
 RBI Governor Shaktikanta Das and Ministers from Uttar Pradesh, Tamil
Nadu and Telangana will also take part.
 In tandem with the India Lounges, the establishment of five State Lounges
underscores India's consolidated and prominent role at WEF, fostering
collaboration and discourse on a global scale. The 5 states include
Karnataka, Maharashtra, Tamil Nadu, Telangana and Uttar Pradesh.
 World Economic Forum is a non-governmental organization founded in
1971. Its headquarters is located in Cologny, Switzerland.
 News Link - https://pib.gov.in/PressReleaseIframePage.aspx?PRID=1996767

Click Here For Ultimate Bundle PDF Course 2022| Click Here to Subscribe Our Yearly Mock Test Package Follow Us- Telegram. Facebook , Twitter ,Instagram
Weekly Optimum Current Q & A PDF – Based On Recent Exam Trends Second Week of January (9th January –16th January 2024)

65. Recently as per report India has been ranked ____________ on a global index of countries best placed for leveraging future possibilities.
A. 42nd
B. 35th
C. 38th
D. 31st
E. 51st

 India has been ranked 35th on a global index of countries best placed for leveraging future possibilities while the UK topped the list.
 The Future Possibilities Index (FPI), a flagship global future trends study released by Newsweek Vantage and Horizon Group on sidelines of
the World Economic Forum Annual Meeting, has Denmark, the US, the Netherlands and Germany in the top five after the UK.
 Among large emerging markets, China is placed highest at 19th this year, followed by Brazil at 30th, India (35th) and South Africa (50th).
 The study compared the factors that will help governments, investors and other private sector stakeholders leverage six global,
transformational trends for growth and wellbeing across 70 countries.
 These six global transformational trends are Exabyte Economy (advanced digital technologies), Wellbeing Economy (health prevention and
wellness), Net Zero Economy (reduction of carbon emissions), Circular Economy (recycling and reuse), BioGrowth Economy (food and
agriculture innovations), and Experience Economy (consumption of experiences instead of physical goods).
 The combined business opportunity of all six trends is estimated at more than USD 44 trillion by 2030, more than 40 per cent of global GDP
in 2023.
 News Link - India ranked 35th for leveraging future possibilities; UK on top - The Economic Times (indiatimes.com)

Banking / Economy News

66. During the quarter ended December 31, 2023, the Reserve Bank of India has received one application under the Guidelines for ‘on tap’
Licensing of Small Finance Banks in the Private Sector dated December 05, 2019 from Fino Payments Bank Limited.
Consider the following statement regarding Small Finance Banks. Which of the following statement is incorrect?
1. Minimum paid-up voting equity capital / net worth requirement shall be ₹ 200 crore
2. As per the current FDI policy, the aggregate foreign investment in a private sector bank from all sources will be allowed upto a
maximum of 74 per cent of the paid-up capital of the bank
3. SFBs are granted the scheduled bank status after being operational and are deemed suitable under section 42 of the RBI Act,1934.
4. They are required to maintain a minimum Capital to Risk-Weighted Assets Ratio (CRAR) of 15%.
5. They are required to extend 60% of their Adjusted Net Bank Credit to Priority Sector Lending.

 During the quarter ended December 31, 2023, the Reserve Bank of India has
received one application under the Guidelines for ‘on tap’ Licensing of Small
Finance Banks in the Private Sector dated December 05, 2019 from Fino Payments
Bank Limited.
 According to the guidelines, payments banks were allowed to apply for a
conversion into SFB post five years of operations along with meeting other
guidelines mandated by the RBI.
 The bank must also have a minimum paid-up voting equity capital or net worth of
Rs 200 crore.
 For Primary (Urban) Co-operative Banks (UCBs), desirous of voluntarily transiting
into Small Finance Banks (SFBs) initial requirement of net worth shall be at ₹ 100
crore which will have to be increased to ₹ 200 crore within five years from the
date of commencement of business.
 In July 2023, the board of Fino Payments Bank, which started operations in 2017,
gave approval to the transition into a small finance bank, subject to necessary
approval.

Click Here For Ultimate Bundle PDF Course 2022| Click Here to Subscribe Our Yearly Mock Test Package Follow Us- Telegram. Facebook , Twitter ,Instagram
Weekly Optimum Current Q & A PDF – Based On Recent Exam Trends Second Week of January (9th January –16th January 2024)

 During the July-September quarter of FY24, the net profit of the bank had increased by 41.5 per cent year-on-year (Y-o-Y) to Rs 19.5 crore.
The revenue increased by 18.2 per cent to Rs 358.6 crore.
 Rishi Gupta, MD & CEO of Fino Payments Bank.
 News Link - Reserve Bank of India - Press Releases (rbi.org.in) Reserve Bank of India - Press Releases (rbi.org.in)

 Small Finance Banks (SFBs) are specialized banks that are licensed by RBI to provide financial services and products to low-income
individuals and underserved communities, including microfinance and micro-enterprise services, as well as other basic banking services.
 The small finance bank shall be registered as a public limited company under the Companies Act, 2013.
 It will be licensed under Section 22 of the Banking Regulation Act, 1949 and governed by the provisions of the Banking Regulation Act,
1949; Reserve Bank of India Act, 1934; Foreign Exchange Management Act, 1999; Payment and Settlement Systems Act, 2007; Credit
Information Companies (Regulation) Act, 2005; Deposit Insurance and Credit Guarantee Corporation Act, 1961; other relevant Statutes and
the Directives, Prudential Regulations and other Guidelines/Instructions issued by RBI and other regulators from time to time. SFBs are
granted the scheduled bank status after being operational and are deemed suitable under section 42 of the RBI Act,1934.
 They are required to maintain a minimum Capital to Risk-Weighted Assets Ratio (CRAR) of 15%.
 They are required to extend 75% of their Adjusted Net Bank Credit to Priority Sector Lending.
 SFBs are required to open at least 25% of their total branches in unbanked rural areas.
 The minimum paid-up voting equity capital for small finance banks shall be Rs.200 crore.
 SFBs are required to maintain at least 50% of their loan portfolio as microfinance and advances of up to Rs. 25,00,000.
 The foreign shareholding in the small finance bank would be as per the Foreign Direct Investment (FDI) policy for private sector banks as
amended from time to time. As per the current FDI policy, the aggregate foreign investment in a private sector bank from all sources will
be allowed upto a maximum of 74 per cent of the paid-up capital of the bank (automatic upto 49 per cent and approval route beyond 49
per cent to 74 per cent).
 News Link - Reserve Bank of India - Database (rbi.org.in) Reserve Bank of India - Press Releases (rbi.org.in)

67. Recently as per report Reserve Bank of India (RBI) Governor Shaktikanta Das will attend the 2024 World Economic Forum (WEF) Annual
Meeting between January 15-19 in Davos, Switzerland.
The theme of this year’s WEF Annual Meeting is ________.
A. Reducing Carbon Emission
B. Globalization 4.0
C. Rebuilding Trust
D. Creating a Shared Future in a Fractured World
E. Working Together, Restoring Trust

 Reserve Bank of India (RBI) Governor Shaktikanta Das will attend the 2024 World Economic Forum (WEF)
Annual Meeting between January 15-19 in Davos, Switzerland.
 He is slated to address a Luncheon with CII Session on January 17 with theme “High Growth, Low Risk: The
India Story”.
 This is the first time ever an RBI Governor will attend and give an address at a WEF Annual Meeting at
Davos.
 The theme of this year’s WEF Annual Meeting is ‘Rebuilding Trust’.
 This is the first time ever an RBI Governor will attend and give an address at a WEF Annual Meeting at
Davos, they added. The theme of this year’s WEF Annual Meeting is ‘Rebuilding Trust’.
 The World Economic Forum is an international non-governmental organization for public–private sector collaboration based in Cologny,
Canton of Geneva, Switzerland.
 It was founded on 24 January 1971 by German engineer Klaus Schwab
 News Link - RBI Guv Shaktikanta Das to Address WEF Annual Meet at Davos on Jan 17 - The Hindu BusinessLine

Click Here For Ultimate Bundle PDF Course 2022| Click Here to Subscribe Our Yearly Mock Test Package Follow Us- Telegram. Facebook , Twitter ,Instagram
Weekly Optimum Current Q & A PDF – Based On Recent Exam Trends Second Week of January (9th January –16th January 2024)

68. In January as per report DLabs at the Indian School of Business (ISB) along with RBI Innovation Hub and ___________ had joined hands to
launch a startup accelerator programme named ___________.
A. Canara Bank
B. Bank Of Baroda
C. Punjab National Bank
D. Union Bank of India
E. State Bank Of India

 HyderabadDLabs at the Indian School of Business (ISB) launched ‘Build for Billions’, a startup accelerator programme, themed around
financial inclusion for the informal economy, in partnership with Reserve Bank Innovation Hub (RBIH) and Union Bank of India.
 ‘Build for Billions’ is a startup accelerator programme. It is themed around financial inclusion for the informal economy.
 It will support visionary fintech startups devoted to addressing the issues/hurdles/challenges faced by gig workers, migrant workers,
domestic workers, street vendors, carpenters, plumbers, electricians, and construction labourers.
 ‘Build for Billions’ is the 50-day equity-free accelerator by the business incubator DLabs.
 It offers comprehensive support, which include mentorship, capacity building, resources, and networking opportunities.
 Startups can apply for this programme by 21 January, 2024.
 The Reserve Bank Innovation Hub (RBIH) is a wholly-owned subsidiary of the RBI. Rajesh Bansal is its CEO.
 It has been set-up to facilitate and promote an environment which accelerates innovation across the financial sector. Rajesh Bansal, CEO,
RBIH
 Hyderabad DLabs is a business incubator at the Indian School of Business (ISB).
 News Link - DLabs at ISB launches `Build for Billions’ jointly with RBI Innovation Hub, Union Bank of India - The Hindu BusinessLine

69. In January as per report Reserve Bank of India has imposed a penalty of ₹1 crore on Punjab and Sind Bank for _________and
___________crore penalty on Dhanlaxmi Bank for non-compliance with certain directions on lending activities, KYC and deposit rates.
A. for violating norms on ‘Loans and Advances, ₹1.20 crore
B. for violating rules on risk management, outsourcing, recovery agents, and loan management, ₹2.20 crore
C. for violations in directions on acceptance of deposits from non-residents (NRIs), ₹1.90 crore
D. for violating regulatory standards , ₹4.25 crore
E. for a shortfall in soiled note remittances, ₹1.50 crore

 The Reserve Bank of India has imposed a penalty of ₹1 crore on Punjab and Sind Bank and
₹1.2 crore on Dhanlaxmi Bank for non-compliance with certain directions on lending
activities, KYC and deposit rates.
 The central bank said that its inspection of Punjab and Sind Bank for FY22 revealed that the
public sector lender violated norms on ‘Loans and Advances – Statutory and Other
Restrictions’.
 Dhanlaxmi Bank, which has incurred a penalty of ₹1.20 crore, violated regulatory norms on
‘Loans and Advances – Statutory and Other Restrictions’, ‘Reserve Bank of lndia (Know Your
Customer (KYC)) Directions, 2016’ and ‘Reserve Bank of lndia (Interest Rate on Deposits)
Directions, 2016’.
 In addition, the RBI also imposed a penalty of ₹29.5 lakh on ESAF Small Finance Bank for
breach of guidelines on ‘Customer Service in Banks’.
 During FY22, the bank allowed certain Basic Savings Bank Deposit (BSBD) account holders
to open savings bank deposit account, and failed to close certain savings bank deposit
accounts within thirty days from the date of opening of BSBD accounts for such customers.
 Punjab & Sind Bank is an Indian public sector bank headquartered in New Delhi. Swarup
Kumar Saha (MD & CEO).

Click Here For Ultimate Bundle PDF Course 2022| Click Here to Subscribe Our Yearly Mock Test Package Follow Us- Telegram. Facebook , Twitter ,Instagram
Weekly Optimum Current Q & A PDF – Based On Recent Exam Trends Second Week of January (9th January –16th January 2024)

 Dhanlaxmi Bank Ltd is an Indian private sector bank headquartered in Thrissur, Kerala. J. K. Shivan; (CEO & MD).
 ESAF Small Finance Bank is an Indian small finance bank headquartered in Thrissur, Kerala.
 News Link - Reserve Bank of India - Press Releases (rbi.org.in) Reserve Bank of India - Press Releases (rbi.org.in) Reserve Bank of India - Press
Releases (rbi.org.in) RBI imposes a penalty of ₹1 crore on Punjab & Sind Bank, Dhanlaxmi Bank - The Hindu BusinessLine

70. In January as per report RBI released a ‘draft framework for recognising Self-Regulatory Organisations (SRO) for_________ laying down
the characteristics of the SRO, and includes the required functions and governance standards.
A. Insurance Sector
B. FinTech Sector
C. Banking Sector
D. Infrastructure Sector
E. Export – Import Sector

 The Reserve Bank of India released a 'draft


framework for recognising Self-Regulatory
Organisations (SRO) for FinTech Sector',
stating the characteristics of a FinTech SRO,
including the required functions and
governance standards.
 RBI stated that self-regulation within the
FinTech sector is a preferred approach for
achieving the desired balance.
 According to the norms, SROs for fintech
sector is expected to operate objectively,
with credibility and responsibility under the
oversight of the regulator.

 According to the draft framework, the SRO-FT should put in place systems for managing ‘user harm’ instances that come to its notice or are
referred to it by the Reserve Bank or any other stakeholder and should not set up entities / offices overseas without the prior approval of the
Reserve Bank.
 The SRO-FT would derive its strength from its membership, ensuring that it is truly representative of the FinTech sector.
 On the eligibility and membership front, draft norms said the applicant should be set up as a not-for-profit company and should have
sufficient net worth and demonstrate the capability of establishing the necessary infrastructure to fulfil the responsibilities of Fintech
Self Regulatory Organisation (SRO-FT) effectively, and consistently.
 The applicant should be set up as a not-for-profit company registered under Section 8 of the Companies Act, 2013. The applicant SRO-FT
should represent the FinTech sector with membership across entities of all size, stage and activities.
 The SRO-FT should not set up entities / offices overseas without the prior approval of the Reserve Bank.
 News Link - Reserve Bank of India - Press Releases (rbi.org.in) Rbi Fintech Sros: RBI releases draft framework for recognising FinTech SROs, ET
BFSI (indiatimes.com)

71. Consider the following statement regarding NITI Aayog’s Discussion Paper ‘Multidimensional Poverty in India since 2005-06’. Which of the
following statement is incorrect?
1. According to a NITI Aayog report, 24.82 crore people came out of multi-dimensional poverty in India in 9 years to 2022-23.
2. Bihar registered the largest decline followed by Uttar Pradesh and Madhya Pradesh.
3. As per NITI Aayog’s discussion paper, multidimensional poverty in India decreased from 29.17% in 2013-14 to 11.28% in 2022-23.
4. As per NITI Aayog, the national multidimensional poverty judges simultaneous deprivations across three dimensions health, education,
and standard of living.
5. NITI Aayog’s National Multidimensional Poverty Index (MPI) uses the Alkire Foster methodology to assess the decline in poverty rates.

Click Here For Ultimate Bundle PDF Course 2022| Click Here to Subscribe Our Yearly Mock Test Package Follow Us- Telegram. Facebook , Twitter ,Instagram
Weekly Optimum Current Q & A PDF – Based On Recent Exam Trends Second Week of January (9th January –16th January 2024)

 According to NITI Aayog’s Discussion Paper


‘Multidimensional Poverty in India since 2005-06’,
24.82 crore people came out of multi-dimensional
poverty in India in 9 years to 2022-23.
 The discussion paper was released today by Prof
Ramesh Chand, Member, NITI Aayog in presence of
Shri B. V. R. Subrahmanyam, CEO NITI Aayog.
 Oxford Policy and Human Development Initiative
(OPHI) and United Nations Development
Programme (UNDP) have provided technical inputs
for this paper.
 Uttar Pradesh registered the largest decline in the
number of poor with 5.94 crore people escaping
multidimensional poverty during the last nine years
followed by Bihar at 3.77 crore, Madhya Pradesh at
2.30 crore and Rajasthan at 1.87 crore.
 As per NITI Aayog’s discussion paper,
multidimensional poverty in India decreased from
29.17% in 2013-14 to 11.28% in 2022-23.
 All 12 indicators of MPI have recorded significant
improvement during the entire study period.
 2.75 crore people came out of multidimensional
poverty every year.
 As per NITI Aayog, the national multidimensional
poverty judges simultaneous deprivations across
three dimensions.

 These three dimensions are health, education, and standard of


living.
 These three dimensions are represented by 12 sustainable
development goals-aligned indicators.
 These include nutrition, child and adolescent mortality, maternal
health, years of schooling, school attendance, cooking fuel,
sanitation, drinking water, electricity, housing, assets, and bank
accounts.
 NITI Aayog’s National Multidimensional Poverty Index (MPI) uses
the Alkire Foster methodology to assess the decline in poverty
rates.
 National MPI covers 12 indicators. On the other hand, global MPI
covers 10 indicators.
 As per NITI Aayog CEO BVR Subrahmanyam, “Government has a
goal to bring down multidimensional poverty to below 1%.”

Click Here For Ultimate Bundle PDF Course 2022| Click Here to Subscribe Our Yearly Mock Test Package Follow Us- Telegram. Facebook , Twitter ,Instagram
Weekly Optimum Current Q & A PDF – Based On Recent Exam Trends Second Week of January (9th January –16th January 2024)

 According to the discussion paper, India is all set to reach single-digit poverty levels during 2024.
 The discussion paper said, “India is likely to achieve Sustainable Development Goals (SDGs) 1.2 (reducing multidimensional poverty by at
least half) much ahead of 2030.”
 The recent National MPI was based on National Family Health Surveys 4 (2015-16) and 5 (2019-21).
 News Link - Press Information Bureau (pib.gov.in)

72. In January as per report Oil and Natural Gas Corporation (ONGC) has started oil production from its deep-sea project in the Krishna
Godavari basin in the Bay of Bengal.
ONGC has started producing from the Cluster-2 project in the KG-DWN-98/2 block.
This 98/2 project is likely to increase ONGC’s total oil and gas production by ________per cent and ________per cent, respectively.
A. 12%, 15%
B. 11%, 15%
C. 15%, 19%
D. 16%, 20%
E. 18%, 16%

 Oil and Natural Gas Corporation (ONGC) has started oil production
from its deep-sea project in the Krishna Godavari basin in the Bay of
Bengal.
 ONGC has started producing from the Cluster-2 project in the KG-DWN-
98/2 block. It will slowly increase output.
 This 98/2 project is likely to increase ONGC’s total oil and gas
production by 11 per cent and 15 per cent, respectively.
 Around 45 thousand barrels per day and more than 10 million standard
cubic metres per day of gas will be produced.
 This project will add 7 per cent to current national oil production and 7
per cent to national Natural Gas production.
 This project is located nearly 35 kilometres off the coast of Andhra
Pradesh at a depth of 300-3,200 metres.
 The discoveries in the block are divided into three clusters -- Cluster 1, 2
and 3. Cluster 2 is being put into production first.
 Oil and Natural Gas Corporation was formed in 1956. It is
headquartered in New Delhi. It comes under the Ministry of Petroleum
and Natural Gas.
 Arun Kumar Singh is the Chairman & CEO of ONGC. It was conferred the Maharatna status in 2010.
 News Link - ONGC starts oil production from its flagship deep-sea project in Krishna Godavari basin in Bay of Bengal
(newsonair.gov.in) ONGC flows oil from east coast block - The Hindu BusinessLine

73. In January as per report Oil and Natural Gas Corporation (ONGC) has discovered two significant natural gas reserves in the Mahanadi basin
block in the Bay of Bengal.
The first discovery has been named as ___________.
A. Durga
B. Moga
C. Pinth
D. Kumbhir
E. Utkal

Click Here For Ultimate Bundle PDF Course 2022| Click Here to Subscribe Our Yearly Mock Test Package Follow Us- Telegram. Facebook , Twitter ,Instagram
Weekly Optimum Current Q & A PDF – Based On Recent Exam Trends Second Week of January (9th January –16th January 2024)

 State-owned Oil and Natural Gas Corporation (ONGC) has made two significant back-to-back natural gas discoveries
in a Mahanadi basin deepwater block in the Bay of Bengal as its calculated game plan of venturing into high-risk
deep water exploration starts yielding results.
 The firm made the discoveries in the block MN-DWHP-2018/1, which it had won in the third round of auction under
the open acreage licensing policy in 2019.
 The first discovery, named Uktal, is in 714 metres of water depth and flowed more than 3 lakh cubic metres per day
of gas during initial testing.
 The other find is at a water depth of 1,110 metres.
 For a nation that imports roughly half of its gas needs, finding new reserves augurs well for its energy security.
India is targeting raising the share of natural gas in its energy basket to 15 per cent by 2030 from the current
6.3 per cent and more domestic production will aid that.
 News Link - ONGC makes back-to-back gas discoveries in Mahanadi basin block - The Economic Times
(indiatimes.com)

74. In January as per report _________ plans to set up an operations support subsidiary to provide value-added services
while optimising costs, especially for operations beyond large cities.
A. Indian Bank
B. Canara Bank
C. YES Bank
D. Bandhan Bank
E. ICICI Bank

 Chennai-based Indian Bank plans to set up an operations support


subsidiary to provide value-added services while optimising costs,
especially for operations beyond large cities.
 Two large public sector banks — State Bank of India (SBI) and Bank of
Baroda (BoB) — have already formed such units for servicing customers
in rural and semi-urban areas.
 After Indian Bank’s request for proposal (RFP) to empanel manpower
consultants for the subsidiary, the bank has received in-principle approval
from the Reserve Bank of India (RBI).
 In July 2022, country’s largest lender SBI set up a subsidiary State Bank
Operations Support Services Pvt Ltd (SBOSS) for providing operations
support services at branches in rural and semi-urban areas of the country.
 SBOSS, which is headquartered in Delhi, will also work as a corporate business correspondent of the SBI.
 Mumbai-based BoB has established Baroda Global Shared Services, a wholly-owned subsidiary, to integrate service functions into a single
entity.
 News Link - Indian Bank to float operations support subsidiary, aims to optimise cost (business-standard.com)

75. In January as per report ________ has excluded the Red Sea shipping route from standard marine insurance after rebel attacks on vessels
in the area.
A. Kotak General Insurance Company
B. HDFC ERGO General Life Insurance
C. Tata AIG General Insurance
D. SBI General Insurance Company
E. Oriental General Insurance Company

Click Here For Ultimate Bundle PDF Course 2022| Click Here to Subscribe Our Yearly Mock Test Package Follow Us- Telegram. Facebook , Twitter ,Instagram
Weekly Optimum Current Q & A PDF – Based On Recent Exam Trends Second Week of January (9th January –16th January 2024)

 Tata AIG General Insurance has excluded the Red Sea


shipping route from standard marine insurance after
rebel attacks on vessels in the area, while other
insurers are raising premiums for cargo being carried
through the vulnerable water channel that helps slash
the maritime distance between continental Europe and
Asia.
 Cancellation notices in relation to war, terrorism, and
piracy risks in the surrounding areas of the Indian
Ocean, the Gulf of Aden and the Southern Red Sea
have been sent out effective January 6.
 Insurers have raised premiums, while others are
issuing notices of cancellations of policies related to
war, terrorism, and piracy risks in areas like the Indian
Ocean, Gulf of Aden, Southern Red Sea, and Cabo
Delgado.
 Some insurance companies have cancelled war and
strike cover for oil companies taking this route.
 The Cancellation clause in the policy gives either party, the insurer or the insured to cancel the policy after giving 30 days notice.
 Red Sea is a semi-enclosed inlet (or extension) of the Indian Ocean between the continents of Africa and Asia. It is one of the world’s
warmest seas.
 It is connected to the Arabian Sea and the Indian Ocean to the south through the Gulf of Aden and the narrow strait of Bab El-Mandeb.
 News Link - TATA AIG General Insurance excludes Red Sea route, others raise premiums - The Economic Times (indiatimes.com)

76. In January as per report LIC has cleared the proposal to invest in a new company, promoted by the NHB, for residential mortgage-backed
securities.
NHB is an All-India Financial Institution (AIFl) established under the National Housing Bank Act, 1987.
Who is the current MD of National Housing Bank?
A. Shri Maneesh Kumar Singh
B. Shri Sarada Kumar Hota
C. Shri Rajesh Kumar Mehta
D. Shri Amitabh Joshi
E. Shri Kalyan Choubey

 Insurance behemoth Life Insurance Corporation of India (LIC) is planning to acquire up to 10 percent equity stake in a
company promoted by the National Housing Bank (NHB).
 The Board of Directors of LIC gave their approval to the proposal to invest in the NHB promoted entity that would
focus on residential mortgage-backed securities (RMBS).
 RMBS are basically debt-based assets backed by the interest paid on residential loans. RMBS are bonds which are
secured against a large pool of residential mortgages (home loans).
 NHB is an All-India Financial Institution (AIFl) established under the National Housing Bank Act, 1987.
 It is wholly owned by the Government of India.
 NHB supervises HFCs, while regulation of HFCs is with the RBI.
 Head Office: New Delhi, MD - Shri Sarada Kumar Hota
 NHB RESIDEX: It is the country’s first official housing price index (HPI). It captures movements in the prices of
residential real estate prices.
 News Link - LIC to pick up to 10% equity stake in NHB promoted ‘RMBS’ entity - The Hindu BusinessLine

Click Here For Ultimate Bundle PDF Course 2022| Click Here to Subscribe Our Yearly Mock Test Package Follow Us- Telegram. Facebook , Twitter ,Instagram
Weekly Optimum Current Q & A PDF – Based On Recent Exam Trends Second Week of January (9th January –16th January 2024)

77. In January as per report __________announced its ambitious plan to increase its financial sanctioning in the infrastructure space,
including roads and highways, to ₹1 lakh crore in the current financial year.
A. Rail Vikas Nigam Limited
B. GAIL Limited
C. Indian Railway Finance Corporation
D. Power Finance Corporation Ltd
E. Rural Electrification Corporation Limited

 REC Ltd. recently announced its ambitious plan to increase its financial sanctioning in the infrastructure space, including roads and
highways, to ₹1 lakh crore in the current financial year.
 In line with the national focus on renewable energy, REC also aims to scale up its portfolio in this sector.
 The company has set a target to enhance its renewable energy portfolio tenfold, aiming to reach ₹3 lakh crore by 2030, up from the
current ₹30,000 crore.
 REC Limited (formerly Rural Electrification Corporation Limited) is a Central Public Sector Undertaking under the Ministry of Power
involved in financing projects in the complete power sector value chain from generation to distribution.
 It is registered with the RBI as a Non-Banking Finance Company (NBFC), a Public Financial Institution (PFI) and an Infrastructure Financing
Company (IFC).
 REC provides long-term loans and other financing products to State, Centre, and Private Companies for creation of infrastructure assets in
the country.
 Established in 1969. CMD, REC Limited, Shri Vivek Kumar Dewangan.
 News Link - pib.gov.in/PressReleaseIframePage.aspx?PRID=1994397 REC sets ₹1 lakh crore sanction target for infrastructure space including
roads and highways in FY24, ET EnergyWorld (indiatimes.com)

78. In January as per report Investors in the United Arab Emirates (UAE) are set to get their first regional exchange-traded fund (ETF) tracking
Indian equities.
Which of the following is launching the Chimera S&P India Shariah ETF, replicating the performance of the S&P India Shariah Liquid 35/20
Capped Index?
A. Scottish Mortgage Capital LLC
B. Fundsmith Equity Fund
C. Dunn Capital LLC
D. Ardevora Global Equity Funds
E. Lunate Capital LLC

 Investors in the United Arab Emirates (UAE) are set to get their first
regional exchange-traded fund (ETF) tracking Indian equities.
 Lunate Capital LLC is launching the Chimera S&P India Shariah ETF,
replicating the performance of the S&P India Shariah Liquid 35/20
Capped Index.
 The fund will track the performance of Shariah-compliant Indian
equities listed on the BSE.
 Indian stocks rose for an eighth year in 2023 as India continues to be
one of the most favoured emerging markets.
 The new ETF is set to include large-cap Indian stocks such as Reliance
Industries, Infosys, and TCS, and is open for investor subscription from
Jan 12 to Jan 17.
 There were an estimated 3.5 million Indian expatriates in the UAE as of
2021, comprising about 30 per cent of the Gulf country’s population.
 UAE is considering investing as much as $50 billion in India, its second-largest trading partner.

Click Here For Ultimate Bundle PDF Course 2022| Click Here to Subscribe Our Yearly Mock Test Package Follow Us- Telegram. Facebook , Twitter ,Instagram
Weekly Optimum Current Q & A PDF – Based On Recent Exam Trends Second Week of January (9th January –16th January 2024)

 News Link - indian stocks: Indian stocks get first ETF in Gulf Region as UAE deepens ties - The Economic Times (indiatimes.com)

79. In January as per report ___________ became the first Indian PSU to successfully issue Yen Green Bonds of Japanese Yen (JPY) 61.1 billion
5-year, 5.25-year and 10-year Green bonds, issued under its US$ 10 billion Global Medium Term Notes Programme.
A. Power Finance Corporation Ltd
B. REC Limited
C. State Bank Of India
D. Bank Of Baroda
E. Indian Renewable Energy Development Agency

 REC Limited, a Maharatna Central Public Sector Enterprise under the Ministry of Power,
has successfully issued its inaugural Japanese Yen (JPY) 61.1 billion 5-year, 5.25-year and
10-year Green bonds, issued under its US$ 10 billion Global Medium Term Notes
Programme.
 Proceeds from the issue of the Bonds will be applied to finance the Eligible Green
Projects in accordance with the Company’s Green Finance Framework, RBI’s External
Commercial Borrowings Guidelines and the approvals granted by it from time to time.
 REC Limited’s eleventh venture into the international bond market and inaugural Yen
Bond issuance, which is also the first Yen Green Bonds issuance by any Indian PSU
 5-year, 5.25-year and 10-year bonds issued at yield of 1.76%, 1.79% and 2.20%
respectively
 Largest ever Euro-Yen issuance in South and South East Asia
 Largest Yen-denominated issuance from India
 Largest non-sovereign Yen-denominated issuance ever from South and South East Asia
 The transaction witnessed interest from both Japanese and international accounts, with
number of orders from each at 50%, international allocation being one of the highest for
any other Indian Yen deal.
 The notes will be rated Baa3/BBB–/BBB+ (Moody's/Fitch/JCR) and will be listed exclusively on
Global Securities Market of India International Exchange (India INX) and NSE IFSC in GIFT City,
Gandhinagar, Gujarat. DBS Bank, Mizuho, MUFG, and SMBC Nikko are the joint lead managers
for the issue.
 REC Ltd. continues to play a key strategic role in the flagship schemes of the Government for
the power sector and has been nodal agency for Pradhan Mantri Sahaj Bijli Har Ghar Yojana
(SAUBHAGAYA), Deen Dayal Upadhaya Gram Jyoti Yojana (DDUGJY) and National Electricity
Fund (NEF) Scheme.
 News Link - Press Information Bureau (pib.gov.in)

80. In January as per report ____________ and and the state-owned enterprise of Catamarca
province of Argentina CAMYEN SE signed an agreement for Lithium Exploration & Mining
Project in Argentina.
A. Coal India
B. NLC India Limited
C. Hindalco Industries
D. Khanij Bidesh India Limited
E. National Aluminium Company

Click Here For Ultimate Bundle PDF Course 2022| Click Here to Subscribe Our Yearly Mock Test Package Follow Us- Telegram. Facebook , Twitter ,Instagram
Weekly Optimum Current Q & A PDF – Based On Recent Exam Trends Second Week of January (9th January –16th January 2024)

 Ministry of Mines, Government of India has achieved a significant milestone with the signing of an agreement between Khanij Bidesh India
Limited (KABIL) and the state-owned enterprise of Catamarca province of Argentina CATAMARCA MINERA Y ENERGÉTICA SOCIEDAD DEL
ESTADO (CAMYEN SE) at Catamarca, Argentina on 15th January, 2024.
 This is the first ever lithium exploration and mining project by a Government Company of India. KABIL will start exploration and
development of 5 lithium brine blocks viz 1. Cortadera-I, 2. Cortadera-VII, 3. Cortadera-VIII, 4. Cateo-2022-01810132 and 5.
 KABIL is also preparing to set up a branch office at Catamarca, Argentina.
 The project cost is about 200 crores.
 Argentina is the part of “Lithium Triangle” along with Chile and Bolivia with more than half of world’s total lithium resources and having
the distinction of having 2nd largest lithium resources, 3rd largest lithium reserves and 4th largest production in world.
 Khanij Bidesh India Limited (KABIL), is a joint venture between NALCO, Hindustan Copper, and Mineral Exploration.
 News Link - Press Information Bureau (pib.gov.in)

81. In January as per report government has increased benchmark central financial assistance for the residential sector under the rooftop solar
program to give a push to the projects and cover the higher cost of solar panels.
Support for up to 3 kW capacity has been raised to _________per kW for general states.
A. Rs 12,000 per kW
B. Rs 18,000 per kW
C. Rs 14,000 per kW
D. Rs 16,000 per kW
E. Rs 20,000 per kW

 The government has increased benchmark central financial assistance for the
residential sector under the rooftop solar program to give a push to the projects
and cover the higher cost of solar panels.
 Support for up to 3 kW capacity has been raised to Rs 18,000 per kW for general
states, up 23.4%, from Rs 14,588 per kW earlier, according to a notification from
the Ministry of New and Renewable Energy.
 For over 3kW and up to 10 kW capacity, the assistance has been raised to Rs 9,000
per kW for general states from Rs 7,294 per kW.
 The last revision in the benchmark for central financial assistance for rooftop
projects was done about a year ago.
 The new rates will apply to all future bids that will be closed after January 20 and
claims submitted after January 5.
 According to a report by the Council on Energy, Environment and Water (CEEW) in
November, over 25 crore households across India have the potential to deploy 637
GW of solar energy capacity on rooftops.
 News Link - Govt ups financial support for residential rooftop solar projects - The
Economic Times (indiatimes.com)

82. In January as per report World Bank has released a report titled _____________ in which it kept its FY25 economic growth projection for
India unchanged at 6.4 per cent
A. Global Economic Outlook
B. Global Financial Development Report
C. Global Economic Prospects
D. Digital Dividends Reports
E. Global Competitiveness Report

Click Here For Ultimate Bundle PDF Course 2022| Click Here to Subscribe Our Yearly Mock Test Package Follow Us- Telegram. Facebook , Twitter ,Instagram
Weekly Optimum Current Q & A PDF – Based On Recent Exam Trends Second Week of January (9th January –16th January 2024)

 The World Bank kept its FY25 economic growth projection for India
unchanged at 6.4 per cent, mainly on account of strong domestic
demand, rising public infrastructure spending and strong private-
sector credit growth.
 However, it projected that the private consumption growth might
taper off due to high food inflation and diminishing pent-up
demand.
 In its biannual ‘Global Economic Prospects’ report, the World Bank
said India was likely to maintain the fastest growth rate among the
world’s largest economies, but its post-pandemic recovery was
expected to slow, with estimated growth of 6.3 per cent in FY24,
before recovering gradually to 6.5 per cent in FY26.
 The Washington-based organisation forecast that the world
economy will expand 2.4 per cent this year.
 On India, the multilateral development bank’s forecast did not take
into account the National Statistical Office’s (NSO’s) first advance
estimates, which had projected 7.3 per cent growth for India in FY24
— higher than the Reserve Bank of India’s estimate of 7 per cent —
assuming an investment-led recovery.
 News Link - World Bank keeps India's FY25 GDP growth estimate
unchanged at 6.4% (business-standard.com)

83. In January as per report ___________ has partnered with fintech


company Digivriddhi (DGV) to offer financial services catering to the
needs of dairy farmers and milk societies.
A. Karnataka Bank
B. Bandhan Bank
C. Jana Small Finance Bank
D. ESAF Small Finance Bank
E. Utkarsh Small Finance Bank

 Karnataka Bank has partnered with fintech company Digivriddhi (DGV) to offer financial services catering to the needs of dairy farmers
and milk societies.
 The services will be made available at village dairy co-operative societies associated with milk unions of Karnataka Milk Federation (KMF).
 To begin with, the services are now made available to milk societies associated with Chamarajnagar Milk Union.
 Srikrishnan H, Managing Director and Chief Executive Officer (CEO) of Karnataka Bank.
 Recently Karnataka Bank has also opened its 915th branch with a mini e-lobby at Ayodhya.
 News Link - Karnataka Bank inks pact with Digivriddhi to offer financial services to dairy farmers - The Hindu BusinessLine Karnataka Bank
opens branch in Ayodhya - The Hindu BusinessLine

84. Ministry of Coal has set a target to produce __________ million tonne (MT) coal exclusively from captive /commercial coal mines during FY
2024-25.
A. 186.63 million tonne
B. 175.63 million tonne
C. 193.63 million tonne
D. 181.63 million tonne
E. 172.63 million tonne

Click Here For Ultimate Bundle PDF Course 2022| Click Here to Subscribe Our Yearly Mock Test Package Follow Us- Telegram. Facebook , Twitter ,Instagram
Weekly Optimum Current Q & A PDF – Based On Recent Exam Trends Second Week of January (9th January –16th January 2024)

 The Ministry of Coal aims to produce 186.63 million tonne (MT) coal exclusively from captive /commercial coal
mines during FY 2024-25.
 Production will be further stepped up to 225.69 million tonne during FY 2025-26 and as per the present plans of the
Ministry, production target from such mines will be touching 383.56 million tonne by FY 2029-30.
 As per the latest figures (31 December, 2023) of the Ministry, 50 captive/commercial coal mines are under
production and out of which 32 mines are allocated to the power sector, 11 to non-regulated sector and seven
mines allocated for sale of coal.
 In December 2023, the total coal production from captive and commercial coal mines was 14.04 MT, up by 38% from
10.14 MT in the same month of the previous year.
 News Link - Press Information Bureau (pib.gov.in)

85. In January as per report Rural Development Ministry signed a memorandum of understanding (MoU) with State
Bank of India to facilitate enterprise financing for rural self-help groups (SHG).
State Bank of India introduced a specialised financial product Svyam Siddha, exclusively tailored for SHG women
entrepreneurs seeking loans up to ________lakhs.
A. Rs 2 lakhs
B. Rs 10 lakhs
C. Rs 5 lakhs
D. Rs 15 lakhs
E. Rs 1 lakhs
 The Rural Development Ministry and State Bank of India signed a memorandum of understanding (MoU) to
facilitate enterprise financing for rural self-help groups (SHG).
 The MoU was signed by Additional Secretary Deendayal Antyodaya Yojana - National Rural Livelihoods Mission,
Charanjit Singh and Chief General Manager, State Bank of India, Corporate office Mumbai, Shantanu Pendsey.
 State Bank of India introduced a specialised financial product Svyam Siddha, exclusively tailored for SHG women
entrepreneurs seeking loans up to Rs 5 lakhs.
 This initiative is designed to alleviate the challenges associated with extensive documentation requirements for bank
loan applications and reduce the turnaround time (TAT).
 DAY-NRLM will facilitate the loan application process and oversee loan repayment through its dedicated field cadre.
 A training toolkit package was also launched at the event to promote access to formal finance by SHG women entrepreneurs.
 The toolkit covers crucial topics like fundamentals of banking, concepts on financial statements, working capital calculation etc.
 Developed in collaboration with the International Finance Corporation (IFC) under the World Bank-funded National Rural Economic
Transformation Project (NRETP), this toolkit aims to enhance the capacities of State Rural Livelihoods Missions (SRLMs) to facilitate
increased access to formal finance by SHG members for their economic enterprises.
 News Link - Press Information Bureau (pib.gov.in)

86. In January as per report State Bank of India (SBI) has concluded the issue of USD 600 million senior unsecured fixed rate notes having
maturity of _____________ years at a coupon of 5 per cent, payable semi-annually, under Regulation-S.
A. 3 years
B. 5 years
C. 10 years
D. 15 years
E. 20 years

 State Bank of India (SBI) has concluded the issue of USD 600 million senior unsecured fixed rate notes having maturity of 5 years at a
coupon of 5 per cent, payable semi-annually, under Regulation-S.
 The Bonds will be issued through our London branch as of January 17 2024 and shall be listed on Singapore Stock Exchange and India
International Exchange, GIFT City.

Click Here For Ultimate Bundle PDF Course 2022| Click Here to Subscribe Our Yearly Mock Test Package Follow Us- Telegram. Facebook , Twitter ,Instagram
Weekly Optimum Current Q & A PDF – Based On Recent Exam Trends Second Week of January (9th January –16th January 2024)

 Under Regulation-S, offers and sales of securities that happen outside the United States are exempt from
SEC registration requirements.
 This fundraising comes on the heels of the bank concluding placement of USD 250 million senior unsecured
Green floating rate notes, referred to as “The Green Notes.”
 This “Green Notes” issuance, maturing on December 29, 2028, is part of SBI’s USD 10 Billion medium-term
note program and was conducted through a private placement facilitated by its London branch.
 The Green Notes rated BBB- by S&P, were issued on a floating rate basis of 1.20 per cent above SOFR
(secured overnight financing rate) and have been listed on the India International Exchange, India’s
largest bank said in a statement issued on January 4.
 News Link - SBI raises $600 million via 5-year senior unsecured fixed rate notes - The Hindu BusinessLine

87. In January as per report ____________has partnered with EazyDiner to launch platinum credit card.
A. Indian Bank
B. Canara Bank
C. IndusInd Bank
D. Bandhan Bank
E. YES Bank

 In partnership with EazyDiner, IndusInd Bank has launched the EazyDiner IndusInd Bank Platinum Credit
Card.
 When paying for both dining and takeaway at specific restaurants using the EazyDiner App and
PayEazy, the credit card offers an additional 500 discount.
 With assured discounts of 25% to 50% at over 2,000 restaurants, cardholders receive a three-month
EazyDiner Prime membership free of joining or annual fees.
 Every ninety days, users can renew their membership by paying ₹30,000.
 For every ₹100 spend, the card offers up to two reward points.
 IndusInd Bank is a Pune-based bank. It was started in 1994. Sumant Kathpalia is its CEO. Its tagline is
we make you feel richer.
 News Link - IndusInd Bank launches EazyDiner IndusInd Bank Platinum Credit Card - The Hindu BusinessLine

88. Consider the following statement regarding SmartFin. Which of the following statement is incorrect?
1. SmartFin has been announced by Canara Bank.
2. It is an end-to-end digital supply chain finance (SCF) platform.
3. It is powered by Veefin Solutions, a leading financial products and digital solutions provider offering innovative solutions.
4. Only 1 and 2
5. All of the above are correct

 The launch of SmartFin has been announced by Yes Bank.


 It is an end-to-end digital supply chain finance (SCF) platform.
 It is powered by Veefin Solutions, a leading financial products and digital solutions provider offering innovative solutions.
 The significance of Supply Chain Financing (SCF) as a model to drive working capital efficiencies has been growing.
 YES Bank was founded in 2004. It is headquartered in Mumbai. Prashant Kumar is its MD and CEO.
 News Link - Yes Bank deploys Veefin’s SmartFin supply chain finance solution - The Hindu BusinessLine Yes Bank deploys Veefin’s SmartFin
supply chain finance solution (ibsintelligence.com)

89. In January as per report _______________ has become 26th member of the International Aluminium Institute (IAI).

Click Here For Ultimate Bundle PDF Course 2022| Click Here to Subscribe Our Yearly Mock Test Package Follow Us- Telegram. Facebook , Twitter ,Instagram
Weekly Optimum Current Q & A PDF – Based On Recent Exam Trends Second Week of January (9th January –16th January 2024)

A. Hindustan Aluminium Company


B. National Aluminium Company Limited
C. Vedanta Aluminium
D. Bharat Aluminium Company Limited
E. Jindal Aluminium

 Vedanta Aluminium, one of the world’s largest producers of aluminium, has become 26th member of the
International Aluminium Institute (IAI).
 The International Aluminium Institute is the only body representing the global primary aluminium industry.
 Vedanta manufactured 2.29 million tonnes (mt) of aluminium in the 2022-23 fiscal, nearly 60 per cent of the
nation’s total production.
 This also places Vedanta Aluminium among the world’s top 10 producers of the metal, projected as the “metal
of the future” for enabling the world’s energy transition while supporting key engineering technologies and
sustainable applications.
 The International Aluminium Institute (IAI) is the only body representing the global primary aluminium
industry. Established in 1972, the current IAI membership includes the leading bauxite, alumina and aluminium
companies globally. Headquarters: London.
 Mr. John Slaven, CEO, Vedanta Aluminium
 News Link - Vedanta joins International Aluminium Institute to strengthen sustainability efforts - The Hindu
BusinessLine

90. In Januaary as per report _____________has crossed the milestone of issuing 1,000 electronic Bank Guarantees (e-BGs) for trade finance
on the National E-Governance Services Ltd (NeSL) platform within eight months of the launch of e-BGs.
A. Bank of Baroda
B. Canara Bank
C. State Bank Of India
D. HDFC Bank
E. ICICI Bank

 Bank of Baroda (BoB) has crossed the milestone of issuing 1,000 electronic Bank Guarantees (e-BGs) for trade finance on the National E-
Governance Services Ltd (NeSL) platform within eight months of the launch of e-BGs.
 e-BG replaces the traditional paper-based process for trade finance and results in substantial reduction in the turnaround time in issuing a
BG.
 In an e-BG, physical stamping is replaced with e-stamping by NeSL.
 NeSL is India’s first Information Utility and is registered with the Insolvency and Bankruptcy Board of India under the aegis of the
Insolvency and Bankruptcy Code, 2016.
 It offers a platform for the issuance of inland BGs through electronic mode.
 Debajyoti Ray Chaudhuri, MD & CEO of NeSL.
 News Link - BoB’s e-Bank Guarantee issuance for trade finance crosses 1,000 mark - The Hindu BusinessLine

91. Recently as per report Reliance will set up India's first and world-class carbon fibre facility at ___________.
A. Jamanagar
B. Ahemdabad
C. Surat
D. Kutch
E. Hazira

Click Here For Ultimate Bundle PDF Course 2022| Click Here to Subscribe Our Yearly Mock Test Package Follow Us- Telegram. Facebook , Twitter ,Instagram
Weekly Optimum Current Q & A PDF – Based On Recent Exam Trends Second Week of January (9th January –16th January 2024)

 Billionaire Mukesh Ambani said his company Reliance will set up


India's first and world-class carbon fibre facility at Hazira.
 Speaking at the Vibrant Gujarat Summit here, Ambani said Reliance
was, is, and will always remain a Gujarati company.
 Reliance has invested over $150 billion (₹12 lakh crore) in creating
world-class assets and capacities across India in last just 10 years. Of
this, more than one third has been invested in Gujarat alone.
 Reliance started building the Dhirubhai Ambani Green Energy Giga
Complex over 5,000 acres in Jamnagar.
 This will generate a large number of green jobs and enable production
of green products and materials that will make Gujarat the leading
exporter of such goods and the company is ready to roll this out in the
second half of 2024.
 Reliance Jio completed the fastest roll-out of 5G infrastructure
anywhere in the world.
 For India's bid for 2036 Olympics, Reliance and Reliance Foundation
will join forces with several other partners in Gujarat to improve the
education, sports, and skills infrastructure that will nurture the
champions of tomorrow in various Olympics sports.
 News Link - Reliance to set up carbon fibre facility in Gujarat - The
Hindu BusinessLine

92. Which of the following is not among the five standalone health insurers?
A. Star Health Insurance
B. Care Health Insurance
C. Niva Bupa Health Insurance
D. Aditya Birla Health Insurance
E. TATA AIG Health Insurance

 Standalone Health Insurers (SAHI) have recorded 26 per cent growth in retail
annual premium equivalent (APE) in December 2023 at ₹2900 crore (₹2,300
crore).
 For the year ended December 2023, retail APE was up 25 per cent at ₹22,500
crore (₹17,900 crore), official IRDAI and General Insurance Council data
showed.
 All the five standalone health insurers—Star Health, Care Insurance, Niva
Bupa, Aditya Birla Health Insurance and ManipalCigna-recorded double digit
growth for both December 2023 and year-to-date as of December 2023.
 Among these five standalone health insurers, the best performing company was Star Health, which recorded 15 per cent y-o-y growth in
retail APE at ₹1,300 crore in December 2023 and 18 per cent growth year-to-date in end December 2023 at ₹10,300 crore (₹8,800 crore).
 News Link- Standalone health insurers sizzle in December 2023 - The Hindu BusinessLine

93. Recently as per data from the Association of Mutual Funds in India (AMFI) India’s mutual fund assets crossed the ________trillion mark
for the first time in December 2023.
A. Rs 10 trillion
B. Rs 20 trillion
C. Rs 50 trillion
D. Rs 75 trillion
E. Rs 100 trillion
Click Here For Ultimate Bundle PDF Course 2022| Click Here to Subscribe Our Yearly Mock Test Package Follow Us- Telegram. Facebook , Twitter ,Instagram
Weekly Optimum Current Q & A PDF – Based On Recent Exam Trends Second Week of January (9th January –16th January 2024)

 India’s mutual fund assets crossed the Rs 50-trillion mark for the first time in
December 2023, hitting RS 50.77 trillion, data from the Association of Mutual
Funds in India (Amfi) showed.
 Inflows into systematic investment plans (SIPs) touched another new high of
RS 17,611 crore in December, 2023. The number of SIP accounts also hit a high
of 76.36 million.
 Small-cap funds accounted for most of the investments for the 15th straight
month at 38.57 billion rupees, more than one-fifth of the overall equity
mutual fund inflows.
 Association of Mutual Funds in India, abbreviated as AMFI, is the association
of all the asset management companies of SEBI registered mutual funds in
India. It was incorporated on 22 August 1995, as a non-profit organisation.
As of now, 44 Asset Management Companies that are registered with SEBI,
are its members.
 Venkat Chalasani, CEO of Amfi, Navneet Munot, chairman of AMFI.
 News Link - MF assets top record Rs 50 trillion in December - Market News |
The Financial Express
 Old Bridge Asset Management Company Private Ltd will be launched on
January 10 and become India’s 44th fund house.
 The fund house backed by Old Bridge Capital Management Private Limited
announced the new fund offer (NFO) -- Old Bridge Focused Equity Fund, an
open-ended equity scheme investing in up to 30 stocks.
 Investors can participate through a systematic investment plan with a
minimum investment of Rs 2,500 and in multiples of Rs 1 thereafter.
 News Link - Will maverick mid-cap fund manager Kenneth Andrade do an encore with Old Brid Old Bridge Mutual Fund launches maiden
equity-ended scheme | Headlines (devdiscourse.com)e MF? (moneycontrol.com)

94. Recently as per report Bloomberg Emerging Market (EM) Local Currency Index will include the Indian government’s Fully Accessible Route
(FAR) bonds.
Consequently, in September 2024, the Indian FAR bonds will be included with an initial weight of _________% of their whole market value
in the EM Local Currency indices.
A. 15%
B. 25%
C. 30%
D. 20%
E. 10%

 Bloomberg Emerging Market (EM) Local Currency


Index will include the Indian government’s Fully
Accessible Route (FAR) bonds.
 This will be done gradually beginning in
September 2024 and lasting five months.
 Consequently, in September 2024, the Indian FAR
bonds will be included with an initial weight of
20% of their whole market value in the EM Local
Currency indices.
 Throughout the five months period that will end in
January 2025, the weight of FAR bonds will
increase monthly in increments of 20% of their full
market value.
Click Here For Ultimate Bundle PDF Course 2022| Click Here to Subscribe Our Yearly Mock Test Package Follow Us- Telegram. Facebook , Twitter ,Instagram
Weekly Optimum Current Q & A PDF – Based On Recent Exam Trends Second Week of January (9th January –16th January 2024)

 In January 2025, they will be weighted at their full market value (100%) in the indices.
 After India FAR bonds are fully incorporated into the Bloomberg Emerging Market 10% Country Capped Index, their weight will be totally
capped at 10%.
 By then, the Indian rupee will become the third-largest currency component in the Bloomberg Emerging Market Local Currency Index.
 The G-Sec FAR is a window that allows foreign investors to purchase Indian government bonds without many restrictions, facilitating easier
investment.
 The Reserve Bank of India (RBI) has introduced the ‘Fully Accessible Route’ (FAR) from April 1, 2020.
 This channel allows non-residents to invest in specified government bonds without any restrictions.
 VRR is another scheme introduced by the RBI to encourage Foreign Portfolio Investors (FPIs) for long-term investments in Indian debt
markets.
 It requires a minimum retention period of three years, with FPIs maintaining a minimum of 75% of the allocated amount in India.
 Investment limits are available on tap and allotted by Clearing Corporation of India Ltd. (CCIL) on a ‘first come first served’ basis.
 News Link - Economists predict $2-3 billion additional inflows into India’s debt markets with Bloomberg EM Index inclusion - The Hindu
BusinessLine

95. In January as per report __________ has launched the Green Rupee Term Deposit (SGRTD) of 1,111 days, 1,777 days, and 2.222 days tenor
for resident individuals, non-individuals, and NRI customers.
A. State Bank of India
B. Canara Bank
C. Indian Bank
D. Bank Of India
E. Punjab National Bank

 State Bank of India (SBI) has launched the SBI Green Rupee Term Deposit (SGRTD)
of 1,111 days, 1,777 days, and 2.222 days tenor for resident individuals, non-
individuals, and NRI customers.
 The deposit has been introduced with the aim of mobilising funds to support
environment-friendly initiatives and projects, thereby fostering the growth of a
green finance ecosystem in India.
 Currently, the scheme is available through the branch network, and it will soon be
made available through other digital channels such as YONO and Internet Banking
Services (INB), it added.
 As RBI’s “Framework for acceptance of Green Deposits”, Banks are not
permittedto offer a differential rate of interest on green deposits. However, they
are allowed to offer overdraft facilities to customers against Green Deposits.
 News Link - SBI launches Green Rupee Term Deposit - The Hindu BusinessLine

96. Consider the following statement regarding Green deposits. Which of the
following statement is incorrect?
1. Green deposits are interest-bearing deposits.
2. They are received by the regulated entities for a fixed period.
3. The deposits raised under RBI’s Framework for acceptance of Green Deposits are covered by the Deposit Insurance and Credit Guarantee
Corporation.
4. As per the current framework, green deposits can be denominated in Indian Rupees and foreign currency.
5. All of the above are correct

 Green Rupee Term Deposit of 1,111 days, 1,777 days, and 2.222 days tenor have been launched by State Bank of India (SBI).

Click Here For Ultimate Bundle PDF Course 2022| Click Here to Subscribe Our Yearly Mock Test Package Follow Us- Telegram. Facebook , Twitter ,Instagram
Weekly Optimum Current Q & A PDF – Based On Recent Exam Trends Second Week of January (9th January –16th January 2024)

 These have been launched for resident individuals, non-individuals, and NRI customers.
 RBI’s “Framework for acceptance of Green Deposits” prohibits banks from offering a differential interest rate on green deposits.
 There is no restriction on premature withdrawal of green deposits, however, the REs will adhere to the extant guidelines.
 However, it allows them to offer overdraft facilities to customers against Green Deposits.
 Green deposits are interest-bearing deposits. They are received by the regulated entities for a fixed period.
 The proceeds of green deposits are earmarked for being allocated towards green finance according to RBI’s notification dated 11 April
2023.
 The deposits raised under RBI’s Framework for acceptance of Green Deposits are covered by the Deposit Insurance and Credit Guarantee
Corporation in line with the Deposit Insurance and Credit Guarantee Corporation Act of 1961.
 As per the current framework, green deposits can be denominated in Indian Rupees only. They cannot be denominated in foreign currency.
 News Link - SBI launches green rupee fixed deposit (FD): Who can invest, interest rate, how to buy - The Economic Times (indiatimes.com)

97. In January as per report New Development Bank (NDB) has signed an agreement for providing a ________million loan to Gujarat
government for the State’s Mukhayamantri Gram Sadak Yojana.
A. $200 million
B. $250 million
C. $400 million
D. $500 million
E. $100 million

 Shanghai-headquartered New Development Bank (NDB) signed an agreement for providing a $500-million loan to Gujarat government.
 As part of Vibrant Gujarat Summit, a project loan will be provided for the State’s Mukhayamantri Gram Sadak Yojana.
 NDB will support the roads and buildings department of the State government for construction of an estimated 1,200 km of roads.
 NDB will also provide knowledge support to strengthen the rural road network in the State by adopting new technology, safeguarding the
environment and implementing safer road designs.
 The New Development Bank, formerly referred to as the BRICS Development Bank, is a multilateral development bank established by the
BRICS states. President: Dilma Rousseff.
 News Link - Shanghai’s NDB to provide $500 million loan to Gujarat - The Hindu BusinessLine

98. Recently as per report Central Board of Indirect Taxes & Customs (CBIC) roll out a new mobile application named _________ designed to
alert officials promptly about the status and timely disposition of tax appeals by tracking cases throughout the litigation process.
A. AIS for Taxpayer
B. Samay
C. Pramarsh
D. Kanun
E. JustIs

 The Central Board of Indirect Taxes & Customs (CBIC) will roll out an app to alert
officials about timely disposition of tax appeals by tracking all cases under the
litigation process.
 Named 'Samay', the app will capture all the pending orders across levels from
adjudicating officer to the Customs, Excise and Service Tax Appellate Tribunal
(Cestat) to High Courts, and reflect the pendency of review orders that are
either under process at various levels or awaiting processing at the
commissionerate level.
 The initiative will greatly reduce the litigation time.

Click Here For Ultimate Bundle PDF Course 2022| Click Here to Subscribe Our Yearly Mock Test Package Follow Us- Telegram. Facebook , Twitter ,Instagram
Weekly Optimum Current Q & A PDF – Based On Recent Exam Trends Second Week of January (9th January –16th January 2024)

 The Central Board of Indirect Taxes & Customs (CBIC) is the authority responsible for administering indirect taxes such as GST, central
excise, service tax, customs, among others in India.
 Shri Sanjay Kumar Agarwal is Chairman, Central Board of Indirect Taxes & Customs and Special Secretary to the Government of India.
 News Link - Central Board of Indirect Taxes & Customs app to cut litigation time - The Economic Times (indiatimes.com)

99. In January as per report ______________has launched Samman RuPay Credit Card for government employees in collaboration with the
National Payments Corporation of India (NPCI).
A. IndusInd Bank
B. Indian Bank
C. Bandhan Bank
D. Canara Bank
E. Axis Bank

 Samman RuPay Credit Card has been launched by IndusInd Bank for government employees.
 It is a UPI-enabled credit card that has been introduced by IndusInd Bank in collaboration with the National Payments Corporation of India
(NPCI).
 The card is especially tailored for government sector employees. It offers different kind of perks to them.
 The card comes with benefits such as complimentary movie tickets, cashback on various expenses, and zero fees on cash advances.
 The card also offers exemptions from surcharges on IRCTC transactions and fuel purchases.
 IndusInd Bank is a Pune-based bank. It was started in 1994.
 Sumant Kathpalia is its CEO. Its tagline is we make you feel richer.
 News Link - RuPay credit card on UPI for Govt employees; all you need to know - IndusInd Bank Samman RuPay credit card launched | The
Economic Times (indiatimes.com)

100.According to S&P Global Market Intelligence data & analysis, HDFC Bank is __________ largest bank in Asia Pacific by market
capitalization, while SBI stands at ___________.
A. 4th , 12th
B. 7th , 15th
C. 3rd , 11th
D. 2nd , 7th
E. 5th ,9th

 Indian banks racked up gains in their


market capitalisation for the quarter
ended December 31, 2023, with HDFC Bank
leading the gains, according to S&P Global
Market Intelligence data & analysis.
 HDFC Bank's market cap increased 12.1%
to $155.91 billion, cementing its position as
India's largest bank, following its merger
of parent Housing Development Finance
Corp Ltd in July 2023.
 State Bank of India, the nation's top bank
by assets, rose 7.3% to $68.84 billion, while
ICICI Bank Ltd. gained 4.9% to $83.98
billion.

Click Here For Ultimate Bundle PDF Course 2022| Click Here to Subscribe Our Yearly Mock Test Package Follow Us- Telegram. Facebook , Twitter ,Instagram
Weekly Optimum Current Q & A PDF – Based On Recent Exam Trends Second Week of January (9th January –16th January 2024)

 Industrial and Commercial Bank of China Ltd., China's largest bank by assets, also remained the largest bank by market cap in the region
in the fourth quarter. The bank's market cap rose 4.2% to $224.19 billion.
 Agricultural Bank of China Ltd. retained its spot as the second-largest bank in the region, with the bank's market cap rising 3.7% to
$175.72 billion.
 China Merchants Bank Co. Ltd. had the largest decline in market capitalization for the quarter as its market cap fell 16.2% to $96.92 billion,
slipping to 8th place in the list.
 The government forecasts a 7.3% expansion for the fiscal year ending March.
 India's economy remains one of the world's fastest growing, according to the UN World Economic Situation and Prospects 2024 report
released on January 4. Domestic demand and investment are projected to push 2024 growth to 6.2%.
 News Link - HDFC Bank third largest bank in Asia Pacific by market cap, SBI at 11th position: S&P, ET BFSI (indiatimes.com) Indian banks
improve market cap in Q4 as investors chase world's fastest growth | S&P Global Market Intelligence (spglobal.com)

101.In January as per report ____________has extended a INR 1 billion (c. USD 12.1 million) loan to Everest Fleet, an independent fleet
management provider, under its partnership with GuarantCo, as part of the Private Infrastructure Development Group (PIDG) to facilitate
the purchase of electric vehicles (EV) by Everest Fleet for use as green taxis in India.
A. Bandhan Bank
B. Indian Bank
C. Canara Bank
D. HDFC Bank
E. Axis Bank

 Axis Bank has extended a INR 1 billion (c. USD 12.1 million) loan to Everest Fleet, an independent fleet management provider, under its
partnership with GuarantCo, part of the Private Infrastructure Development Group (PIDG).
 The loan, provided for over four years, aims to facilitate the purchase of electric vehicles (EV) by Everest Fleet for use as green taxis in
India.
 This financial collaboration is a result of the USD 200 million INR equivalent framework guarantee agreement signed by GuarantCo and
Axis Bank in May 2022.
 The agreement allows the mobilization of funds between USD 300 and 400 million in INR equivalent to finance the e-mobility ecosystem in
India.
 The transaction is expected to benefit at least 1,000 drivers, aligning with Sustainable Development Goals (SDGs) 11.2, 11.6, and 13, which
focus on providing access to sustainable transport systems, reducing environmental impact, and combating climate change.
 News Link - Axis Bank and GuarantCo facilitate INR 1 billion loan to Everest Fleet for electric taxis, ET BFSI (indiatimes.com)

102.In January as per report ____________has launched an ‘One co-branded credit card’ with FPL Technologies to offer a range of features
tailored to the diverse needs of digitally savvy Indian consumers.
A. Bandhan Bank
B. Canara Bank
C. Axis Bank
D. Indian Bank
E. HDFC Bank

 Indian Bank has launched an ‘Indian Bank One co-branded credit card’ with FPL Technologies.
 Leveraging the Visa and RuPay platforms, these cards will be issued by Indian Bank and managed
by FPL Technologies.
 The Indian Bank One co-branded credit card boasts of international validity with a 1% forex fee
and aims to offer a range of features tailored to the diverse needs of digitally savvy Indian
consumers.

Click Here For Ultimate Bundle PDF Course 2022| Click Here to Subscribe Our Yearly Mock Test Package Follow Us- Telegram. Facebook , Twitter ,Instagram
Weekly Optimum Current Q & A PDF – Based On Recent Exam Trends Second Week of January (9th January –16th January 2024)

 Enabled by the OneCard mobile app, the card provides users complete digital control over their credit card experience - from usage and
rewards redemption to bill payment.
 Shanti Lal Jain, MD & CEO of Indian Bank.
 News Link - Indian Bank launches a co-branded credit card - The Hindu BusinessLine

103.Recently Net direct tax collections have touched about 81% of the Budget Estimate during April 1-January 10 period of the current fiscal.
What is the direct collection target for the current fiscal year?
A. ₹16.23-lakh crore
B. ₹17.23-lakh crore
C. ₹18.23-lakh crore
D. ₹19.23-lakh crore
E. ₹20.23-lakh crore

 Net direct tax collections have touched about 81% of the


Budget Estimate during April 1-January 10 period of the
current fiscal.
 At this trend, tax collection is expected to be more than the
Budget target by ₹80,000-90,000 crore.
 Direct tax collection target for current fiscal is ₹18.23-lakh
crore.
 Corporate income tax (CIT) and personal income tax (PIT)
are included in Direct Taxes.
 According to Central Board of Direct Taxes (CBDT), direct
tax collection, net of refunds, stands at ₹14.70-lakh crore.
 This collection is 19.4% more than the net collections for
the corresponding period of last year.
 A direct tax is a tax that a person or organization pays
directly to the entity that imposed it. Examples include
income tax, real property tax, personal property tax, and
taxes on assets, all of which are paid by an individual
taxpayer directly to the government.
 News Link - At ₹14.70-lakh crore, direct tax collections reach
81% of Budget Estimates for FY24 - The Hindu BusinessLine

104.In January as per report Asia's richest billionaire Mukesh Ambani joined the exclusive club of tycoons having net worth of at least $100
billion.
In the global rich list, Ambani is now at the __________ spot.
A. 9th
B. 11th
C. 15th
D. 7th
E. 13th

 Asia's richest billionaire Mukesh Ambani joined the exclusive


club of tycoons having net worth of at least $100 billion after
shares of Reliance Industries (RIL) ended at fresh all-time high
levels.

Click Here For Ultimate Bundle PDF Course 2022| Click Here to Subscribe Our Yearly Mock Test Package Follow Us- Telegram. Facebook , Twitter ,Instagram
Weekly Optimum Current Q & A PDF – Based On Recent Exam Trends Second Week of January (9th January –16th January 2024)

 The total net worth of the Chairman and Managing Director of RIL was estimated at $105.1 billion, shows data from the Forbes Real-Time
Billionaires List.
 In the global rich list, Ambani is now at the No.11 spot. The Forbes list has only 12 billionaires in the $100 billion club.
 Ambani had earlier entered into the $100 billion elite club in 2021 to drop out of the list later on. Gautam Adani, in the meantime, is at
No.16 on the list of the world's richest billionaires.
 Elon Musk remains on top of the list with a net worth of #240.9 billion, followed by Bernard Arnault, Jeff Bezos, Larry Ellison, and Mark
Zuckerberg.
 News Link - Mukesh Ambani joins $100-billion net worth club as RIL breaks record - The Economic Times (indiatimes.com)

105.In January as per report ___________has become the world’s biggest exporter of liquefied natural gas for the first time, with 2023
shipments overtaking leading suppliers Australia and Qatar.
A. Qatar
B. US
C. UAE
D. India
E. Saudi Arabia

 The US has become the world’s biggest exporter of liquefied natural gas for the first time,
with 2023 shipments overtaking leading suppliers Australia and Qatar.
 The US exported 91.2 million metric tons of LNG in 2023, a record for the country.
 Qatar, the top LNG supplier in 2022, saw its volumes shrink, with a 1.9 per cent decline
dropping the nation into third spot for shipments of the super-chilled fuel.
 Australia ranked second, with exports that were little changed from 2022.
 News Link - US Becomes World’s Top LNG Supplier After Exports Eclipse Australia and Qatar
- Bloomberg

106.In January as per report Securities and Exchange Board of India said that investors across
all categories will be allowed for short-selling, but __________will not be permitted.
A. Naked short-selling
B. Caked short-selling
C. Term short-selling
D. Fixed short-selling
E. Long short-selling

 The Securities and Exchange Board of India said that investors across all categories will
be allowed for short-selling, but naked short-selling will not be permitted.
 Further, all stocks that trade in the futures and options segment are eligible for short-
selling.
 Naked short-selling shall not be permitted in the Indian securities market and
accordingly, all investors would be required to mandatorily honor their obligation of
delivering the securities at the time of settlement.
 “Short selling” means selling a stock that the seller does not own at the time of trade.
 Under the norms, institutional investors are required to disclose upfront at the time of
placement of order whether the transaction is a short sale.
 News Link - sebi mandate: SEBI allows short-selling by all investors; F&O stocks also
eligible - The Economic Times (indiatimes.com)

Click Here For Ultimate Bundle PDF Course 2022| Click Here to Subscribe Our Yearly Mock Test Package Follow Us- Telegram. Facebook , Twitter ,Instagram
Weekly Optimum Current Q & A PDF – Based On Recent Exam Trends Second Week of January (9th January –16th January 2024)

107.In January as per report Adidas announced setting up its first and only global capacity centre (GCC) in Asia outside China in _________.
A. Bhopal
B. Kochi
C. Bengaluru
D. Kolkata
E. Chennai

 Adidas is setting up its first and only global capacity centre (GCC) in Asia outside China in
Chennai, to focus on 'key processes'.
 The athletic footwear and apparel giant’s move reinforces the recent boom of
multinationals investing in India in large numbers to set up their GCCs to harness the
abundant software skills in the country.
 The German corporation headquartered in Herzogenaurach, Bavaria, will set up its Global
Business Services (GBS) hub with Akhil Kapoor leading operations.
 Adidas has a corporate office in Gurgaon, but the Chennai facility will be their first GCC
outside of China in Asia.
 News Link - Adidas: Adidas to set up its first Asia GCC outside China in Tamil Nadu - The
Economic Times (indiatimes.com)

108.In January as per report ___________ announced to build the world's single largest steel
manufacturing site at Gujarat's Hazira by 2029.
A. Reliance Industries
B. Tata Group
C. ArcelorMittal
D. Welspun
E. JSW Steel

 ArcelorMittal will build the world's single


biggest steel manufacturing factory at
Hazira in Gujarat by 2029.
 The factory will have a capacity of 24
million tonne per annum.
 ArcelorMittal has signed MoUs with the
Gujarat government for phase two of the
Hazira plant in the summit.
 Prime Minister Narendra Modi had done the 'bhumi puja' for the first phase of the plant in 2021 and the construction is going on as per
schedule for commissioning in 2026.
 News Link - ArcelorMittal to build world's single largest steel site at Hazira by 2029 (business-standard.com)

109.In January as per report ___________ partnered with Hindustan Zinc to foster economic growth and bring financial inclusion in the lives of
rural community beneficiaries under CSR program.
A. Jio Payments Bank
B. Aitel Payments Bank
C. India Post Payments Bank
D. Fino Payments Bank
E. NSDL Payments Bank

Click Here For Ultimate Bundle PDF Course 2022| Click Here to Subscribe Our Yearly Mock Test Package Follow Us- Telegram. Facebook , Twitter ,Instagram
Weekly Optimum Current Q & A PDF – Based On Recent Exam Trends Second Week of January (9th January –16th January 2024)

 In a ground breaking alliance set to transform the financial landscape of rural Rajasthan, India Post Payments Bank (IPPB) & Hindustan
Zinc Limited (HZL – a subsidiary of Vedanta Group) join forces to pioneer inclusive financial solutions.
 The collaboration will ensure in creating a lasting impact on the lives of community beneficiaries around Hindustan Zinc’s operational
areas.
 With this collaboration IPPB, a public limited company under the Department of Posts, reinforce its commitment to reach out to the last
mile with best-in-class digital banking services.
 India Post Payments Bank (IPPB) has been established under the Department of Posts, Ministry of Communication with 100% equity
owned by Government of India. The IPPB was launched on September 1, 2018.
 News Link - pib.gov.in/PressReleaseIframePage.aspx?PRID=1995250

110.Recently retail inflation reached a four-month high of _________ per cent in December.
A. 6.24 %
B. 5.29 %
C. 5.69 %
D. 2.61 %
E. 3.45 %

 Retail inflation reached a four-month high of 5.69 per cent in


December.
 The Index of Industrial Production (IIP) slipped to an eight-month
low of 2.4 per cent in November as against 11.6 per cent a month
ago.
 In December, nine of the 22 major states/UTs registered an
inflation rate above the headline inflation rate of 5.69 per cent.
 The highest inflation was registered in Gujarat (7.07 per cent)
followed by Rajasthan (6.95 per cent) and Haryana (6.72 per cent).
 Crucially, the inflation rate has stayed within the RBI's tolerance
band of 2-6 per cent.
 CPI is the measuring index of the changes in the price level of
consumer goods and services bought out by households.
 The National Statistical Office (NSO) under Ministry of Statistics
and Programme Implementation (MoSPI) releases Consumer Price
Index (CPI) in India with the base year 2012.
 News Link - India's retail inflation at four-month-high of 5.69 per
cent in December, ETCFO (indiatimes.com)

111.Recently wholesale price index (WPI)-based inflation increases from 0.26% in November
to __________% in December.
A. 1.29%
B. 0.73%
C. 2.43%
D. 1.56%
E. 0.29%

 The wholesale price index (WPI)-based inflation increases from 0.26% in November to
0.73% in December.
 This is the highest wholesale inflation registered in the past nine months.

Click Here For Ultimate Bundle PDF Course 2022| Click Here to Subscribe Our Yearly Mock Test Package Follow Us- Telegram. Facebook , Twitter ,Instagram
Weekly Optimum Current Q & A PDF – Based On Recent Exam Trends Second Week of January (9th January –16th January 2024)

 The WPI inflation in December has increased mainly due to higher prices of food articles, machinery & equipment, other manufacturing,
other transport equipment, and computer, electronics & optical products, among others.
 The WPI inflation was in the negative zone from April to October.
 Wholesale Price Index, also known as WPI, measures the changes in wholesale prices in India. The Wholesale Price Index tracks the price
change in commodities at the main stages before it reaches the retailers.
 The WPI is released every 14th of the month.
 The Ministry of Commerce and Industry uses the wholesale price index to calculate the inflation rates in India.
 From 2004-05 to 2011-12, the wholesale price index base year was updated in May 2017. Several key features have been added to WPI with
the base 2011-12-
 There are now 697 items in the basket which have been raised from 676.
 News Link - Top news of the day: Wholesale inflation rises to 0.73% in December; human vaccine trials for deadly Nipah virus launched, and
more - The Hindu

112.In January as per report Group (Term) Insurance Scheme for Casual Paid Labourers engaged by BRO has been approved by Defense
Minister Rajnath Singh.
Under this scheme, an insurance amount worth ___________lakh will be provided to the families/close relatives of Casual Paid Workers
(CPL) in case of any death.
A. Rs 20 lakh
B. Rs 30 lakh
C. Rs 10 lakh
D. Rs 05 lakh
E. Rs 25 lakh

 Group (Term) Insurance Scheme for


Casual Paid Labourers engaged by
BRO has been approved by Defense
Minister Rajnath Singh.
 The proposal to launch a Group (Term) Insurance Scheme for Casual Paid Labourers (CPL) engaged by Border Roads Organisation/General
Reserve Engineer Force for various ongoing project works has been approved.
 Under this scheme, an insurance amount worth Rs 10 lakh will be provided to the families/close relatives of Casual Paid Workers (CPL) in
case of any death.
 The scheme is being implemented keeping in mind hazardous workplaces, inclement weather, inaccessible areas, and occupational health
hazards.
 This scheme will serve as a social security and welfare measure for CPLs working in interior and remote areas of the country.
 In addition, the Defense Minister had recently approved several welfare measures for the betterment of CPLs.
 These include: Preservation and transportation of mortal remains and eligibility for transportation allowance of assistant (attendant).
 Funeral assistance increased from Rs 1,000 to Rs 10,000.
 Advance payment of ex-gratia amount of Rs 50,000 as immediate assistance in case of death etc.
 News Link - Raksha Mantri approves proposal to launch Group Insurance Scheme for Casual Labourers engaged by Border Roads Organisation
(newsonair.gov.in)

113.In January as per report United States International Development Finance Corporation (USIDFC) and India’s National Infrastructure
Investment Fund (NIIF) are “working on the possibility of creating a fund” that would invest in renewable energy and green projects in
India.
The idea is broadly to create a fund with the USIDFC and NIIF bringing in _________ million each and getting more from other investors.
A. $500 million
B. $200 million
C. $300 million

Click Here For Ultimate Bundle PDF Course 2022| Click Here to Subscribe Our Yearly Mock Test Package Follow Us- Telegram. Facebook , Twitter ,Instagram
Weekly Optimum Current Q & A PDF – Based On Recent Exam Trends Second Week of January (9th January –16th January 2024)

D. $400 million
E. $250 million

 The United States International Development


Finance Corporation (USIDFC) and India’s National
Infrastructure Investment Fund (NIIF) are “working
on the possibility of creating a fund” that would
invest in renewable energy and green projects in
India.
 The idea is broadly to create a fund with the USIDFC
and NIIF bringing in $500 million each and getting
more from other investors.
 NIIF is a “collaborative investment platform” for
international and Indian investors, anchored by the
Government of India, which manages funds with
investments in different asset classes and diversified
sectors that generate attractive risk-adjusted
returns.
 NIIF manages over $44.9 billion of equity capital
commitments across its four funds – Master Fund,
Private Markets Fund, Strategic Opportunities Fund,
and India-Japan Fund.

 USIDFC has an exposure of $4 billion in India, which is 10 per cent of its global exposure.
 NIIF is India’s first sovereign wealth fund set up by the Government of India in 2015.
 It is an investor-owned fund manager, anchored by the Government of India (GoI) in collaboration with leading global and domestic
institutional investors.
 NIIF’s mandate includes investing in areas such as energy, transportation, housing, water,
waste management and other infrastructure-related sectors in India.
 It is being operationalized by establishing three Alternative Investment Funds (AIFs) under
the SEBI Regulations.
 The proposed corpus of NIIF is Rs. 40,000 Crores funded of 49% from Government of India.
 Rest from strategic anchor partners (Overseas sovereign/quasi-
sovereign/multilateral/bilateral investors).
 News Link - USIDFC, NIIF to set up $1 -billion fund to finance renewable energy projects - The
Hindu BusinessLine

114.Recently as per report Simmtech will be setting up a unit to support semiconductor


ecosystem at Sanand near Ahmedabad at a cost of ₹1,250 crore.
Simmtech is company of which of the following country?
A. Vietnam
B. Japan
C. Indonesia
D. South Korea
E. UAE

 South Korean firm Simmtech will be setting up a unit to support semiconductor ecosystem at
Sanand near Ahmedabad at a cost of ₹1,250 crore.
 South Korean firm Simmtech Co Ltd has signed an MoU with the Gujarat government to set up a plant near the Micron plant at Sanand.
Click Here For Ultimate Bundle PDF Course 2022| Click Here to Subscribe Our Yearly Mock Test Package Follow Us- Telegram. Facebook , Twitter ,Instagram
Weekly Optimum Current Q & A PDF – Based On Recent Exam Trends Second Week of January (9th January –16th January 2024)

 The company is the world’s largest manufacturer of semiconductor substrate.


 Simmtech has been allotted 30 acres at Sanand.
 This is the second semiconductor plant to be announced in Gujarat.
 The first semiconductor unit is being set up by Micron Technology. As per schedule, the first chip will be produced from the Sanand factory in
December 2024.
 Semiconductor fab unit proposed to be set up at Dholera by the Tata Group will be much bigger in scale than the Micron plant.
 Namtech will train technicians for the semiconductor industry in Gujarat, while IIT Gandhinagar will be made the centre of excellence for
semiconductor research and development.
 News Link - South Korean Simmtech to set up ₹1,250 crore unit for semiconductor ecosystem - The Hindu BusinessLine

115.Which of the following company is setting a 40 MW solar power plant is being established on 165 acres of land in Ayodhya?
A. SJVN
B. NTPC
C. Larsen & Turbo
D. REC Ltd
E. PFC Ltd

 Ayodhya, which is scheduled to witness the consecration


ceremony of the Ram Janmabhoomi temple, is set to emerge
as Uttar Pradesh’s maiden solar city with a 40 megawatt
(MW) solar plant likely becoming operational in the temple
town by March 2024.
 As part of the solar city programme, a 40 MW solar power
plant is being established on 165 acres of land in Ayodhya by
state-run power generator NTPC.
 The land has been identified, and the project is underway. Part
of the project (10 MW capacity) has already commenced and
the remaining is expected to be operational by March 2024.
 On January 22, Prime Minister Narendra Modi and around
11,000 guests are set to witness the ‘Pran Pratishtha’
ceremony of the temple.
 Spearheaded by the Uttar Pradesh New and Renewable Energy Department (UPNEDA), the ambitious project encompasses the
development of solar parks, solar-powered boats, street lights, and widespread adoption of solar energy in public transport.
 As a part of the broader framework outlined in Uttar Pradesh’s Solar Energy Policy 2022, Ayodhya is set to pioneer solar innovation,
serving as a model for 17 municipal corporations, including Noida.
 Under the Babu Ji Kalyan Singh Gram Unnati Yojna for solar street lights, the target of 200 lights has been achieved, further enhancing
rural infrastructure and ensuring well-lit public spaces.
 News Link - Spirituality with Sustainability: Ayodhya’s 40 MW solar power plant to be operational by March - The Hindu BusinessLine

116.Which of the following country is among the top 3 FPIs investing country into India?
1. Luxembourg
2. France
3. USA
4. Mauritius
5. Singapore

Select the correct source code:


A. Only 1,2 and 5

Click Here For Ultimate Bundle PDF Course 2022| Click Here to Subscribe Our Yearly Mock Test Package Follow Us- Telegram. Facebook , Twitter ,Instagram
Weekly Optimum Current Q & A PDF – Based On Recent Exam Trends Second Week of January (9th January –16th January 2024)

B. Only 2,3 and 4


C. Only 2 and 5
D. Only 1,3 and 5
E. All of the above

 There has been a change in the pecking order among regions that foreign portfolio investors (FPIs)
invest from into India.
 Luxembourg has dislodged Mauritius to become the region with the third-largest assets under
custody.
 Its AUC grew 30 per cent last year to ₹4.85 lakh crore, with its equity assets now second only to the
US.
 Investments from Mauritius dipped nine per cent to ₹3.9 trillion during the year amid greater
regulatory oversight on the island nation.
 The tax treaty between India and Mauritius was renegotiated a few years back post which capital
gains on sale of shares was made fully taxable after April 1, 2019.
 Ireland and Norway have both moved up one position, now ranking fifth and seventh, respectively,
among the jurisdictions.
 FPI refers to investments made by foreign individuals, corporations, and institutions in the financial
assets of India, such as stocks, bonds, and mutual funds.
 These investments are mainly for the purpose of short-term gains and portfolio diversification, unlike
Foreign Direct Investment (FDI) which involves long-term ownership of assets.
 News Link - Luxembourg, France gain heft among FPIs investing into India - The Hindu BusinessLine

117.According to the latest survey by consulting and audit firm PwC, India has been placed at __________
most attractive place for investment.
A. 9th
B. 4th
C. 5th
D. 3rd
E. 8th

 India was seen to be the


fifth preferred destination
to invest after the US,
China, Germany and the
UK, moving up four places
from last year, a survey by
consulting and audit firm
PwC.

 As per the study, 70% of CEOs in India express high confidence in their company's potential for revenue growth in the coming three years.
 The fourth most-favoured destination in 2020, India slipped to the fifth position a year later, the eighth spot in 2022 and ninth in 2023.
 News Link - India economic growth: 86% of CEOs believe India's economic growth will improve over the next 12 months: PwC’s Annual Global
CEO Survey - The Economic Times (indiatimes.com)

Click Here For Ultimate Bundle PDF Course 2022| Click Here to Subscribe Our Yearly Mock Test Package Follow Us- Telegram. Facebook , Twitter ,Instagram
Weekly Optimum Current Q & A PDF – Based On Recent Exam Trends Second Week of January (9th January –16th January 2024)

118.As per the CREDAI CBRE report titled ‘MSMEs: Unleashing the Engines of Economic Prosperity’, which of the following are top 3 state in
contribution to India’s MSME sector?
1. Maharashtra
2. Uttar Pradesh
3. Tamil Nadu
4. Karnataka
5. Odisha

Select the correct source code:


A. Only 1,2 and 5
B. Only 2,3 and 4
C. Only 1,2 and 3
D. Only 3,4 and 5
E. Only 1,4 and 5

 As per the CBRE report, UP is among the top three states in contribution to India’s MSME sector.
 CBRE South Asia Private Limited released a report titled ‘MSMEs: Unleashing the Engines of Economic Prosperity’.
 MSME activities are mainly concentrated in Maharashtra, Tamil Nadu, and Uttar Pradesh.
 Three states account for around 40% of the total MSMEs registered in India.
 Uttar Pradesh has secured a significant position among the top three States, holding a 9 per cent share in the national MSME landscape.
 This report was released at the two-day event at Deen Dayal Upadhyaya Trade Facilitation Centre in Varanasi.
 Women entrepreneurs and MSME industrial park developers have supported the MSME ecosystem in Uttar Pradesh.
 Agra, Kanpur, Varanasi, Lucknow, Meerut and Ghaziabad emerged as MSME clusters in Uttar Pradesh.
 The report highlighted the key role played by the Udyam scheme.
 News Link - Top 3 States contribute 40% of registered MSMEs in India: report - The Hindu BusinessLine

119.Recently as per report Indian Railway Freight Activity (IRFA) Index has been introduced by __________.
1. Dun & Bradstreet
2. Alstom Ltd.
3. Jupiter Wagons Limited
4. Siemens Global
5. Integral Coach Factory

Select the correct source code:


A. Only 2,3 and 5
B. Only 2,4 and 5
C. Only 1 and 3
D. Only 1,3 and 5
E. All of the above

 Indian Railway Freight Activity (IRFA) Index has been introduced by Dun & Bradstreet and Jupiter Wagons.
 IRFA Index is aimed at providing a data-driven assessment of the dynamics of Indian Railway freight activity.
 It combines factual data obtained from the Ministry of Railways with survey responses from railway freight users.
 IRFA Index consists of two vital sub-indices-the activity sub-index and the experience sub-index.
 The activity sub-index is based on data released by the Indian Railways.
 The experience sub-index is based on a survey of railway freight users for the current quarter.
 The survey also assesses the level of optimism of railway freight users for the forthcoming quarter.
 The index combines quantitative data with qualitative feedback from users. Its value ranges from 0-100.

Click Here For Ultimate Bundle PDF Course 2022| Click Here to Subscribe Our Yearly Mock Test Package Follow Us- Telegram. Facebook , Twitter ,Instagram
Weekly Optimum Current Q & A PDF – Based On Recent Exam Trends Second Week of January (9th January –16th January 2024)

 A value of 50 is the neutral level of railway freight activity on a year-on-year basis.


 At 48, the Indian Railway Freight Activity (IRFA) index for Q3 2023 indicates a sub-neutral level of rail freight activity compared with the
same quarter last year.
 Dun & Bradstreet is a global leader in business decisioning data and analytics.
 Jupiter Wagons Limited is an Indian private manufacturer of railway freight wagons. It is headquartered in Kolkata, West Bengal.
 News Link - Dun & Bradstreet and Jupiter Wagons Unveil the Indian Railway Freight Activity (IRFA) Index (business-standard.com)

120.In January as per report India Post Payments Bank (IPPB) announced the attainment of a significant milestone, with eight crore customers
now benefiting from its innovative and inclusive financial services.
Who is the current MD & CEO (Interim) of IPPB?
A. J. Venkatramu
B. Suresh Sethi
C. Easwaran Venkateswaran
D. Vineet Pandey
E. Ananta Narayan Nanda

 India Post Payments Bank (IPPB) proudly announces the attainment of a significant milestone, with eight crore customers now benefiting
from its innovative and inclusive financial services.
 Since its inception, the IPPB has been dedicated to providing accessible and affordable banking solutions to every corner of the nation.
 India Post Payments Bank (IPPB) has been established under the Department of Posts, Ministry of Communication with 100% equity
owned by Government of India. IPPB was launched on September 1, 2018.
 The IPPB’s reach and its operating model is built on the key pillars of India Stack – enabling Paperless, Cashless and Presence-less banking
in a simple and secure manner at the customers’ doorstep.
 IPPB delivers simple and affordable banking solutions through intuitive interfaces available in 13 languages.
 Shri Easwaran Venkateswaran, MD & CEO (Interim) and COO of IPPB.
 News Link - Press Information Bureau (pib.gov.in)

121.On 13 January, India's first National Highway Steel Slag Road section on NH- 66 Mumbai-Goa National Highway was inaugurated by Dr.
V.K. Saraswat, Member (S&T), NITI AAYOG.
____________under CSIR-CRRI technical guidance, has constructed a 1 km long four-lane steel slag road section on the Indapur-Panvel
section of NH-66 Mumbai-Goa.
A. Jindal Steel
B. Tata Steel
C. JSW Steel
D. Godawari Power and Ispat
E. Steel Authority of India

 India's first National Highway Steel Slag Road section inaugurated on NH-66 Mumbai-Goa National Highway.
 On 13 January, India's first National Highway Steel Slag Road section on NH- 66 Mumbai-Goa National Highway was inaugurated by Dr.
V.K. Saraswat, Member (S&T), NITI AAYOG.
 Steel slag road technology developed by the Council of Scientific and Industrial Research (CSIR)-Central Road Research Institute (CRRI)
(CSIR-CRRI) is converting waste from steel industries into wealth.
 It is assisting the National Highways Authority of India (NHAI) in building sustainable and environment-friendly highways in the country.
 JSW Steel, under CSIR-CRRI technical guidance, has constructed a 1 km long four-lane steel slag road section on the Indapur-Panvel section
of NH-66 Mumbai-Goa.
 For the construction of this road, approximately 80,000 tonnes of CONARC steel slag was converted into steel slag aggregates processed at
JSW Steel Dolvi, Raigarh plant.

Click Here For Ultimate Bundle PDF Course 2022| Click Here to Subscribe Our Yearly Mock Test Package Follow Us- Telegram. Facebook , Twitter ,Instagram
Weekly Optimum Current Q & A PDF – Based On Recent Exam Trends Second Week of January (9th January –16th January 2024)

 Processed steel slag aggregates are superior to natural aggregates in various mechanical properties. Steel slag is used in place of natural
aggregates in all layers of road construction.
 The bituminous steel slag road on NH-66 has been constructed with 28% less thickness than the bituminous road on NH-66 however, the
cement concrete section has been constructed with an identical thickness.
 Both bituminous and cement concrete road sections are about 32% more profitable than conventional roads and have greater durability.
 News Link - pib.gov.in/PressReleaseIframePage.aspx?PRID=1995906

122.In January as per report UPI collaborated with Pay Now of ___________to facilitate direct fund transfers.
A. Singapore
B. UK
C. Usa
D. UAE
E. Nepal

 The cross-border linkage between India’s Unified Payments Interface (UPI) and Singapore’s PayNow is live on BHIM (Bharat Interface for
Money), PhonePe and Paytm.
 This feature helps the Indian diaspora in Singapore to send funds into their Indian bank accounts directly via UPI.
 Axis Bank, DBS Bank India, ICICI Bank, Indian Bank, Indian Overseas Bank and State Bank of India are offering this service through their
respective banking applications too.
 This interoperability between the two countries is the result of the collaboration between the Reserve Bank of India (RBI) and the
Monetary Authority of Singapore (MAS).
 This facility provides an instantaneous, secured and cost-effective fund transfer mechanism between the two countries. The transfer rails
almost replicate the payment experience on UPI for domestic transactions.
 News Link - PhonePe, Paytm, SBI and others go live on India-Singapore cross-border UPI-based payments, ET BFSI (indiatimes.com)

Appointment News

123.In January as per report Gabriel Attal becomes the youngest Prime Minister of France.
Gabriel Attal is presently working as an __________.
A. Finance Minister
B. Education Minister
C. Defence Minister
D. Road & Highways Minister
E. Foreign Minister

 Gabriel Attal has been named the next Prime Minister of France.
 Gabriel Attal is presently working as an education minister.
 At the age of 34, Mr Attal is the youngest Prime Minister in modern French history.
 Socialist Laurent Fabius was 37 years old when he was appointed by François
Mitterrand in 1984.
 Mr Attal replaces Élisabeth Borne. Élisabeth Borne resigned after 20 months in office.
 Mr Attal has become France’s first openly gay head of government.
 The new head of government will be the fourth prime minister since 2017 under Macron.
 News Link - Gabriel Attal becomes France’s youngest Prime Minister (newsonair.gov.in)

 After recent political turmoil over a new immigration law, French Prime Minister Elisabeth Borne has decided to resign from his post.
 She is the second woman to hold the position of Prime Minister of France after Édith Cresson.

Click Here For Ultimate Bundle PDF Course 2022| Click Here to Subscribe Our Yearly Mock Test Package Follow Us- Telegram. Facebook , Twitter ,Instagram
Weekly Optimum Current Q & A PDF – Based On Recent Exam Trends Second Week of January (9th January –16th January 2024)

 After Castex's resignation, she was appointed as Prime Minister on 16 May 2022.
 She was then minister of labour, employment and integration in the Castex government from 2020 to 2022.
 She is a member of President Emmanuel Macron's party Renaissance.
 News Link - French PM Borne resigns following political turmoil over immigration - The Hindu

124.In January as per report government has extended RBI Deputy Governor ___________ term by a year till 15th
January 2025.
A. MD Patra
B. Rajeshwar Rao
C. T. Rabi Sankar
D. Swaminathan J
E. M.K. Jain

 RBI Deputy Governor MD Patra’s term has been extended by the government by a year.
 Government has for the second time extended his tenure by a year.
 The Appointments Committee of the Cabinet (ACC) has approved his re-appointment for one year with effect from January 15, 2024 or
until further orders, whichever is earlier.
 He was appointed RBI Deputy Governor for three years in January 2020.
 Before his elevation to the post of RBI Deputy Governor, he was Executive Director at RBI.
 Patra is the only member of the MPC who has been on the rate-setting panel since its inception in 2016.
 News Link - Reserve Bank of India - Press Releases (rbi.org.in) RBI deputy governor Michael Patra gets
another 1-year extension (business-standard.com)

125.Which of the following party has won parliamentary elections in Bhutan?


A. Bhutan Democratic Party
B. People Congress Party
C. People Democratic Party
D. Bhutan Tendrel Party
E. Bhutan Rising Party

 People’s Democratic Party has won parliamentary elections in Bhutan. People’s Democratic Party (PDP)
is the party of former Prime Minister Tshering Tobgay.
 It has won most seats in the parliamentary elections of Bhutan. It will form the new government.
 PDP had won 30 of the 47 National Assembly seats. Bhutan Tendrel Party had won 17 seats.
 Tobgay is expected to become Prime Minister for the second time.
 Nearly half a million voters selected the members of parliament from 94 candidates presented by the Bhutan Tendrel Party and the PDP.
 Bhutan Capital - Thimphu, Currency - Ngultrum.
 News Link - Former Prime Minister Tshering Tobgay’s People’s Democratic Party wins parliamentary elections in Bhutan (newsonair.gov.in)

126.Recently seen in news Tshering Tobgay became which country's Prime Minister for the second time?
A. Nepal
B. Bhutan
C. Myanmar
D. Indonesia
E. Tibet

Click Here For Ultimate Bundle PDF Course 2022| Click Here to Subscribe Our Yearly Mock Test Package Follow Us- Telegram. Facebook , Twitter ,Instagram
Weekly Optimum Current Q & A PDF – Based On Recent Exam Trends Second Week of January (9th January –16th January 2024)

 Bhutanese voters have elected Tshering Tobgay to become Prime Minister for the
second time.
 Tshering Tobgay party has won nearly two-thirds of the seats in the election.
 Mr. Tobgay was also the leader of the Opposition in Bhutan's first parliament
when it was established in 2008.
 He heads the liberal People's Democratic Party (PDP). He served as Prime Minister
of Bhutan from 2013 to 2018.
 News Link - Bhutan's Tshering Tobgay to become PM for second time - The Hindu

127.Which of the following parties has won Parliamentary elections for 5th term in
Bangladesh?
A. Awami League
B. Jatiya Party
C. Bangladesh Nationalist Party
D. Bangladesh Democratic Party
E. None of these

 In Bangladesh, the Awami League has won Parliamentary elections for 5th term.
 Awami League is Bangladesh Prime Minister Sheikh Hasina's party. It secured a
majority in the national elections.
 The Awami League won 165 seats. Nine candidates of the Jatiya Party won and 49
independent candidates won.
 Awami League President Sheikh Hasina will form the government for the fourth
continuous term.
 The voting was held on 07 January in 299 out of 300 constituencies.
 As per Election Commission (EC) statistics, overall voter turnout in the parliamentary
elections is estimated to be about 40%.
 Bangladesh Nationalist Party (BNP) and its allies had boycotted the polls.
 News Link - Bangladesh: Sheikh Hasina-led Awami League secures majority in national
elections (newsonair.gov.in)

128.Recently Sheikh Hasina took oath as PM of Bangladesh for the fifth term overall.
She will serve as the _________ prime minister of Bangladesh.
A. 15th
B. 12th
C. 17th
D. 10th
E. 14th

 Sheikh Hasina took oath as PM of Bangladesh for the fifth term overall.
 Awami League President Sheikh Hasina took oath as PM for the fourth consecutive term.
 She will serve as the 12th prime minister of Bangladesh.
 President Mohammed Shahabuddin administered the oath of office and secrecy to her and new cabinet ministers.
 He appointed 25 ministers and 11 state ministers as new cabinet ministers.
 Hasina was elected the president of the Awami League at the age of 34.
 The Awami League won the June 1996 parliamentary elections and came to power for the
first time since the killing of Bangabandhu in 1975.

Click Here For Ultimate Bundle PDF Course 2022| Click Here to Subscribe Our Yearly Mock Test Package Follow Us- Telegram. Facebook , Twitter ,Instagram
Weekly Optimum Current Q & A PDF – Based On Recent Exam Trends Second Week of January (9th January –16th January 2024)

 In 2001 Hasina became the first leader to step down peacefully after serving a full five-year term in office and handed over power to a
caretaker administration as per the constitution
 Her party won elections in 2008, 2014, 2018 and the latest on January 7, 2024.
 News Link - Sheikh Hasina takes oath as Prime Minister of Bangladesh for the fifth time (newsonair.gov.in)

129.In January as per report William Lai, also known as Lai Ching-te, won the presidential election in Taiwan.
What is the Capital of Taiwan?
A. Taichung
B. Kaohsiung
C. Tainan
D. Taipei
E. Hualien

 William Lai, also known as Lai Ching-te, won the


presidential election in Taiwan.
 Lai Ching-te is from the ruling Democratic Progressive
Party (DPP). He was serving as the vice president of
Taiwan.
 He competed against two other candidates, including
Hou Yu-ih from the conservative Kuomintang (KMT) and
former Taipei Mayor Ko Wen-je from Taiwan People’s
Party.
 William Lai was first elected as a lawmaker in 1998 then
became the mayor of Tainan City in 2010.
 Capital: Taipei City, Currency: New Taiwan dollar.
 News Link - Who is William Lai, the new president of Taiwan? | Explained News - The Indian Express

130.In January as per report Frederik X becomes King of Denmark.


What is the capital of Denmark?
A. Zurich
B. Madrid
C. Copenhagen
D. Berlin
E. Reykjavik

 Denmark’s King Frederik X ascended the


throne, succeeding his mother, Queen
Margrethe II, who formally abdicated after 52
years as monarch, with big crowds gathered in
the capital to witness history.
 Margrethe, 83, stunned the nation on New
Year’s Eve when she announced she planned to
become the first Danish monarch in nearly 900
years to voluntarily relinquish the throne.

 Australian-born Mary and wife of Frederik X is the first commoner to become Queen of Denmark.
 Capital: Copenhagen, The krone is the official currency of Denmark, Greenland.

Click Here For Ultimate Bundle PDF Course 2022| Click Here to Subscribe Our Yearly Mock Test Package Follow Us- Telegram. Facebook , Twitter ,Instagram
Weekly Optimum Current Q & A PDF – Based On Recent Exam Trends Second Week of January (9th January –16th January 2024)

 News Link - Crown Prince Frederik X becomes King of Denmark (newsonair.gov.in)

Days and Obituary News

131.In January as per report greatest football player of all time Franz Beckenbaue known as “the Kaiser” meaning “the Emperor” passed away
belongs to which of the following country?
A. Brazil
B. France
C. Italy
D. Germany
E. Argentina

 Franz Beckenbauer was the greatest


football player of all time. He is known as
“the Kaiser” meaning “the Emperor” in
Germany.
 He played a critical role in some of
Germany’s greatest sporting
achievements.
 He was part of the team that had won the 1972 European championship and then the World Cup on home soil.
 During the mid-1970s, Franz Beckenbauer's Bayern Munich team was the best club side in the world.
 He was twice named European Footballer of the Year.
 In 2016 he was fined by FIFA’s ethics committee for failing to co-operate with an inquiry into corruption over the awarding of the 2018 and
2022 World Cups.
 He last appeared at Bayern Munich’s Allianz Arena in August 2022.
 News Link - Franz Beckenbauer football player passed away at 78 in Germany (newsonair.gov.in)

132.Pravasi Bharatiya Divas (PBD) is celebrated every year on _________to mark the contribution of the Overseas Indian community to the
development of India.
A. 7th January
B. 9th January
C. 8th January
D. 10th January
E. 6th January

 Pravasi Bharatiya Divas (PBD) is celebrated every year on 9th January.


 This day marks the contribution of the Overseas Indian community to the development of India.
 The day marks the return of Mahatma Gandhi to India South Africa in 1915.
 The inaugural Pravasi Bharatiya Divas was observed in 2003.
 From 2003 till 2015, the PBD Convention was held every year. After 2015, its format was changed to once in two years.
 The Pravasi Bharatiya Divas is the flagship event of the Ministry of External Affairs.To date, 17 conventions have been held. The last
Pravasi Bharatiya Divas was celebrated in the Indore of Madhya Pradesh in 2023.
 News Link - Why Pravasi Bharatiya Divas is celebrated? Its history, theme and significance | Mint (livemint.com)

133.Recently India Meteorological Department (IMD) celebrated 150th Year of its establishment and service to the nation on 15th January,
2024.

Click Here For Ultimate Bundle PDF Course 2022| Click Here to Subscribe Our Yearly Mock Test Package Follow Us- Telegram. Facebook , Twitter ,Instagram
Weekly Optimum Current Q & A PDF – Based On Recent Exam Trends Second Week of January (9th January –16th January 2024)

Who is the current chairman of IMD?


A. Sourabh Mishra
B. Sanjay Aggarwal
C. Mrutyunjay Mohapatra
D. Rohit Biswal
E. Anish Singh Dayala

 The Vice President of India, Shri Jagdeep Dhankhar, attended the inaugural ceremony of the 150th year of its establishment as the chief
guest.
 President appreciated IMD for its pivotal role in increasing the farmer’s income through the appropriate forecast of daily weather,
hazards, and crop weather advisories.
 This forecast is issued by IMD in collaboration with the Ministry of Agriculture, state & central Agricultural Universities, and the state
Agricultural departments.
 The number of Doppler radars in the country increased from 15 in 2014 to 39 in 2023, and an additional 25 radars will be added in the next
2-3 years.
 Rainfall monitoring stations increased from 3,955 in 2014 to 6,095 in 2023, upper air stations increased from 43 in 2014 to 56 in 2023, and
high wind speed recorders increased from 19 in 2014 to 35 in 2023.
 On the occasion of IMD's 150th year celebrations, the Vice President released and launched Panchayat Mausam Seva for farmers, IMD’s
Mobile App and Mausamgram, National Framework of Climate Services (NFCS).
 The India Meteorological Department has developed the in-house Web-GIS-based integrated Decision Support System (DSS) known as the
Weather Analysis and Forecast Enabling System (WAFES), inspired by Pancha Mahabhuta i.e. Water, Air, Fire, Earth and Sky.
 It provides real-time information for various sectors such as Urban, Power, Hydrology, Health, Energy, Agriculture, Transport and Tourism
under the "UPHHEATT" initiative (for the cause of welfare).
 India Meteorological Department, Ministry of Earth Sciences, Ministry of Panchayati Raj and Green Alert Mausam Sewa have jointly
developed Panchayat Mausam Sewa Portal.
 IMD launched an integrated GIS based interactive mobile app MAUSAM for all weather-related services like current weather, forecast of
every hour to 7 days.
 The app supports 12 Indian languages for varied users in India. Upholding the vision "Har Har Mausam, Har Ghar Mausam," IMD
introduces "mausamgram," available through the Mobile App “Mausam”
 India Meteorological Department (IMD) was established in 1875 as one of the first scientific departments of the Government of India and
the principal government agency in all matters relating to meteorology and allied subjects.
 News Link - pib.gov.in/PressReleaseIframePage.aspx?PRID=1996364

134.National Road Safety Week is observed from ___________ to create awareness about road safety.
A. 9th to 15th January
B. 11th to 17th January
C. 10th to 17th January
D. 8th to 15th January
E. 12th to 18th January

 National Road Safety Week is observed from 11th to 17th January to create awareness about road safety.
 It aims to educate people about the perils of irresponsible driving.
 In its annual report for 2022-23, the Ministry of Road Transport & Highways published Road Accident statistics.
 In the report, the total number of accidents in 2021 was 4,12,432.
 The National Safety Council (NSC) has appealed to its members to observe the 35th National Road Safety Month-2024 in January 2024.
 According to PIB, the National Road Safety Week was first observed in 1989.

Click Here For Ultimate Bundle PDF Course 2022| Click Here to Subscribe Our Yearly Mock Test Package Follow Us- Telegram. Facebook , Twitter ,Instagram
Weekly Optimum Current Q & A PDF – Based On Recent Exam Trends Second Week of January (9th January –16th January 2024)

135.Every year on ___________World Hindi Day is celebrated to mark the anniversary of the first World Hindi Conference organized in Nagpur
on 10 January 1975.
A. 8th January
B. 10th January
C. 7th January
D. 6th January
E. 4th January

 Every year on 10 January, World Hindi Day is celebrated.


 It is celebrated to mark the anniversary of the first World Hindi Conference organized in Nagpur on 10 January 1975.
 The theme of World Hindi Day 2024 theme is “Hindi – Bridging Traditional Knowledge and Artificial Intelligence.”
 In 2006, the first World Hindi Day was celebrated by the then Prime Minister Dr. Manmohan Singh.
 Its main aim is to promote the Hindi language across the world.
 In India, Hindi Diwas is observed every year on 14th September.

136.Every year on _________National Youth Day is celebrated in India, which is also known as Rashtriya Yuva Diwas to commemarate the
birth anniversary of India's greatest philosopher and spiritual leader, Swami Vivekananda.
A. 10th January
B. 12th January
C. 15th January
D. 09th January
E. 07th January

 Every year on 12th January, National Youth Day is celebrated in India, which is also known as Rashtriya Yuva Diwas.
 The birth anniversary of India's greatest philosopher and spiritual leader, Swami Vivekananda is celebrated every year as National Youth
Day since 1985.
 In 1984, the government announced 12th January as National Youth Day.
 Swami Vivekananda founded the Ramakrishna Mission in 1897.
 PM Modi inaugurated the 27th National Youth Festival in Nashik, Maharashtra on January 12.
 The theme of the National Youth Festival 2024 is “MYBharat-ViksitBharat@2047- By the Youth, For the Youth ''.
 This year, National Youth Day was celebrated by all the field organizations of the Department of Youth Affairs in the districts in
collaboration with several government departments.
 News Link - pib.gov.in/PressReleaseIframePage.aspx?PRID=1995339#:~:text=12th January is celebrated as,in the City of Kumbh.

137.Every year on ___________ National Start-up Day is celebrated to promote start-up culture at the grass-root level.
A. 15th January
B. 16th January
C. 18th January
D. 13th January
E. 24th January

 Every year on 16 January, National Start-up Day is celebrated to promote start-up culture at the grass-root level.
 In 2022, Prime Minister Narendra Modi had announced celebrating January 16 as National Start-up Day.
 Several organizations organize this event with the aim to foster the spirit of entrepreneurship and innovation by engaging the start-up
community.
 The first National Start-up Day was celebrated in 2022.

Click Here For Ultimate Bundle PDF Course 2022| Click Here to Subscribe Our Yearly Mock Test Package Follow Us- Telegram. Facebook , Twitter ,Instagram
Weekly Optimum Current Q & A PDF – Based On Recent Exam Trends Second Week of January (9th January –16th January 2024)

 Launched in 2016, the Startup India initiative has seen tremendous growth, with the number of startups increasing from around 340 in the
year of its launch to over 1,15,000 in 2023.
 2024 is the eighth anniversary of the Startup India program.
 According to data from the InvestIndia website, India recorded more than 112,718 DPIIT-recognized startups spread across 763 districts by
October 2023.
 News Link - pib.gov.in/PressReleaseIframePage.aspx?PRID=1996759

138.Recently as per report DPIIT is organizing the Startup India Innovation Week 2024 from 10 January 2024. In which year was Startup India
initiative launched?
A. 2014
B. 2016
C. 2018
D. 2019
E. 2020

 DPIIT is organising it to bring together India’s startups, entrepreneurs, investors, policymakers, and other stakeholders.
 Startup India Innovation Week 2024 is being organised from 10th January 2024 to 18th January 2024.
 The result declaration ceremony for the States’ Startup Ranking and National Startup Awards 2023 will be held on 16th January 2024.
 PM Modi launched the Startup India initiative on 16th January 2016.
 It was launched to build a strong ecosystem for nurturing innovation.
 According to the Commerce and Industry Ministry, the number of recognized startups has grown to more than one lakh 17 thousand from
about 400 startups in 2016.
 The Department for Promotion of Industry and Internal Trade (DPIIT) is also partnering with the Department of Youth Affairs to promote
the Mera Yuva Bharat (MY Bharat) initiative.
 Prime Minister Modi launched Mera Yuva Bharat (MY Bharat) initiative on 31st October 2023.
 News Link - Press Information Bureau (pib.gov.in)

139.National Human Trafficking Awareness Day is observed every year on _________ to raise awareness about the condition of human
trafficking victims, as well as to promote and protect their rights.
A. 10th January
B. 11th January
C. 12th January
D. 08th January
E. 06th January

 On January 11 every year, National Human Trafficking Awareness Day is observed.


 The day aims to raise awareness about the condition of human trafficking victims, as well as to promote and protect their rights.
 In 2007, January 11 was proclaimed National Human Trafficking Awareness Day by the United States Senate.

140.Every year on _________ Indian Army Day is celebrated to remember the sacrifice of our soldiers to protect India and its citizens.
A. 12th January
B. 13th January
C. 14th January
D. 16th January
E. 15th January

Click Here For Ultimate Bundle PDF Course 2022| Click Here to Subscribe Our Yearly Mock Test Package Follow Us- Telegram. Facebook , Twitter ,Instagram
Weekly Optimum Current Q & A PDF – Based On Recent Exam Trends Second Week of January (9th January –16th January 2024)

 Every year on 15 January,


Indian Army Day is
celebrated to remember
the sacrifice of our soldiers
to protect India and its
citizens.
 The day is also celebrated
to commemorate the
moment when Lieutenant
General KM Cariappa took
over from Francis Butcher
as the first Commander-in-
Chief of the Indian Army
on 15 January 1949.
 Francis Butcher was the
last British Commander-in-
Chief of India.
 This year, the 76th Army Day celebration is being organized in Lucknow.
 The event is being held outside the national capital for the second time as part of the Government of India's initiative to take major
movements from the capital to different parts of the country.
 The best marching contingent for the 76th Army Day Parade to be held on January 15, 2024, will be selected using Artificial Intelligence.
 For the first time, Artificial Intelligence is being used to identify the best marching drill and identify the best contingent in the parade.

141.Recently as per report _____________will observe 2024 as the “Year of Technology Absorption” as it’s trying to gradually mature into a
modern force that will have a new operational philosophy.
A. Indian Army
B. Indian Navy
C. Indian Air Force
D. Railway Protection Force
E. ISRO

 Indian Army will observe 2024 as the “Year of Technology Absorption” as it’s trying to gradually mature into a modern force that will have
a new operational philosophy for induction of drones and counter-drones systems right across infantry, artillery and armoured battalions,
and establish Command Cyber Operations Support Wings (CCOSWs), besides bridging other conventional asymmetries.
 The Army’s new pitch is an attempt to grow out of previous declaration of making 2023 as the “Year of Transformation” that saw moves
such as reorientation of its arms like that of artillery, induction of fresh human resource through Agniveers, downsizing of some verticals
and tech infusion.
 The Army has already inducted 2500 Secure Army Mobile Bharat Version (SAMBHAV) handsets which has multi-layered inscription and are
5G compliant. Developed in-house in close collaboration with National Centres of Excellence and industry, the Army’s requirement is about
35000 SAMBHAV handsets which would be distributed to officials handling sensitive assignments.
 Army Chief General Manoj Pande.
 News Link - 2024 is ‘Year of Technology Absorption’ for Army - The Hindu BusinessLine

142.What is the name of the renowned classical singer from Kirana Gharana, who was awarded with Padma Vibhushan in 2022, who passed
away in January, 2024?
A. Prabha Atre
B. Rashmi Shukla
C. Rashid Khan
D. Jaya Singh
Click Here For Ultimate Bundle PDF Course 2022| Click Here to Subscribe Our Yearly Mock Test Package Follow Us- Telegram. Facebook , Twitter ,Instagram
Weekly Optimum Current Q & A PDF – Based On Recent Exam Trends Second Week of January (9th January –16th January 2024)

E. Lakshmi Sharma

 Renowned classical singer Prabha Atre passed


away at the age of 92.
 Legendary classical singer and Padma
Vibhushan died at the age of 92.
 She was one of the prominent dancers of the
Kirana gharana.
 She played a significant role in popularising
classical music across the globe.
 She received the Padma Shri in 1990, the
Padma Bhushan in 2002 and the Padma
Vibhushan in 2022.
 She was born in Pune in September 1932 and received her training in classical music from the renowned Sureshbabu Mane.
 News Link - Renowned classical singer Prabha Atre passes away at 92 - The Hindu

143.In January as per report renowned Hindustani classical singer- Ustad Rashid Khan passed away.
Ustad Rashid Khan was from which of the following state?
A. Madhya Pradesh
B. Odisha
C. Tamil Nadu
D. Uttar Pradesh
E. Bihar

 Ustad Rashid Khan dies at the age of 55 in


Kolkata. The music maestro passed away
after a prolonged battle with cancer.
 He was born in Uttar Pradesh’s Badayun. He
was from the Rampur-Sahaswan Gharana.
He is the great-grandson of gharana founder
Inayat Hussain Khan.
 He was awarded the Padma Shri and Sangeet Natak Akademi Award in 2006. He was awarded Padma Bhushan in 2022.
 News Link - Music maestro Rashid Khan passes away after prolonged battle with cancer | Latest News India - Hindustan Times

Sports Award Books News

144.In January as per report Argentine superstar Lionel Messi won the Best FIFA Men's Player 2023 for the ___________ times, while Aitana
Bonamati of _________ was awarded the Best FIFA Women's Player award.
A. 3rd, Spain
B. 4th, Germany
C. 5th, Brazil
D. 7t , France
E. 8th, Spain

 Argentine superstar Lionel Messi won the Best FIFA Men's Player 2023.
 The announcement was made at the Best FIFA Football Awards ceremony in London on 16
January.
Click Here For Ultimate Bundle PDF Course 2022| Click Here to Subscribe Our Yearly Mock Test Package Follow Us- Telegram. Facebook , Twitter ,Instagram
Weekly Optimum Current Q & A PDF – Based On Recent Exam Trends Second Week of January (9th January –16th January 2024)

 He retained the title he won in 2022.


Messi also won the FIFA Best Men's
Player of 2019 award.
 This is the third time in the last 4
years that Messi has received this
award.
 He had previously been recognized
by FIFA as the leading player in the
men's game on five occasions – in
2009, 2010, 2011, 2012, and 2015–
making it his eighth individual
award overall.
 Messi topped the voting, leaving
behind other finalists Erling Haaland
and Kylian Mbappe.
 Spain and Barcelona striker Aitana
Bonamati was awarded the Best
FIFA Women's Player award.
 Manchester City's treble-winning
coach Pep Guardiola won the Best
Men's Manager of 2023 award while
England coach Sarina Wiegman
claimed the women's best coach
award for a record-extending fourth
time.

 Manchester City stopper Ederson won the Best Men's Goalkeeper award while Mary Earps, the Manchester United and England No. 1, won
the Women's award in the ceremony in London.
 The Brazil men’s national team have been recognised with the FIFA Fair Play award for their powerful stand against racism.
 Brazilian midfielder Guilherme Madruga has won the FIFA Puskás Award 2023 for his astonishing overhead kick.
 News Link - fifa.com/fifaplus/en/the-best-fifa-football-awards/articles/lionel-messi-wins-best-fifa-mens-player-award-2023

145.Consider the following match the column in respect with National Sports and Adventure Awards 2023. Which of the following is matched
incorrectly?
1. Aditi Gopichand Swami - Archer
2. Mohammed Shami - Cricketer
3. Diksha Dagar - Golfer
4. Pawan Kumar - Archer
5. Divyakriti Singh – Equestrian

 President Droupadi Murmu presented the National


Sports and Adventure Awards 2023 at a function in
Rashtrapati Bhavan.
 Cricketer Mohammed Shami, Chess player R Vaishali,
Archers Ojas Pravin Deotale and Aditi Gopichand
Swami, Golfer Diksha Dagar and Kabaddi players
Pawan Kumar and Ritu Negi received the Arjuna
Award.

Click Here For Ultimate Bundle PDF Course 2022| Click Here to Subscribe Our Yearly Mock Test Package Follow Us- Telegram. Facebook , Twitter ,Instagram
Weekly Optimum Current Q & A PDF – Based On Recent Exam Trends Second Week of January (9th January –16th January 2024)

 Besides, Hockey players Krishan Bahadur Pathak and P Sushila Chanu, Kho-Kho player Nasreen, Lawn Bowls player Pinki, Shooters
Aishwary Pratap Singh and Esha Singh, Squash player Harinder Pal Singh Sandhu, Table Tennis player Ayhika Mukherjee, Wrestler Sunil
Kumar and Antim, Wushu player N Roshibina Devi, Blind Cricketer Illuri Ajay Kumar Reddy, Para Archer Sheetal Devi and Para Canoeing
player Prachi Yadav were among those received the Arjuna Awards.
 Aditi Swami and Ojas Deotale, and Sheetal Devi, the armless 16-year-old who won the Asian Para Games, were presented with Arjuna
Awards at a ceremony in Delhi on 9 January 2024.
 Chirag Shetty and Satwiksairaj Rankireddy: Honored with the Major Dhyan Chand Khel Ratna award for their exceptional performance in
badminton during 2023.
 Dhyan Chand Awards for Lifetime Achievement in Sports and Games were given to Manjusha Kanwar for Badminton, Vineet Kumar
Sharma for Hockey and Kavitha Selvaraj for Kabaddi.
 During the function, Jain Deemed to be University and Bengaluru and Odisha Mining Corporation Limited got Rashtriya Khel Protsahan
Puruskar.
 Major Dhyan Chand Khel Ratna Award: It is considered India’s highest sporting honour; the Khel Ratna was established in 1991-92. The
award is given for outstanding performances in sports spanning over a period of four years.
 Arjuna Award: It is awarded for consistent good performance over a period of four years. The winners of the Arjuna Award receive a
statuette of Arjuna, a certificate and a cash prize.
 Rashtriya Khel Protsahan Puraskar: Awarded to organisations or corporates (both private and public) and individuals for playing a role in
the area of sports promotion and development over the last three years.
 Maulana Abul Kalam Azad Trophy (MAKA) 2023: Guru Nanak Dev University, Amritsar (overall winner university); Lovely Professional
University, Punjab (first runner -up); Kurukshetra University, Kurukshetra (second runner-up).
 News Link - President Droupadi Murmu presents National Sports and Adventure Awards 2023 at Rashtrapati Bhavan (newsonair.gov.in)
pib.gov.in/PressReleaseIframePage.aspx?PRID=1988607

146.In January as per report Board of Control for Cricket in India (BCCI) announced Campa and Atomberg Technologies as the official partners
for the BCCI Domestic and International Season __________.
A. 2024-2026
B. 2024-2027
C. 2024-2025
D. 2024-2028
E. 2024-2030

 The Board of Control for Cricket in India (BCCI) announced Campa and Atomberg
Technologies as the official partners for the BCCI Domestic and International Season 2024-
2026.
 Beverages brand Campa has signed a multi-year deal with the cricket governing body, a
development that blocks out rivals Coca-Cola and PepsiCo for BCCI matches played in India.
 Notably, this also marks Campa's first big cricket sponsorship since the brand was
relaunched by Reliance over a year back, and comes ahead of the summer season.
 Meanwhile, consumer appliances brand Atomberg, have recently ventured into smart locks
and other home appliances and looks to scale up brand presence.
 Reliance Retail had acquired Campa in mid-2022 from Pure Drinks group for an estimated
Rs 22 crore.
 News Link - BCCI official sponsors: Reliance's Campa, Atomberg Technologies are now BCCI's
official partners for 2024-26 cricket season - The Economic Times (indiatimes.com)

147.Consider the following match the column in regards with Golden Globe Awards 2024.
Which of the following is matched incorrectly?
1. Best Director - Christopher Nolan (Oppenheimer)
2. Best Film Actor - Cillian Murphy (Oppenheimer)
Click Here For Ultimate Bundle PDF Course 2022| Click Here to Subscribe Our Yearly Mock Test Package Follow Us- Telegram. Facebook , Twitter ,Instagram
Weekly Optimum Current Q & A PDF – Based On Recent Exam Trends Second Week of January (9th January –16th January 2024)

3. Best Performance by a Female Actor – Emma Stone (Poor Things)


4. Best Film – Non-English Language - Anatomy of a Fall, France (Neon)
5. Best Original Song - “What Was I Made For?” Barbie, Music and lyrics by Billie Eilish, Finneas

 The Golden Globes is an annual award ceremony honouring outstanding


entertainment industry achievements.
 It is one of the most prestigious events in Hollywood, bringing together the top
celebrities and talents from film and television.
 This year marked the 81st edition of the Golden Globes. Comedian and actor Jo
Koy hosted this year’s award ceremony, becoming the second Asian host in the
awards’ history.
 This is the first Golden Globes ceremony since the Hollywood Foreign Press
Association (HFPA) disbanded in 2023.
 Oppenheimer won 5 awards in the Golden Globe Awards 2024.
 It includes Best Film (Drama), Best Director (Christopher Nolan), Best Film
Actor (Drama) for Cillian Murphy, and Robert Downey Jr took Best Supporting
Actor (Film).
 Oppenheimer also won in the Best Original Score category.
 The 81st Golden Globe Awards were given in Beverly Hills, California, USA on
January 8.
 In the realm of television, Succession took the spotlight, garnering the most
wins in the TV categories following its acclaimed fourth and final season.
 Meanwhile, Barbie made history by winning the inaugural box office
achievement award, a testament to its global earnings of $1.4 billion (£1.13
billion).
 Lily Gladstone, the star of Killers of the Flower Moon, made history by
becoming the first indigenous person to win the best drama actress award.
 Lily Gladstone, hailing from the Blackfeet Indian Reservation in northern
Montana, delivered part of her acceptance speech in the Algonquian
language Blackfoot
 Emma Stone was named best actress in a musical or comedy for her role in
Poor Things, which also received the award for best musical or comedy film.
 News Link - Oppenheimer | Golden Globes 2024 : Golden Globes 2024: How
Oppenheimer and Succession stole the show (indiatimes.com)

148.Recently India's first multi-sports Beach Games, “The Beach Games 2024”
was held at the Blue Flag-certified Ghoghla Beach in Diu.
___________ became the overall champion in the first Beach Games held in
Diu.
A. Uttar Pradesh
B. Odisha
C. Haryana
D. Madhya Pradesh
E. Punjab

 Madhya Pradesh became the overall champion in the first Beach Games held in
Diu.

Click Here For Ultimate Bundle PDF Course 2022| Click Here to Subscribe Our Yearly Mock Test Package Follow Us- Telegram. Facebook , Twitter ,Instagram
Weekly Optimum Current Q & A PDF – Based On Recent Exam Trends Second Week of January (9th January –16th January 2024)

 India's first multi-sports Beach Games, “The


Beach Games 2024” was held at the Blue Flag-
certified Ghoghla Beach in Diu.
 Madhya Pradesh topped the medal tally with a
total of 18 medals, including 7 gold.
 Maharashtra won 14 medals, including 3 gold,
while Tamil Nadu, Uttarakhand, and hosts
Dadra, Nagar Haveli, Diu, and Daman won 12
medals each.
 Assam won 8 medals, of which 5 were gold
medals.
 In a thrilling final, Lakshadweep won the gold
medal in beach soccer, a historic achievement
for this ancient island region. They defeated
Maharashtra 5-4 in a tightly contested final.
 This excellence of sports remained at its peak from 4 to 11 January.
 During this period, 1404 athletes under the age of 21 years from 28 states and union territories participated in a variety of sports with the
support of 205 match officials.
 News Link - First ever Beach Games held in Diu; Madhya Pradesh emerged as champion (newsonair.gov.in)

149.In January as per report ___________won United Cup title after thrilling final tie against Poland.
A. Australia
B. Spain
C. Germany
D. Switzerland
E. USA

 Reigning Olympic champion Alexander Zverev saved two match points in a remarkable comeback
to win his singles tie in an epic three-setter and then helped clinch the United Cup final for Germany
in the deciding mixed doubles match.
 In a thrilling final showdown, Germany emerged victorious over top seeds Poland in the United Cup.
 The United Cup is an international hard court tennis competition featuring mixed-gender teams
from 18 countries.
 The first event was held in December 2022 through January 2023.

 The event is played across multiple Australian cities over 11 days in the leadup to the Australian
Open
 News Llink - Germany wins United Cup title after thrilling final tie against Poland - The Hindu

150.In January as per report __________ has received the award for International Sports Personality of the Year at the Mohammed Bin Rashid
Al Maktoum Creative Sports Awards ceremony in Dubai, United Arab Emirates.
A. Kylian Mbappé
B. Gianni Infantino
C. Lionel Messi
D. Karim Benzama
E. Cristiano Ronaldo

Click Here For Ultimate Bundle PDF Course 2022| Click Here to Subscribe Our Yearly Mock Test Package Follow Us- Telegram. Facebook , Twitter ,Instagram
Weekly Optimum Current Q & A PDF – Based On Recent Exam Trends Second Week of January (9th January –16th January 2024)

 FIFA President Gianni Infantino has received the award for International Sports Personality of the Year at the Mohammed Bin Rashid Al
Maktoum Creative Sports Awards ceremony in Dubai, United Arab Emirates.
 Inspired by Sheikh Mohammed bin Rashid Al Maktoum’s love of sport, the awards were established in 2009, and recognise individual
achievement in a broad range of sport-related fields.
 International recipients have been eligible since 2012, and the International Sports Personality of the Year is the highest international
honour, acknowledging innovators who develop sport and increase fair competition.
 News Link – Gianni Infantino receives International Sports Personality award in Dubai (african.business)

151.In January as per report Shaun Marsh announced his retirement from professional cricket.
He belongs to which of the following country?
A. West Indies
B. England
C. South Africa
D. New Zealnad
E. Australia

 Australian cricketer Shaun Marsh announced his retirement from


professional cricket.
 Marsh also had a great stint with Australia across formats, scoring
more than 5,200 runs and 13 centuries, while his last outing for the
Kangaroos was against Sri Lanka at The Oval during the 2019 ODI
World Cup.
 News Link - Shaun Marsh announces retirement from professional cricket - Sportstar (thehindu.com)

Science Technology Defence And Environment

152.Recently seen in news Thirty Meter Telescope (TMT) project is a joint collaboration involving institutions of which of the following country?
1. Japan
2. China
3. Canada
4. India
5. US

Select the correct source code:


A. Only 1,2 and 5
B. Only 2,4 and 5
C. Only 1,3 and 4
D. Only 1,3,4 and 5
E. All of the above

 Recently, an official delegation from the Department of Science and


Technology visited Mauna Kea to discuss “challenges” to the Thirty
Meter Telescope (TMT) project.

 Thirty Meter Telescope (TMT) is a 30-meter diameter primary mirror optical and infrared telescope being established at Mauna Kea, Hawaii,
USA.

Click Here For Ultimate Bundle PDF Course 2022| Click Here to Subscribe Our Yearly Mock Test Package Follow Us- Telegram. Facebook , Twitter ,Instagram
Weekly Optimum Current Q & A PDF – Based On Recent Exam Trends Second Week of January (9th January –16th January 2024)

 India is a Founder-Member Partner in this project which aims to open new windows to the universe through optical and infrared
astronomy. Indian participation in this project was approved by Union Cabinet in 2014.
 It is proposed as a joint collaboration involving institutions in the US., Japan, China, Canada, and India.
 It will be the world’s most advanced and capable ground-based optical, near-infrared, and mid-infrared observatory.
 At the heart of the telescope is the segmented mirror, made up of 492 individual segments. Precisely aligned, these segments will work as
a single reflective surface of 30m diameter.
 Location: Mauna Kea, an inactive volcano on the island of Hawai’i in the United States.
 India expects to be a major contributor to the project and will provide hardware (segment support assemblies, actuators, edge sensors,
segment polishing, and segment coating), instrumentation (first light instruments), software (observatory software and telescope control
systems) worth $200 million.
 The Indian Institute of Astrophysics (IIAP) is leading the consortium of Indian institutions that are involved with the TMT project.
 India TMT will be jointly funded by the Departments of Science and Technology and Atomic Energy.
 News Link - pib.gov.in/PressReleaseIframePage.aspx?PRID=1995476

153.Consider the following statement regarding India's first indigenously built UAV. Which of the following statement is incorrect?
1. It has been manufactured by Defence Research & Development Organisation.
2. India's first indigenously built UAV has been named 'Drishti 10 Starliner'.
3. The UAV began its journey from Hyderabad to Porbandar as it joined the naval maritime operations.
4. It is based on Elbit's Hermes 900 MALE (Medium Altitude Long Range) platform.
5. All of the above are correct

 On January 10, the Indigenously Drishti 10 Starliner unmanned aerial vehicle (UAV)
manufactured by Adani Defense and Aerospace was included in the Indian Navy.
 The UAV began its journey from Hyderabad to Porbandar as it joined the naval
maritime operations.
 This is a significant opportunity and transformative step in India's pursuit of self-
reliance in ISR technology and maritime dominance.
 The integration of Drishti 10 into naval operations will enhance naval capabilities,
strengthening our readiness for ever-evolving maritime surveillance and
reconnaissance.
 The Drishti 10 Starliner represents a state-of-the-art intelligence, surveillance, and reconnaissance (ISR) platform.

Click Here For Ultimate Bundle PDF Course 2022| Click Here to Subscribe Our Yearly Mock Test Package Follow Us- Telegram. Facebook , Twitter ,Instagram
Weekly Optimum Current Q & A PDF – Based On Recent Exam Trends Second Week of January (9th January –16th January 2024)

 It has an impressive endurance of 36 hours and a payload capacity of 450 kg and has the distinction of being the only all-weather military
platform with STANAG 4671 certification.
 Additionally, it is authorized for flight in both segregated and unsegregated airspace.
 Drishti 10 'Starliner' is India's first indigenously developed unmanned aerial vehicle (UAV), based on Elbit's Hermes 900 MALE (Medium
Altitude Long Range) platform.
 It is equipped with multiple hard points and a 250 kg modular internal installation bay.
 Ashish Rajvanshi, CEO of Adani Defence and Aerospace
 News Link - Adani Group unveils first indigenously manufactured Drishti 10 UAV with 36 hours of endurance | Mint (livemint.com)

154.In January as per report joint exercise named __________ was conducted between the Indian Coast Guard and Japan Coast Guard off the
coast of Chennai, as part of the five-day drill between both nations.
A. Sahyog Kaijin
B. Dharma Guardian
C. Garuda Exercise
D. Surya Kiran
E. JIMEX

 A joint exercise ‘Sahyog Kaijin’ was conducted between the Indian Coast Guard
and Japan Coast Guard off the coast of Chennai, as part of the five-day drill
between both nations.
 The exercise was conducted in line with the Memorandum of Cooperation (MOC)
between the two countries signed in 2006.
 At the MOC, both countries had agreed to promote interaction between the
coast guards, including visits, annual joint exercises, training exchanges,
workshops, and seminars.
 On January 8, the five-day exercise was started, with a focus on pollution
response training on hazardous and intoxicating substances to combat marine
pollution.
 Inspector General Donny Michael, the Commander of the Coast Guard Region
(EAST) reviewed the joint exercise.
 In this exercise, the units of Indian Coast Guard ships Shaunak, Shaurya, Sujay,
Rani Abbakka, Annie Besant, C-440, Sagar Anveshika, Matsya Drishti Japan Coast
Guard ships Yahsima and Samudra Paheredar participated.
 News Link - Indian, Japanese Coast Guards engage in joint exercise off the coast of
Chennai - The Hindu

155.What is the name of the maiden Bilateral Maritime Exercise between the Indian
Navy (IN) and Royal Thai Navy (RTN) that was conducted from 20 to 23
December 2023?
A. Ex-Corpat
B. Ex-Ayutthaya
C. Ex-Kurma
D. Ex-Maitree
E. Ex-Aggati

 The maiden Bilateral Maritime Exercise between the Indian Navy (IN) and Royal Thai Navy (RTN) was conducted from 20 to 23 December
2023.

Click Here For Ultimate Bundle PDF Course 2022| Click Here to Subscribe Our Yearly Mock Test Package Follow Us- Telegram. Facebook , Twitter ,Instagram
Weekly Optimum Current Q & A PDF – Based On Recent Exam Trends Second Week of January (9th January –16th January 2024)

 The Indo-Thai Bilateral Exercise is being named as ‘Ex-Ayutthaya’, which literally translates to ‘The Invincible One’ or ‘Undefeatable’, and
symbolises the significance of two of the oldest cities Ayodhya in India and Ayutthaya in Thailand, the historic legacies, rich cultural ties
and shared historical narratives dating back to several centuries.
 Indigenously built Indian Naval ships Kulish and IN LCU 56 participated in the inaugural edition of the exercise. The RTN side was
represented by His Thai Majesty's Ship (HTMS) Prachuap Khiri Khan.
 The 36th edition of India-Thailand Coordinated Patrol (Indo-Thai CORPAT) was also conducted along with the maiden bilateral exercise.
 Maritime Patrol Aircraft from both navies participated in the Sea Phase of the exercise.
 As part of Government of India's vision of SAGAR (Security And Growth for All in the Region), the Indian Navy has been proactively
engaging with countries in the Indian Ocean Region towards enhancing regional maritime security.
 The conduct of maiden IN-RTN Bilateral Exercise along with Indo-Thai CORPAT is a testimony to the growing bilateral relations between
the two maritime neighbours and has enabled in enhancing interoperability between the two navies.
 News Link - Press Information Bureau (pib.gov.in)

156.Recently as per report under which project Indian military history will be preserved?
A. Shaurya Shakti
B. Parakram Sankalan
C. Parakram Shakti
D. Shaurya Sankalan
E. Shaurya Samvedna

 Under the Project 'Shaurya Sankalan: Digital Archiving of Military History', nearly 700 hours of audio-visual content and 11 lakh pages of
record on the 1948 Jammu and Kashmir conflict, the 1961 Goa Liberation and the India-Pakistan wars of 1965 and 1971 have been
archived.
 This initiative has been started as part of the Army’s effort to preserve the Army's glorious past for posterity.
 700 hours of audio-visual content and 11 lakh pages of record are held with over more than 1300 units, formations, regimental centres and
military museums in the form of battle memoirs.
 The audio-visual films have been cured, digitised and preserved in the 'Digital Archive Kiosk’, which was created at the United Services
Institute of India, New Delhi.
 Another kiosk will be established at the upcoming 'History Cell' of the Army War College.
 In a web page interactive format, this content will remain available to researchers, academicians and scholar warriors.
 News Link - 'Shaurya Sankalan': Army building archives to preserve military history (business-standard.com)

157.In January as per report which of the following Indian Navy warships were decommissioned on January 12, 2024.
1. Cheetah
2. Guldar
3. Annadot
4. Kumbhir
5. Kurma

Select the correct source code:


A. Only 1,3 and 5
B. Only 1,2 and 4
C. Only 1,3,4 and 5
D. Only 2,3 and 4
E. All of the above

Click Here For Ultimate Bundle PDF Course 2022| Click Here to Subscribe Our Yearly Mock Test Package Follow Us- Telegram. Facebook , Twitter ,Instagram
Weekly Optimum Current Q & A PDF – Based On Recent Exam Trends Second Week of January (9th January –16th January 2024)

 After four decades of glorious service to the nation, Indian Navy warships Cheetah, Guldar, and Kumbhir were decommissioned on January
12, 2024.
 The decommissioning ceremony of these ships was conducted in a traditional ceremony at Port Blair, in which the National Flag, Naval
Ensign, and the decommissioning emblem of the three ships were lowered for the last time at sunset.
 INS Cheetah, Guldar, and Kumbhir were built at the Gdynia Shipyard in Poland as Polnokny class ships that could deliver tanks, vehicles,
cargo, and troops directly to the low-lying coastline without the need to dock.
 These warships were commissioned into the Indian Navy in the presence of Shri SK Arora (Cheetah and Guldar) and Shri AK Das (Kumbhir),
the then Ambassador of India to Poland in 1984, 1985, and 1986 respectively.
 Commander VB Mishra, Lieutenant Commander SK Singh, and Lieutenant Commander J Banerjee were deployed as the commanding
officers of the three ships, respectively.
 During its initial years, INS Cheetah was stationed at Kochi and Chennai for some time and INS Kumbhir and Guldar were serving at
Visakhapatnam.
 Later, these ships were deployed in Andaman and Nicobar Command, where they served till their decommissioning.
 These warships were active in Indian Naval service for almost 40 years and collectively covered a distance of approximately 17 lakh
nautical miles while being at sea for more than 12,300 days.
 As the amphibious platform of Andaman and Nicobar Command, these ships have conducted more than 1300 operations off the coast to
disembark Army personnel.
 News Link - pib.gov.in/PressReleaseIframePage.aspx?PRID=1995782

158.What is the name of the carrier rocket launched by china into space from sea off the coast of Haiyang, east China's Shandong Province?
A. Chang'e-1
B. Tianwen-1
C. Gravity-1
D. Gaofen -1
E. BeiDou -1

 China's Gravity-1 carrier rocket made its debut flight from waters off the coast of Haiyang, east China's Shandong Province.
 The rocket blasted off at 1:30 p.m. (Beijing Time), sending three satellites into the planned orbit. The Taiyuan Satellite Launch Center
conducted the offshore mission.
 Developed by the Chinese commercial aerospace company OrienSpace, the rocket is the world's largest solid-propellant launch vehicle and
the country's first commercial rocket equipped with strap-on boosters.
 The 9-meter-high fairing used by Gravity-1 is the largest among China's current commercial rockets.
 News Link - China launches commercial Gravity-1 rocket from sea-Xinhua (news.cn)

159.In January as per report Indian Navy P-8I aircraft recently reached Guam, United States to participate in the __________ edition of the
Anti-Submarine Warfare (ASW) exercise, Ex Sea Dragon 24, organised by the US Navy.
A. 3rd edition
B. 7th edition
C. 9th edition
D. 4th edition
E. 13th edition

 An Indian Navy P-8I aircraft recently reached Guam, United States


to participate in the 4th edition of the Anti-Submarine Warfare
(ASW) exercise, Ex Sea Dragon 24, organised by the US Navy.
 It is a US-led exercise carried out among the navies of the US, Australia, South Korea, Japan, and India.

Click Here For Ultimate Bundle PDF Course 2022| Click Here to Subscribe Our Yearly Mock Test Package Follow Us- Telegram. Facebook , Twitter ,Instagram
Weekly Optimum Current Q & A PDF – Based On Recent Exam Trends Second Week of January (9th January –16th January 2024)

 The exercise will test the capabilities of various aircraft in tracking both simulated and live underwater targets while sharing expertise
among participating nations.
 It involves coordinated multi-lateral Anti-Submarine Warfare (ASW) exercise for Long Range MR ASW aircraft conducted by US Navy.
 The complexity and scope of these exercises has increased steadily over past years to include advanced ASW drills.
 The Exercise is based on countries’ shared values and commitment to an open, and inclusive Indo-Pacific.
 News Link -

160.What is the name of an indigenous assault rifle launched by DRDO for the Indian Army?
A. Prabal
B. Dhruv
C. Agni
D. Vinashak
E. Ugram

 Ugram has been launched by the Armament Research and Development Establishment (ARDE) and a
Hyderabad-based firm.
 Ugram (ferocious) is an indigenous assault rifle. DRDO has launched it for the Indian Army.
 A DRDO lab has for the first time collaborated with private industry to manufacture a 7.62 x 51 mm caliber rifle.
 The rifle is designed and developed to fulfil the needs of the Indian armed forces, paramilitary forces and state police forces.
 The weight of the rifle is less than four kg. It has a firing range of 500 meters.
 Shailendra Gade, the director general of the DRDO’s Armament and Combat Engineering (ACE) system, unveiled the rifle. ARDE developed
the design of the rifle.
 G Ram Chaitanya Reddy, director of Dvipa Armour India, said we have developed five rifles for testing in
the first slot.
 He said we would give 15 more rifles to the ARDE for advanced testing.
 A dedicated barrel manufacturing facility has been established by the ARDE on its campus.
 News Link - DRDO launches indigenous assault rifle 'Ugram' for armed forces - The Economic Times
(indiatimes.com)

161.What is the name of the lander that was launched aboard the United Launch Alliance's Vulcan rocket to
embark on its pioneering journey to explore the Moon?
A. MESSENGER Lander
B. Titan Lander
C. Viking Lander
D. Peregrine Lander
E. Martian Lander

 A spacecraft developed by a private US company and carrying a bunch of scientific instruments from
NASA, took off from Florida, USA.
 The aim of this launch is to become the first US spacecraft to land on the Moon in more than 50 years.
 If successful, this mission would also involve the first landing of a private spacecraft on the Moon.
 No US spacecraft has landed on the Moon since Apollo 17 in December 1972.
 In 2023, a Japanese company had attempted a moon landing with its Hakuto-R mission but was
unsuccessful.
 In 2019, the first Israeli attempt to land on the Moon was also engineered by a private company, but
the spacecraft, Beresheet, had crash-landed.

Click Here For Ultimate Bundle PDF Course 2022| Click Here to Subscribe Our Yearly Mock Test Package Follow Us- Telegram. Facebook , Twitter ,Instagram
Weekly Optimum Current Q & A PDF – Based On Recent Exam Trends Second Week of January (9th January –16th January 2024)

 The Peregrine Lunar Lander was launched aboard the United Launch Alliance's Vulcan rocket to embark on its pioneering journey to explore
the Moon.
 It is designed to carry scientific instruments and other payloads to the Moon's surface, specifically targeting the Sinus Viscositatis region.
 The Mission is part of NASA’s Commercial Lunar Payload Services (CLPS) initiative that seeks to involve private space sector in its ambitious
Artemis programme.
 This launch was the first under the CLPS initiative.
 Under CLPS, so far, at least 14 private companies have been contracted to carry NASA payloads to the Moon.

 This collaboration is aimed at creating the market and technology ecosystem in the private space industry with respect to science and
technology needs of lunar exploration.
 Five payloads from NASA sat inside the Peregrine lander that was launched by the Vulcan heavy rocket.
 The payloads are meant to carry out various exploratory activities, including detection of water.
 The spacecraft will take more than 40 days to reach the Moon, and is planned to make a landing on February 23.
 Peregrine Mission 1 will transport approximately ten payloads, with a total mass capacity of 90 kilograms.
 News Link - First US lunar lander since 1972 launched: The mission, significance | Explained News - The Indian Express NASA Science Heads to
Moon on First US Private Robotic Artemis Flight - NASA

162.In January as per report ____________ has become the first Southeast Asian SaaS customer of IBS Software and the fourth from the Asia-
Pacific region as it installed IBS Software’s iFly Staff travel solution. .
A. Air New Zealand
B. Philippine Airlines
C. Air India Airlines
D. IndiGo Airlines
E. Oman Air

 Philippine Airlines (PAL) and its affiliate carrier, PAL Express (PALex), have installed IBS Software’s iFly Staff travel solution.
 With this, PAL has become the first Southeast Asian SaaS customer of IBS Software and the fourth from the Asia-Pacific region.
 iFly Staff is a SaaS solution designed to automate the end-to-end business and leisure travel booking process for airline employees.
 The tie up with iFly Staff enables PAL and PALex employees, dependents and retirees to plan, book and manage their travel requirements
for leisure and business trips via multiple channels and devices.
 News Link - Philippine Airlines installs IBS Software’s iFly Staff travel solution - The Hindu BusinessLine

163.Consider the following statement regarding MILAN 2024. Which of the following statement is incorrect?
1. 12th edition of the Indian Navy's multinational exercise MILAN will be held from 19 to 27 February.

Click Here For Ultimate Bundle PDF Course 2022| Click Here to Subscribe Our Yearly Mock Test Package Follow Us- Telegram. Facebook , Twitter ,Instagram
Weekly Optimum Current Q & A PDF – Based On Recent Exam Trends Second Week of January (9th January –16th January 2024)

2. MILAN is a biennial multinational naval exercise that began in 1995 with the participation of four foreign countries (Indonesia,
Singapore, Sri Lanka, Thailand).
3. Visakhapatnam has been selected as the preferred venue for future MILAN exercises.
4. Only 1 and 3
5. All of the above are correct

 MILAN 2024 exercise will be conducted by the Indian


Navy at Visakhapatnam.
 The 12th edition of the Indian Navy's multinational
exercise MILAN will be held from 19 to 27 February.
 Earlier, the 11th edition of the exercise was conducted
at Visakhapatnam under the Eastern Naval Command
on February-March 2022.
 MILAN is a biennial multinational naval exercise that
began in 1995 with the participation of four foreign
countries (Indonesia, Singapore, Sri Lanka, Thailand) in
line with India's 'Look East Policy'.
 The MILAN exercise was held under the aegis of Andaman and Nicobar Command till its 10th edition.
 Visakhapatnam has been selected as the preferred venue for future MILAN exercises.
 The MILAN 24 exercise will consist of two phases– the 'Harbour Phase' and 'Sea Phase'.
 The aim of the harbour phase is to promote cultural sharing amongst the participating Nations and the Sea phase includes ships with
Maritime Patrol aircraft and submarines of Friendly Foreign Countries.
 News Link - Indian Navy to Host MILAN 24 Exercise in Visakhapatnam from 19-27 Feb 24 (newsonair.gov.in)

164.Recently as per report India is set to launch the European Space Agency's (ESA) ___________ mission, world's first precision formation
flying mission in September 2024.
A. Proba-3
B. Viper-3
C. Heera-3
D. Martian-3
E. Clifer-3

 India is set to launch the European Space Agency's (ESA) Proba-3 mission in September 2024.Proba-3 is the world's first precision formation
flying mission, employing a pair of satellites operating as a 'large rigid structure' in space to create a solar coronagraph.
 According to ESA, the innovative mission will demonstrate precision formation flying between two satellites to create an artificial eclipse,
revealing new views of the Sun’s faint corona.
 The mission aims to create a 144-meter long solar coronagraph using the aligned satellites, providing scientists with an unprecedented
and continuous view of the Sun's elusive corona.
 Proba-3 mimics an artificial eclipse in space, enabling continuous observation of the corona closer to the solar rim than ever before.
 ESA has developed cutting-edge technologies, including precision cold gas thrusters and vision-based detection systems, crucial for
maintaining millimeter-scale accuracy in positioning the two spacecraft.
 The mission will be launched using the PSLV-XL rocket, a reliable launch vehicle operated by the Indian Space Research Organisation
(ISRO).
 News Link - ISRO to Launch the World’s First Precision Formation Flying Mission – Indian Defence Research Wing (idrw.org) Sun Study: India
to launch Europe’s Proba-3 set to create artificial eclipse | India News - Times of India (indiatimes.com)

Click Here For Ultimate Bundle PDF Course 2022| Click Here to Subscribe Our Yearly Mock Test Package Follow Us- Telegram. Facebook , Twitter ,Instagram
Weekly Optimum Current Q & A PDF – Based On Recent Exam Trends Second Week of January (9th January –16th January 2024)

165.Recently India’s first solar mission Aditya-L1 reached in a halo orbit around the L1 point, about __________million kilometres from Earth
from where it will orbit the Sun.
A. 2.5 million kilometres
B. 1.0 million kilometres
C. 1.9 million kilometres
D. 1.7 million kilometres
E. 1.5 million kilometres

 India’s first solar mission Aditya-L1 reached in a halo orbit around


the L1 point, about 1.5 million kilometres from Earth from where it
will orbit the Sun.
 The Lagrange point 1 (L1) of the Sun-Earth system is about one per
cent of the total distance between the Earth and the Sun.
 The Halo-Orbit Insertion (HOI) of Aditya-L1 was accomplished on
January 6.
 Halo orbit is that which moves around the L1 point with a size of six
lakh km in one dimension, over two lakh km in another dimension
and one lakh km in one more dimension, forming an egg-shaped
orbit.
 A Polar Satellite Launch Vehicle (PSLV-C57) launched the Aditya-L1
spacecraft from the second launch pad of Satish Dhawan Space
Centre (SDSC), Sriharikota, on September 2, 2023.
 Aditya-L1, weighing about 1,480.7 kg, is the first space-based Indian
observatory to study the Sun.
 Lagrange Points are the positions in space where a small object tends
to stay, if put there.
 These points in space for two body systems such as Sun and Earth can
be used by spacecraft to remain at these positions with reduced fuel
consumption.
 Lagrange Points are named in honour of Italian-French mathematician Joseph-Louis Lagrange.
 Of the five Lagrange points, three are unstable and two are stable.
 The unstable Lagrange points – labelled L1, L2, and L3 – lie along the line connecting the two large masses. The stable Lagrange points –
labelled L4 and L5 – form the apex of two equilateral triangles that have the large masses at their vertices.
 News Link - ISRO’s Aditya-L1 successfully placed in a halo orbit around L1 point - The Hindu

166.In January as per report _____________ has launched a new satellite 'Einstein Probe’ to observe cosmic phenomena.
A. UAE
B. Japan
C. China
D. Iran
E. Saudi Arabia

 China launched a new satellite 'Einstein Probe’ to observe cosmic


phenomena.
 The satellite named Einstein Probe (EP) has been launched from a
Long March-2C carrier rocket from the Xichang Satellite Launch
Centre in southwest China's Sichuan Province.

Click Here For Ultimate Bundle PDF Course 2022| Click Here to Subscribe Our Yearly Mock Test Package Follow Us- Telegram. Facebook , Twitter ,Instagram
Weekly Optimum Current Q & A PDF – Based On Recent Exam Trends Second Week of January (9th January –16th January 2024)

 This satellite will observe mysterious transient phenomena in the universe.


 It is shaped like a lotus in full bloom and features 12 petals and two stamens. Its weight is around 1.45 tonnes.
 The 12 'petals' of the satellite are 12 modules consisting of wide-field X-ray telescopes (WXT). The two stamens consist of two modules of
follow-up X-ray telescopes (FXT).
 Einstein's theory of general relativity predicted black holes and gravitational waves.
 Many fierce celestial activities such as explosions triggered by the death of supermassive stars, collisions between bizarre neutron stars
and black holes, etc. regularly occur in the Universe.
 These extraordinary explosions often emit X-rays, which cannot penetrate Earth's atmosphere
 The study of these events will help in better understanding the formation of the Universe.
 News Link - China launches new satellite 'Einstein Probe' shaped like a lotus to observe violent cosmic phenomena (deccanherald.com)

167.In January as per report debris from the Indian Air Force __________ aircraft was found near the Chennai coast, after it went missing over
the Bay of Bengal in 2016.
A. MiG -21
B. An-32
C. Sukhoi S-21
D. Chetak
E. Boeing - 107

 The Indian Air Force on January 12 said the debris


of the An-32 aircraft that went missing over the
Bay of Bengal in 2016 with 29 people onboard
has been found after seven years.
 An Autonomous Underwater Vehicle (AUV),
deployed by the National Institute of Ocean
Technology, located the missing aircraft on the
sea bed approximately 310 kilometres from the
Chennai coast.
 The search was conducted at a depth of 3,400 metres using several payloads, including a multi-beam SONAR (Sound Navigation and
Ranging).
 News Link - 7 years after IAF plane went missing, its debris found off Chennai coast | India News - The Indian Express

168.In January as per report Defence Research and Development Organisation (DRDO) has conducted a
successful flight-test of the New Generation AKASH (AKASH-NG) missile from the Integrated Test
Range (ITR), Chandipur off the coast of Odisha.
It is which type of missile?
A. Surface-To-Air Missiles
B. Air-to-air missiles
C. Surface-to-surface missiles
D. Ballistic Missile Defence (BMD)/Interceptor Missiles
E. Anti-Tank Missiles

 Defence Research and Development Organisation (DRDO) conducted a successful flight-test of the
New Generation AKASH (AKASH-NG) missile from the Integrated Test Range (ITR), Chandipur off the
coast of Odisha.
 It is surface-to-air new generation missile.
 The missile intercepted the high-speed unmanned aerial vehicle at a very low altitude.

Click Here For Ultimate Bundle PDF Course 2022| Click Here to Subscribe Our Yearly Mock Test Package Follow Us- Telegram. Facebook , Twitter ,Instagram
Weekly Optimum Current Q & A PDF – Based On Recent Exam Trends Second Week of January (9th January –16th January 2024)

 It is a new generation state-of-the-art surface-to-air missile (SAM) for the Indian Air Force (IAF) to
destroy high manoeuvring low radar cross section agile aerial threats.
 It can defend an area 10 times better compared to any short-range SAM and is capable of engaging up to 10 targets simultaneously.
 It can strike targets up to 40 km as against the earlier variant’s maximum range of 30 km.
 News Link - Press Information Bureau (pib.gov.in)

169.In January as per report Indian Landslide Susceptibility Map, first of its kind on a national scale, was prepared recently by ____________ to
identify high-risk areas for landslides, assess vulnerability and allocate resources effectively for mitigation.
A. IIT Kanpur
B. IIT Madras
C. IIT Delhi
D. IIT Bombay
E. IIT Bombay

 Indian Landslide Susceptibility Map, first


of its kind on a national scale, was
prepared recently by researchers at IIT
Delhi.
 In late 2023, devastating floods and
landslides struck North India due to
torrential rains during the northeast
monsoon.
 The aftermath emphasized the need for a
comprehensive approach to tackle
landslides, leading to a significant
initiative by IIT Delhi.
 The aim was to identify high-risk areas for
landslides, assess vulnerability and
allocate resources effectively for
mitigation.
 It covers the entire country as the first
comprehensive national-scale
assessment.
 It includes landslide-prone regions like the
Himalayan foothills and previously
unknown high-risk areas in parts of the
Eastern Ghats. Indian Landslide
Susceptibility Map with a 100m
resolution, set to be published in Catena,
is recognized as a significant contribution.
 News Link - IIT Delhi team makes first hi-res landslide risk map for India - The Hindu

170.Consider the following match the column in regards with space mission. Which of the following is matched incorrectly?
1. Europa Clipper- to explore Europa, one of Jupiter’s largest moons
2. Artemis II - aiming to send humans back to the moon and establish a sustained presence for future Mars missions.
3. VIPER- to explore the Venus south pole to search for water and other volatiles.
4. Lunar Trailblazer and PRIME-1 - to orbit the moon to map water locations, while PRIME-1 will test drilling technology.
5. Hera Mission - to study the Didymos-Dimorphos asteroid system

Click Here For Ultimate Bundle PDF Course 2022| Click Here to Subscribe Our Yearly Mock Test Package Follow Us- Telegram. Facebook , Twitter ,Instagram
Weekly Optimum Current Q & A PDF – Based On Recent Exam Trends Second Week of January (9th January –16th January 2024)

 The year 2023 proved to be an important one for


space missions, with NASA’s OSIRIS-REx mission
returning a sample from an asteroid and India’s
Chandrayaan-3 mission exploring the lunar south
pole region, and 2024 is shaping up to be another
exciting year for space exploration.
 Several new missions under NASA’s Artemis plan
and Commercial Lunar Payload Services initiative
will target the moon.
 The latter half of the year will feature several
exciting launches, with the launch of the Martian
Moons eXploration mission in September, Europa
Clipper and Hera in October and Artemis II and
VIPER to the moon in November – if everything
goes as planned.
 Europa Clipper: Unveiling Jupiter’s Moon
Europa
 Mission Context: NASA’s Europa Clipper aims to
explore Europa, one of Jupiter’s largest moons,
known for its icy surface and potential
subsurface saltwater ocean.
 Scientific Goals: The mission will conduct close
flybys to study Europa’s ice shell, geology, and
subsurface ocean, seeking signs of habitability.
 Launch Window: Scheduled for October 10, 2024,
with 21 days, aboard a SpaceX Falcon Heavy
rocket.
 Artemis II: NASA Plan To Return to the Moon
 Program Background: Artemis II is part of NASA’s
Artemis program, aiming to send humans back to
the moon and establish a sustained presence for
future Mars missions.
 Mission Details: Artemis II will carry four
astronauts on a 10-day mission orbiting the Moon,
building upon the uncrewed Artemis I mission.
 Launch Timeline: Planned for as early as
November 2024, with potential delays to 2025.
 VIPER: Searching for Lunar Water
 Mission Purpose: VIPER (Volatiles Investigating
Polar Exploration Rover), a golf cart-sized rover,
will explore the moon’s south pole to search for
water and other volatiles.
 Technical Challenges: The mission will navigate extreme lunar temperatures and shadowed regions during its 100-day mission.
 Launch Schedule: Set for November 2024, following a delay for additional lander system tests.
 Lunar Trailblazer and PRIME-1: Water Mapping and Drilling
 SIMPLEx Missions: As part of NASA’s low-cost planetary missions, Lunar Trailblazer will orbit the moon to map water locations, while
PRIME-1 will test drilling technology.
 Launch Dependencies: Both missions are secondary payloads, with their launch timing contingent on the readiness of primary payloads.
 JAXA’s Martian Moon eXploration (MMX) Mission
 Mission Focus: MMX aims to study Mars’ moons, Phobos and Deimos, to determine their origin and collect a sample from Phobos.
 Scientific Objectives: The mission will spend three years conducting science operations around Mars and its moons.
 Launch Plan: Scheduled for around September 2024.
Click Here For Ultimate Bundle PDF Course 2022| Click Here to Subscribe Our Yearly Mock Test Package Follow Us- Telegram. Facebook , Twitter ,Instagram
Weekly Optimum Current Q & A PDF – Based On Recent Exam Trends Second Week of January (9th January –16th January 2024)

 ESA’s Hera Mission: Asteroid Defense Study


 Mission Context: Hera will follow up on NASA’s DART mission to the Didymos-Dimorphos asteroid system, where DART tested the kinetic
impact technique for planetary defense.
 Research Goals: Hera will study the physical properties of the asteroids and assess the impact of the DART collision.
 Launch and Arrival: Set for October 2024, with arrival at the asteroid system expected in late 2026.
 News Link - From the moon to Europa, six space missions to be excited for in 2024 - The Hindu

Click Here For Ultimate Bundle PDF Course 2022| Click Here to Subscribe Our Yearly Mock Test Package Follow Us- Telegram. Facebook , Twitter ,Instagram

You might also like